PDA

Archiv verlassen und diese Seite im Standarddesign anzeigen : Konsequenze der Quantenmechanik


Mirko
06.04.12, 18:25
Sorry, hoffe es ist nicht im falschen Topic !

Hallo zusammen !

Zu aller erst möchte ich klar stellen, dass ich sowohl auf dem Gebiet der Quantenmechanik als auch der klassischen Mechanik absoluter Laie bin.

Es kostet mich allergrößte Mühe unter Anwendung meiner Oberstufenmathematik, welche nun auch schon Jahrzehnte hinter mir liegt, den Formalismus und die mathematischen Zusammenhänge nachzuvollziehen bzw. zu verstehen !
Tensorrechnung ist mir völlig unbekannt und, wie gesagt, Integral und Differentialrechnung auch schon einige Monde her :o

Nichts desto Trotz finde ich die Thematik allgemein und die Konsequenzen im Besonderen, faszinierend!
Ich bilde mir auch ein, dass ich grundlegende Dinge, außerhalb der mathematischen Formulierung, verstanden habe.
Für mich entsteht aber ein grundsätzliches Verständnisproblem!
Ich hoffe hier im Forum ist es möglich mir weiter zu helfen, falls es überhaupt eine Lösung für mein Unverständnis gibt:

Die laienhafte und unwissenschafliche, folgende Formulierung bitte ich zu entschuldigen: :)


Nehmen wir mal das Beispiel eines Elektrons:

Befindet sich ein Elektron in einem Quantenzustand, ist also Teil einer Wahrscheinlichkeitswelle, so kann es gleichzeitig in mehreren gleichberechtigten Orten existieren!
Durch eine Messung des Ortes oder des Impulses wird dieser Quantenzustand aufgehoben und es wird einer dieser gleichberechtigten Orte oder Impulse „angezeigt“
Das heißt es ist objektiv zufällig welcher Ort und/oder Impuls gemessen wird !
Soweit so gut !
Wenn man davon ausgeht, dass die menschlichen Sinne, also sehen, hören,schmecken usw.auch als Messung ( also als Aufhebung des Quantenzustands ) angesehen werden,müsste doch auch das Gesehene oder Gehörte oder Geschmeckte objektiv zufällig „daher kommen“, oder ?

Warum passieren aber immer genau die Dinge, die man erwartet.
Wenn Atome mit Quantenzuständen beschrieben werden, Moleküle aus Atomen und Materie aus Molekülen bestehen, warum schmeckt es IMMER sauer wenn ich an einer Zitrone lecke ?
Warum erscheint immer die Farbe grün, wenn ich blau und gelb überlagere ?

Langer Rede kurzer Sinn, warum passiert immer genau das, was ich erwarte ?

Hoffentlich konntet ihr meinem laienhaft formulierten Verständnisproblem folgen und vielleicht sogar lösen.

Danke euch fürs Lesen !

Ich
06.04.12, 21:06
Hi Mirko, willkommen im Forum!

Du hast da einiges durcheinandergebracht.
Nehmen wir mal das System "Rezeptor für sauer" und "Säuremolekül". Die Messung besteht darin, dass das Molekül an den Rezeptor bindet oder nicht. Ob das passiert oder nicht mag rein zufällig sein, die Wahrscheinlichkeit der entsprechenden Ereignisse lässt sich aber (im Prinzip, nicht in Echt!) berechnen. Das Ergebnis der Rechnung ist (zumindest bei Photonen) als Quanteneffizienz (http://de.wikipedia.org/wiki/Quanteneffizienz)bekannt. Sinnvollerweise ist sie möglichst hoch, so dass bei Anwesenheit vieler Säuremoleküle und vieler Säurerezeptoren auch viel Signal entsteht.
Trifft das Säuremolekül auf einen "Süßrezeptor", so ist die Quantenausbeute gering: auch wenn das Ergebnis zufällig ist, so ist es doch sehr unwahrscheinlich, dass das Säuremolekül an diesen Rezeptor bindet. Bei großen Mengen von Rezeptoren und Molekülen wird also sehr wenig Signal entstehen.
Das funktioniert also alles ganz klassisch, weil man es mit großen Zahlen zu tun hat. Wäre da nur ein Molekül und nur zwei Rezeptoren, dann könnte dir die Quantenmechanik tatsächlich einen Streich spielen. Sowas ignoriert das Gehirn aber, das reagiert nur auf starke Signale.

Mirko
07.04.12, 09:33
Hallo Ich !

Vielen Dank erstmal,
okay, ich kann nachvollziehen das im Makrokosmos Quanteneffekte gering bis gar nicht auftreten bzw. nicht spürbar sind !
Nur was wird den konkret gemessen wenn der Quantenzustand eines Elektrons "aufgehoben" wird oder im Moment der Messung verschwindet ?

Wenn dieses Elektron mehrere. gleichzeitige, teilweise gegensätzliche Zustände besitzt und sich im Moment der Messung exakt eine darstellt, was messe ich denn dann ?
Eine, von vielen möglichen Eigenschaften ?

Nun, es ist noch alles schwierig für mich, aber ich bin ehrgeizig und ich werde mich reinlesen, soweit meine Zeit es zulässt.
Danke euch hier im forum sein zu dürfen.
Werde wohl eher im Hintergrund agieren und interessiert euren Beiträgen und Diskussionen lauschen.......

Könnte nur sein, dass ich ab und an mal ne blöde Frage stelle, habt ihr ja schon gemerkt :D

Bauhof
07.04.12, 12:07
Wenn dieses Elektron mehrere. gleichzeitige, teilweise gegensätzliche Zustände besitzt und sich im Moment der Messung exakt eine darstellt, was messe ich denn dann ? Eine, von vielen möglichen Eigenschaften ?
Hallo Mirko,

einem Elektron kann man vor der Messung überhaupt keinen Zustand zusprechen.
Erst durch die Konfiguration der Messanordnung wird festgelegt, welcher Aspekt des Elektrons nach der Messung in Erscheinung tritt.

M.f.G. Eugen Bauhof

Mirko
07.04.12, 13:22
Hallo Bauhof ,

weiter gedacht müsste es doch heissen, ich stelle den Zustand des Elektrons durch die Messung erst her !
Und wenn du schreibst durch die Konfiguration der Messung kann ich das Ergebnis der Messung exakt bestimmen, warum messe ich dann überhaupt noch ???

Das Ergebnis ist mir doch bereits bekannt, eben durch die Versuchsanordnung !

Hoffe das klingt für euch hier nicht alles zu naiv........aber ich bin gezwungen mit meinem gesunden Menschenverstand an die Sachlage ran zu gehen.

By the way.......macht Spass im Forum zu lesen......hochinteressant und spannend !

Bauhof
07.04.12, 14:22
Das Ergebnis ist mir doch bereits bekannt, eben durch die Versuchsanordnung !
Hallo Mirko,

nein.
Das Ergebnis ist vor der Messung unbestimmt. Durch die entsprechende Konfiguration der Messanordnung wird nur festgelegt, welche komplementären Aspekte des Teilchens man messen will. Ob man z.B. den Impuls oder den Ort des Teilchens messen will. Beides zugleich geht nicht.

Der Ort oder der Impuls des Teilchens ist erst nach der Messung bekannt. Vor der Messung kann man dem Teilchen weder einen Ort noch einen Impuls zusprechen. Auch die Versuchsanordnung ändert daran nichts.

M.f.G. Eugen Bauhof

P.S.
Der gesunde Menschverstand scheitert an der Quantentheorie.

Mirko
07.04.12, 14:28
Ja stimmt,
da habe ich dich missverstanden ! Die Versuchsanordnung klärt welchen Aspekt ich bestimmen will und die Messung an sich das Ergebnis dieses Aspektes !

Trotzdem danke für deine Geduld !;)

RoKo
07.04.12, 18:34
Hallo Bauhof,

warum erzählst du interessierten Laien immer deine ideologischen Märchen?

..
einem Elektron kann man vor der Messung überhaupt keinen Zustand zusprechen.
Erst durch die Konfiguration der Messanordnung wird festgelegt, welcher Aspekt des Elektrons nach der Messung in Erscheinung tritt.


Selbstverständlich befindet sich ein Elektron stets in einem Zustand.

eines beliebigen QM-Skripts:
Postulat 1: Der Zustand eines quantenmechnischen Systems ist vollstandig durch die Wellenfunktion Psi(x,t) bestimmt.

Mirko
07.04.12, 18:41
Na watt denn nu ??:D

Auch wenn ich keine Forenkrieg vom Zaun brechen mag, halte ich meine Frage mal sehr allgemein und stelle sie erneut

Warum sind die Dinge stets so wie wir sie erwarten. Zumindest vom rein logischen Aspekt aus, wenn doch quantenmechanisch alles objektiv zufällig ablauft !

MCD
08.04.12, 01:09
Ob man z.B. den Impuls oder den Ort des Teilchens messen will. Beides zugleich geht nicht.


Hi Bauhof,

ist das so?:eek:

Gr.
MCD

Bauhof
08.04.12, 09:21
Hi Bauhof, ist das so?:eek: Gr. MCD
Hallo MCD,

es ist so.
Beide Aspekte gleichzeitig mit beliebiger Schärfe messen geht nicht. --> Heisenbergsche Unbestimmtheitsrelation.

M.f.G. Eugen Bauhof

Timm
08.04.12, 10:30
Selbstverständlich befindet sich ein Elektron stets in einem Zustand.
Vorsicht RoKo, so salopp gehts nicht. Die EPR Experimente zeigen, daß etwa der Spin eines Teilchens vor der Messung nicht definiert ist.

Einen wenngleich bizarren Ausweg bietet die Viele-Welten-Interpretation. Dann sollte man das aber dazu sagen.

Gruß, Timm

MCD
08.04.12, 10:47
Hallo MCD,

es ist so.
Beide Aspekte gleichzeitig mit beliebiger Schärfe messen geht nicht. --> Heisenbergsche Unbestimmtheitsrelation.

M.f.G. Eugen Bauhof

Hallo Bauhof,

so (mit Hinweis auf die Unschärfe) hört sich das ja auch schon anders an.
Ohne diese Einschränkung ist deine Aussage

Zitat von Bauhof:
Ob man z.B. den Impuls oder den Ort des Teilchens messen will. Beides zugleich geht nicht.

irritierend bzw. falsch, da man sehr wohl Impuls und Ort gleichzeitig messen kann.

Gr.
MCD

amc
08.04.12, 10:53
Hi Mirko,

Warum sind die Dinge stets so wie wir sie erwarten. Zumindest vom rein logischen Aspekt aus, wenn doch quantenmechanisch alles objektiv zufällig ablauft !

die Ereignisse, zwischen denen der quantenmechanische Zufall entscheidet, geschehen objektiv zufällig. Aber nicht alle Möglichkeiten sind auch gleich wahrscheinlich. Z.B. wird man ein Elektron im Normalfall so gut wie niemals im Atomkern antreffen, weil hierfür die Wahrscheinlichkeit sehr nahe bei Null liegt.

Vielleicht hilft das etwas, bin auch nur Laie.

Grüße, AMC

RoKo
08.04.12, 12:03
Hallo Timm,

Vorsicht RoKo, so salopp gehts nicht. Die EPR Experimente zeigen, daß etwa der Spin eines Teilchens vor der Messung nicht definiert ist.
Das ändert nichts daran, dass sich ein quantenmechanisches System in einem Zustand befindet. Zustand bedeutet ja nicht, dass Eigenschaften, die man einem solchen System aufzwingen kann, definitiv vorliegen. Mit dem Spin bezüglich einer Raumrichtung verhält es sich so wie mit der Polarisation bei Photonen - im Normalfall undefiniert. Durch einen experimentellen Eingriff, der (abweichend von den üblichen Geflogenheiten) in der Quantenmechanik "Messung" heisst, wird dem System eine Polarisation bzw. Spinrichtung aufgezwungen.

Im Falle der EPR-Experimente haben wir es übrigens mit mindestens zwei verschränkten "Teilchen" zu tun. Durch einen experimentellen Eingriff wird dem System als Ganzes eine Polarisation bzw. Spinrichtung aufgezwungen. Photonen (Bosonen) verhalten sich dabei symmetrisch; Elektronen (Fermionen) antisymmetrisch.

Bei genauerer Betrachtung der mathematischen Struktur der Wellenfunktion ist der Zustand eines quantenmechanischen Systems nur ein komplexwertiger energetischer Zustand. Ein Elektron z.B. hat eine Ruhemasse, eine Ladung, eine Spin-Zahl und ggf. zusätzliche Energie. Mehr Eigenschaften hat es nicht. Alles andere sind kontextuelle Eigenschaften, die mit der "Messung" dem Elektron aufgezwungen werden. (siehe auch Kochem-Specker-Theorem).

Bauhof
08.04.12, 13:14
... warum erzählst du interessierten Laien immer deine ideologischen Märchen?
Hallo RoKo,

erspare uns bitte in Zukunft deine Polemik.

Selbstverständlich befindet sich ein Elektron stets in einem Zustand.
So einfach ist das leider nicht. Ich weiß nicht, was du mit 'Zustand' genau meinst, ich unterstelle mal, dass du dir vorstellst, dass man dem Elektron in jedem Augenblick eine wohldefinierte Position und eine wohldefinierten Impuls zusprechen kann. Das ist nicht der Fall. Anton Zeilinger formuliert das in seinem Buch [1] auf Seite 84 wie folgt;

Hier sind wir bei einem sehr wichtigen Punkt angelangt, der immer wieder betont werden muss: Wir müssen jedem einzelnen Elektron ein Wellenpaket zuordnen. Das Elektron trägt daher nicht einen bestimmten Impuls, also eine wohl definierte Geschwindigkeit, und es ist nicht an einem wohldefinierten Ort lokalisiert. Das Wellenpaket drückt sowohl die Unschärfe der Geschwindigkeit als auch die Unschärfe des Ortes aus. Wenn wir uns entscheiden, den Ort zu messen, werden wir das Elektron irgendwo innerhalb des Wellenpakets finden. Die Unschärfe des Ortes des Elektrons ist plötzlich sehr viel kleiner geworden. Das Elektron, sagen die Physiker, ist durch die Messung lokalisiert worden.

Das Experiment hat also nicht einen Ort ergeben, an dem sich das Elektron vor der Messung befand. Durch diese Messung hat das Elektron aber auch eine viel größere Impulsunschärfe erhalten, eine viel größere Unscharfe in seiner Geschwindigkeit. Alles, was wir vorher hatten, war das Wellenpaket, das lediglich vorschreibt, wo das Elektron mit welcher Wahrscheinlichkeit gefunden werden könnte, mehr nicht.

Zeilinger schreibt weiter auf Seite 92:

Wir haben eben erfahren, dass die Heisenbergsche Unschärfebeziehung allen Versuchen, den Quantenzustand eines Teilchens, etwa eines Elektrons, vollständig zu bestimmen, ein Ende setzt. Um es kurz zu wiederholen:

Wir müssen entscheiden, was wir messen wollen, beispielsweise den Ort oder den Impuls, und wenn wir dann eines von beiden messen, bekommen wir die entsprechende Antwort, die uns im Allgemeinen nicht einmal sagt, dass das Teilchen diese Eigenschaft vor der Beobachtung hatte. Außerdem hat sich der Zustand des Teilchens durch die Beobachtung sehr verändert.

Das heißt, wir dürfen dem Elektron vor der Messung nicht irgendeinen Zustand zusprechen.

Mit freundlichen Grüßen
Eugen Bauhof

[1] Zeilinger, Anton
Einsteins Spuk.
Teleportation und anderer Mysterien der Quantenphysik (http://www.amazon.de/dp/3570006913)
München 2005. ISBN=978-3-570-00691-7

Bauhof
08.04.12, 13:27
Hallo Bauhof, so (mit Hinweis auf die Unschärfe) hört sich das ja auch schon anders an. Ohne diese Einschränkung ist deine irritierend bzw. falsch, da man sehr wohl Impuls und Ort gleichzeitig messen kann.
Hallo MCD,

ja.
Aber beide gleichzeitig nicht mit beliebiger Schärfe.

Außerdem kann man doch Mirko (offenbar ein "Quantenbeginner") nicht sofort in epischer Breite die Quantentheorie erklären wollen. Es genügen doch vorerst einfache, wenn auch unscharfe Erklärungen. Die Feinheiten kann er im Moment sowieso noch nicht erfassen.

M.f.G. Eugen Bauhof

Mirko
08.04.12, 13:52
Außerdem kann man doch Mirko (offenbar ein "Quantenbeginner") nicht sofort in epischer Breite die Quantentheorie erklären wollen. Es genügen doch vorerst einfache, wenn auch unscharfe Erklärungen. Die Feinheiten kann er im Moment sowieso noch nicht erfassen.

M.f.G. Eugen Bauhof


Ich befürchte du hast Recht :D

Nur wenn man das ganze Thema mal losgelöst und/oder laienhaft betrachtet, dann gehen die Dinge doch fast schon ins Philosophische,oder ?

Die Interpretationen dieser quantenmechanischen Effekte und deren ,teilweise sich widersprechenden Erkenntnisse sind ja, soweit ich es beurteilen kann, wissentschaftlich äusserst schwierig bis gar nicht zu erklären !

Vieleicht geht es wirklich in die Richtung, Wissenschaftsübergreifend argumentieren zu müssen !
Natur-und Geisteswissenschaften hängen möglicherweise enger zusammen als bisher vermutet !

.......oder so! :-)

RoKo
08.04.12, 16:22
Hallo Bauhof,

.. Das heißt, wir dürfen dem Elektron vor der Messung nicht irgendeinen Zustand zusprechen.

Ein quantenmechanischer Zustand ist kein klassischer Zustand. Ich hatte das bereits in der Antwort an Timm erläutert. Der quantenmechanische Zustand sollte nicht mit Eigenschaften, die bei einer Messung produziert werden, verwechselt werden.

Das Problem deiner Missverstände meiner Beiträge liegt in deinem Verständnis von Messung. Nach (nicht nur meinem) Verständnis von Messung handelt es sich dabei um einen physischen Vorgang, bei dem (mindestens) zwei Objekte miteinander in Wechselwirkung treten, womit ggf. eine weitere Dynamik angestossen wird. Eine "Ortsmessung" findet z.B. auch beim Beta-Zerfall durch K-Einfang statt, durch den wir dann über die Zerfallsprodukte indirekt Auskunft erhalten. "Messung" ist also keinesfalls an menschliche Beobachter gebunden. Darüber hinaus ist, anders als in allen anderen Bereichen, eine quantenmechanische Messung keine Feststellung von "an sich" vorhandenen Eigenschaften, sondern es ist ihre Herstellung. (Leider können sich diverse Physiker nicht dazu durchringen, diesen hier völlig unangebrachten Begriff zu ersetzen.)

Timm
08.04.12, 17:44
Hi RoKo,


Ein quantenmechanischer Zustand ist kein klassischer Zustand. Ich hatte das bereits in der Antwort an Timm erläutert. Der quantenmechanische Zustand sollte nicht mit Eigenschaften, die bei einer Messung produziert werden, verwechselt werden.

Du hast überwiegend geschrieben, was der quantenmechanische Zustand nicht ist, worüber keine Uneinigkeit besteht. Und wer wollte die Komplexwertigkeit abstreiten, oder daß dieser Zustand nicht klassisch ist?

Hier zu Deiner Info zum Zustand (Quantenmechanik) (http://de.wikipedia.org/wiki/Zustand_(Quantenmechanik)):
In der quantenmechanischen Behandlung eines physikalischen Systems ist der momentane Zustand des Systems ein mathematisches Objekt, mit dem für jede am System mögliche (fehlerfreie) Messung und für jedes der dabei möglichen Messergebnisse die Wahrscheinlichkeit berechnet werden kann, mit der das betreffende Messergebnis erhalten wird.

Dies beispielhaft, in anderen Quellen steht's ähnlich.

Gruß, Timm

Philipp Wehrli
08.04.12, 18:26
Na watt denn nu ??:D

Auch wenn ich keine Forenkrieg vom Zaun brechen mag, halte ich meine Frage mal sehr allgemein und stelle sie erneut

Warum sind die Dinge stets so wie wir sie erwarten. Zumindest vom rein logischen Aspekt aus, wenn doch quantenmechanisch alles objektiv zufällig ablauft !
Die Antwort ist hier bereits gegeben worden, sie ist nur zwischen den Abschweifungen untergegangen. Die Unbestimmtheit ist ein sehr kleiner Effekt, der extrem rasch verschwindet, sobald viele Teilchen beteiligt sind (sogenannte Dekohärenz). Wir haben hier schon länger darüber gestritten, ob der Effekt nach dem Verschwinden nicht mehr existiert (Kollapshypothese) oder ob er nur für uns unsichtbar wird, weil er in einer anderen Welt liegt (Viele-Welten Interpretation). Für deine Frage ist dies aber irrelevant: Wir sind uns einig, dass der Effekt nicht mehr sichtbar ist.

Bei der menschlichen Wahrnehmung sind immer extrem viele Teilchen beteiligt. Deshalb kannst du unmöglich eine Überlagerung mehrerer Zustände wahrnehmen, selbst wenn ein einzelnes Molekül noch in einer Überlagerung existieren könnte, von denen der eine sauer und der andere neutral wäre.

richy
08.04.12, 18:36
Am Anfang war das Chaos, Zufall. Und ohne Sebstorganistation gaebe es nur dieses.Der erste Schritt aus dem Chaos ist die Gravitation.Denn diese fuehrt zu Materieansammlung und Verdichtung. Kernfusion, die die noetige Energie fuer die Selbstorganisation, Ordnung liefert.
Die Erde ist kein Maßstab und abgesehen von einer sich vernetzenden Strurktur herrscht im Universum noch Chaos, Zufall.
Gruesse

Hawkwind
09.04.12, 13:30
Warum sind die Dinge stets so wie wir sie erwarten. Zumindest vom rein logischen Aspekt aus, wenn doch quantenmechanisch alles objektiv zufällig ablauft !

Wenn ich recht verstehe, wunderst du dich darüber, dass die Quantentheorie überhaupt überprüfbare quantitative Vorhersagen für Messungen macht?

Das tut sie, indem sie Erwartungswerte (d.h. wahrscheinlichste Werte) für Messungen, aber auch "Dinge" wie die Streuung der Messwerte ("Unschärfen") um diese Erwartungswerte herum, vorhersagt. Wenn man das Experiment nun hinreichend oft wiederholt, wird man halt sehen, ob man meist etwas in der Nähe des vorhergesagten Erwartungswertes gemessen hat. Wenn nicht, dann schaut es schlecht aus oder die vorhergesagte Streuung ("Unschärfe") ist tatsächlich so groß, dass es problematisch ist, einen wahrscheinlichsten Wert überhaupt zu definieren bzw. zu finden. Aber selbst das lässt sich experimentell überprüfen: in so einem Fall misst man beliebige Werte.
Manchmal werden auch "scharfe" Werte von der Quantentheorie vorhergesagt (Unschärfe = 0) ; in so einem Fall ist die Wahrscheinlichkeit, den vorhergesagten Wert zu messen, gleich 100 %.

Harti
10.04.12, 12:03
Hallo zusammen,


Das ändert nichts daran, dass sich ein quantenmechanisches System in einem Zustand befindet. Zustand bedeutet ja nicht, dass Eigenschaften, die man einem solchen System aufzwingen kann, definitiv vorliegen. Mit dem Spin bezüglich einer Raumrichtung verhält es sich so wie mit der Polarisation bei Photonen - im Normalfall undefiniert. Durch einen experimentellen Eingriff, der (abweichend von den üblichen Geflogenheiten) in der Quantenmechanik "Messung" heisst, wird dem System eine Polarisation bzw. Spinrichtung aufgezwungen.

diese Äußerung von RoKo hat mich als Laien stutzig gemacht und zu einer Frage geführt, die ich am Ende stellen will.

Zur Darstellung meines Vorstellungsproblems konstruiere ich ein Beispiel:

Vor mir auf dem Tisch habe ich zwei Eier stehen (Oster ist ja gerade vorbei). Aus meiner Beobachterposition (Perspektive, Bezugssystem) ist das eine das linke Ei und das andere das rechte. Ich gehe um den Tisch herum und betrachte die Eier aus der entgegengesetzten Richtung. Jetzt haben die Eier die Eigenschaft rechts und links getauscht. Die Eier befinden sich folglich, solange ich als Beobachter das Bezugssystem darstelle, bezüglich der Eigenschaft rechts/links in einer Überlagerungsposition. Sodann markiere ich die Eier bei einer Betrachtung, schreibe also auf das eine "links" und das andere "rechts". Damit mache ich das Gesamtsystem "Eier" zum Bezugssystem und egal, aus welcher Position ich sie betrachte, sie bleiben bezüglichg der Ausrichtung links/rechts gleich.

Meine Fragen nun:
Handelt es sich bei den Zuständen von Quanten (oder größeren Objekten), die verschränkt sein können um solche, die durch die Perspektive, das Bezugssystem bestimmt werden (links/rechts;horizontal/vertikal;plus/minus).

Kann man das von RoKo definierte "quantenmechanische Messen" mit der von mir beispielhaft aufgeführten Kennzeichnung beschreiben.

Ich hoffe mal, meine Fragen passen noch zum Thema. Gegen eine Verschiebung habe ich natürlich auch nichts einzuwenden.

MfG
Harti

Bauhof
10.04.12, 13:47
Meine Fragen nun: Handelt es sich bei den Zuständen von Quanten (oder größeren Objekten), die verschränkt sein können um solche, die durch die Perspektive, das Bezugssystem bestimmt werden (links/rechts;horizontal/vertikal;plus/minus). Kann man das von RoKo definierte "quantenmechanische Messen" mit der von mir beispielhaft aufgeführten Kennzeichnung beschreiben.
Hallo Harti,

das kann man nicht.
Die quantenmechanische Verschränkung hat nichts mit Bezugssystemwechseln zu tun.
Ich hoffe mal, meine Fragen passen noch zum Thema. Gegen eine Verschiebung habe ich natürlich auch nichts einzuwenden.
Ich verschiebe deinen Beitrag nicht, denn dann wüsste ich nicht mehr, wo ich mit dem Verschieben anfangen und aufhören soll. Da sind einfach zu viele Beiträge, die nicht zum Thema passen. Nicht unbedingt in diesem Thread, sondern ganz allgemein.

M.f.G. Eugen Bauhof

RoKo
10.04.12, 17:14
Hallo Timm,

..Du hast überwiegend geschrieben, was der quantenmechanische Zustand nicht ist ..
Da hast du leider nicht genau gelesen. Ich hatte geschrieben:
Bei genauerer Betrachtung der mathematischen Struktur der Wellenfunktion ist der Zustand eines quantenmechanischen Systems nur ein komplexwertiger energetischer Zustand. Ein Elektron z.B. hat eine Ruhemasse, eine Ladung, eine Spin-Zahl und ggf. zusätzliche Energie. Mehr Eigenschaften hat es nicht.Hier zu Deiner Info .. Dies beispielhaft, in anderen Quellen steht's ähnlich.In fast allen ernsthaften Quellen (Skripten von Hochschulserven, von quantph oder den Beiträgen von Standford Univerity) und wird das 1.Postulat so formuliert, wie ich es bereits zitiert habe:Postulat 1: Der Zustand eines quantenmechnischen Systems ist vollstandig durch die Wellenfunktion Psi(x,t) bestimmt.

Hawkwind
10.04.12, 18:10
Hallo Timm,

Da hast du leider nicht genau gelesen. Ich hatte geschrieben:
In fast allen ernsthaften Quellen (Skripten von Hochschulserven, von quantph oder den Beiträgen von Standford Univerity) und wird das 1.Postulat so formuliert, wie ich es bereits zitiert habe:

Der quantenmechanische "Zustand" ist nichts als ein Synonym für die Wellenfunktion, die ja auch manchmal "Zustandsfunktion" oder "Zustandsvektor" genannt wird.
Und nicht zu verwechseln mit anderen Verwendungen des Begriffes "Zustand" in der Physik (z.B. Thermodynamik), wo sich der Zustand als eine Kombination von Messgrößen ergibt.
Das ist ja in der Quantenmechanik anders: all diese Größen legen den quantenmechanischen Zustand immer noch nicht eindeutig fest, denn man hat immer die Freiheit, einen komplexen Phasenfaktor einzuführen, wodurch sich die Physik nicht ändert.

Neben den erwähnten Größen können übrigens - je nach Kontext - weitere Quantenzahlen benötigt werden um einen Zustand zu charakterisieren; bei den Lösungen der Schrödingergleichung für das H-Atom hat man z.B. die Quantenzahlen n, l und m: n=Hauptquantenzahl (Energie), l=Bahndrehimpulsbetrag und m=magnetische Quantenzahl (z-Komponente des Bahndrehimpulses). Berücksichtigt man den Spin des Elektrons, kommt der noch hinzu.
In der Teilchenphysik können Quantenzahlen wie Leptonenzahl, Isospin, ... hinzukommen.

JoAx
10.04.12, 18:42
Servus allerseits!

Na watt denn nu ??:D


"Beides" würde ich sagen. Was RoKo imho meint, ist der quantenmechanische "Zustand". Was Eugen meint (und du vermutlich auch), ist der "klassische" Wert einer "Observablen".

"scharfer" Ort <=> "verschmiert" über einen Gebiet. (z.B.)


Warum sind die Dinge stets so wie wir sie erwarten. Zumindest vom rein logischen Aspekt aus, wenn doch quantenmechanisch alles objektiv zufällig ablauft!

Zufall ist nicht der Willkür gleich zu setzen. Es ist ein Zufall "innerhalb eines bestimmten Rahmens", könnte man vlt. sagen. Ob diese Feststellung weiter hilft, ist eine andere Frage. :)


Gruß, Johann

RoKo
10.04.12, 21:02
Hallo Mirko,

ich versuche mal, dein Problem zu klären.

Ich habe dein Problem so verstanden, dass du wissen willst, warum es einerseits in der Quantenmechanik einen objektiven Zufall gibt, unsere makroskopische Welt sich jedoch meist berechenbar verhält.

Die gängige Antwort hast du bereits erhalten: Die Heisenbergsche Unbestimmtheitsrelation ist so klein, dass sie makroskopisch keine Rolle spielt.

Man kann sich mit dieser Erklärung zufrieden geben; zum wirklichen Verständnis trägt es jedoch (m.E !!) nichts bei.

Um die wirklichen Vorgänge zu erklären, muss ich jedoch etwas weiter ausholen. Ferner muss ich dich darauf hinweisen, dass einige es als Tabubruch ansehen, Quantenmechanik verständlich zu machen.

1. Gedanke:
In der klassischen physikalischen Mechanik wird Alles auf seinen Schwerpunkt reduziert. Damit lässt sich erstens einfach rechnen und mit z.B. Stahlkugeln lässt sich auch experimentell nachweisen, dass es mit der klasischen Mechanik seine Richtigkeit hat. Ein klassischer Zustand ist dann schlicht durch Ort und Impuls des Schwerpunktes bestimmt.

Nun machen wir ein Doppelexperiment. Wir nehmen zwei Eimer Wasser und lassen sie von einem Balkon fallen; den ersten mit dem Boden nach unten, den zweiten mit der Öffnung nach unten. Beim ersten Eimer liegen wir einigermaßen richtig, wenn wir ihn auf seinen Schwerpunkt reduzieren. Um den zweiten Eimer richtig zu beschreiben, müssen wir jedoch den Standpunkt, dass es sich um ein einheitliches Objekt handelt, aufgeben und jedes Wassermolekül einzeln betrachten. Hier hat nun jedes Wassermolekül seinen eigenen durch Ort und Impuls gegeben Zustand.

1. Anmerkung
Doch Vorsicht! Gemäß der positivistischen Anschauung, die hier manchmal vertreten wird, dürfen wir dem einzelnen Wassermolekül keinen Zustand zusprechen, solange wir ihn nicht gemessen haben. :D

2. Gedanke:
Bis zum Dezember des Jahres 1900 ging man davon aus, dass alle physikalischen Größen beliebig klein sein könnten. Max Planck sah sich jedoch zur Erklärung der Temperaturstrahlung gezwungen, eine kleinste Wirkung anzunehmen. Die Quantenphysik wurde geboren.

3. Gedanke:
Über Einstein (Lichtquanten-Hypothese) und de Broglie (Materie-Wellen) kommt Schrödinger dann zu seiner Gleichung, deren Lösung eine komplexwertige Wellenfunktion ist. Ursprünglich war sie nur dazu gedacht, das Verhalten von Elektronen im Atom zu erklären, aber Born wendete sie auf seine Streuversuche an und schlussfolgert, dass das Absolutquadrat der Wellenfunktion ein Mass für die Wahrscheinlichkeit einer lokalen Wechselwirkung ("das Elektron zu finden" bzw. zu "messen") ist.

4. Gedanke:
Eine komplexwertige Wellenfunktion war damals (und heute?) ein Novum. In der Wechselstromlehre sind komplexwertige Wellenfunktionen üblich. Elektromagnetische Wellen lassen ebenso als komplexwertige Wellenfunktion darstellen. Andererseits lässt sich die quantenmechanische Wellenfunktion auch in Real- und Imaginarteil zerlegen. Tut man das, dann hat man erhält man in Analogie zur Elektrotechnik bzw. zum Licht die Vorstellung, dass eine Impulsänderung eine Ortsveränderung bewirkt und umgekehrt. (Achtung: der letzte Satz ist meine Vorstellung!)

5. Gedanke:
Bis zum Jahre 1963 bzw. bis zum Jahre 1980 war Lokalität in der Physik sakrosant. 1963 zeigte John Bell theoretisch, 1980 Allan Aspect auch empirisch, dass die Quantenmechanik nichtlokal ist.

Zusammenfassung/Auflösung:
A) Quantenmechanische Objekte sind ausgedehnte Objekte und können verschiedene Formen haben. Man betrachte beispielsweise die verschiedenen Formen von Elektronen im Atom bei unterschiedlichen Energieniveaus. Bei atomgebundenen Elektronen haben wir es jedoch mit shpärischen stehenden Wellen zu tun. Die Energie, die das Elektron als ganzes hat, kann man berechnen. Da das Ganze jedoch ein ausgehntes Objekt ist, also einen Raum einnimmt, muss man (sie Gedanke 1 - Eimer Wasser) jedem Raumpunkt etwas von dieser Energie zuordnen. Das Objekt hat also eine Dichte bzw. Energie-Intensität; diese ist gleich dem Absolutquadrat der Wellenfunktion an diesem Raumpunkt.
B) Im Atom ist alles schön geordnet. Doch sobald unser Elektron das Atom verlässt, was nur durch zusätzlichen Energieübertrag möglich ist, wird es zu einer sich ausbreitenden Welle, die auf Grund unterschiedlicher Frequenzanteile zerfliesst. Es wird also noch ausgedehnter. So kann es auch locker einen Doppelspalt passieren - es geht eben durch beide Spalte! Dahinter interferiert es mit sich selbst, wodurch es sich auch noch in verschiedene Räume aufteilt, wo es ist und wo es nicht ist. Alles kein Problem. Sofern die Wellenfunktion bekannt ist, kann man das alles exakt berechnen.
C) Das Problem beginnt dann, wenn man nun hinter dem Doppelspalt viele Detektoren aufstellt, um das Elektron zu messen. Jetzt müssen wir uns des Gedankens zwei erinnern - es gibt eine kleinste Wirkung. Und da unser Elektron ein Quantenobjekt ist, kann es nur einmal wirken. Dieses ausgedehnete Objekt trifft nun mehr oder weniger gleichzeitig auf viele Atome (wo alles schön geordnet = lokalisiert ist) , mit denen es in Wechselwirkung treten könnte. Und genau hier kommt nun der Zufall ins Spiel. Die Wahrscheinlichkeit, dass ein Quantenobjekt an einem Raumpunkt eine Wechselwirkung eingeht, ist proportional zur Dichte des Quantenobjektes an diesem Raumpunkt. Alles weitere Geschehen ergibt sich dann aus der bedingten Wellenfunktion; d.h. dem quantenmechanischen Zustand an diesem Raumpunkt.

Bei genauerer Betrachtung haben wir es also einerseits mit einem Zufall zu tun, der aber andererseits auch berechenbar ist.

D) Die Schwierigkeit, Quantenmechanik zu verstehen, ist im Grunde eine historische. Weil - siehe Gedanke 5 - ürsprünglich das Prinzip der Lokalität als sakrosant galt, war für die Gründerväter vieles unverständlich. Und auch heute noch ist das reale Geschehen deshalb strittig, weil die Theorie (= Schrödingergleichung) das Geschehen bei einer lokalen Wechselwirkung nicht beschreibt.

amc
10.04.12, 21:08
Warum sind die Dinge stets so wie wir sie erwarten. Zumindest vom rein logischen Aspekt aus, wenn doch quantenmechanisch alles objektiv zufällig ablauft !

Mal dazu ein Zitat (Witz) aus "The Big Bang Theory":

Sitz ein Quantenphysiker alleine im Eiscafe und bestellt, wie jeden Tag, zwei Eisbecher. Da fragt ihn der Kellner: "Warum bestellen Sie denn immer zwei Eisbecher und essen doch nur einen?" Der Physiker: "Ja wissen Sie, nach der Quantenphysik ist es theoretisch möglich, dass sich dieser Stuhl hier augenblicklich in eine wunderschöne Frau verwandelt, und wenn dies geschieht, dann möchte ich vorbereitet sein und der Frau einen Eisbescher anbieten können." Der Kellner: "Warum fragen Sie denn nicht einfach eine von den schönen Frauen, die hier ständig im Cafe sind, vielleicht möchte ja eine mit Ihnen ein Eis essen?" Der Physiker: "Und wie wahrscheinlich ist es, dass das passiert?"

;)

Grüße, AMC

RoKo
10.04.12, 21:24
.. Das ist ja in der Quantenmechanik anders: all diese Größen legen den quantenmechanischen Zustand immer noch nicht eindeutig fest, denn man hat immer die Freiheit, einen komplexen Phasenfaktor einzuführen, wodurch sich die Physik nicht ändert.
Sic!

Aber auch das ändert nichts daran, dass sich ein quantenmechanisches Objekt stets in einem Zustand befindet. JoAx hat es zuvor etwas "salomonisch" ausgedrückt: von JoAx: Zufall ist nicht der Willkür gleich zu setzen. Es ist ein Zufall "innerhalb eines bestimmten Rahmens" Um im Bilde zu bleiben, mit dem Phasenfaktor wird der Rahmen nur minimal verschoben.

JoAx
11.04.12, 09:19
dass sich ein quantenmechanisches Objekt stets in einem Zustand befindet.


Ja gut, Rolf. Ein beschleunigtes Objekt hat auch stets eine momentane Geschwindigkeit.
Bringt das weiter? Vlt.

Mirko
11.04.12, 10:52
Guten Morgen,

erstmal recht herzlichen Dank für die zahlreichen Antworten und Denkanstösse ! !
Ich werde versuchen meine Gedanken zu ordnen und mich weiter in die Materie einlesen.

Die nächsten Fragen werden sicher kommen.........

Hawkwind
11.04.12, 11:01
Sic!

Aber auch das ändert nichts daran, dass sich ein quantenmechanisches Objekt stets in einem Zustand befindet. JoAx hat es zuvor etwas "salomonisch" ausgedrückt: Um im Bilde zu bleiben, mit dem Phasenfaktor wird der Rahmen nur minimal verschoben.


Roko, ich war mir schon bewusst, dir nicht unbedingt zu widersprechen. :)

Es kommt halt drauf an, was man unter einem "Zustand" versteht. Für Eugen ist das anscheinend ein Punkt im Phasenraum (d.h. Impuls- und Ortsvektor eines teilchens), wogegen du vom quantenmechanischen Zustandsvektor sprichst, wenn ich recht verstehe. So gesehen, hat jeder recht.

Gruß

JoAx
11.04.12, 12:37
1. Anmerkung
Doch Vorsicht! Gemäß der positivistischen Anschauung, die hier manchmal vertreten wird, dürfen wir dem einzelnen Wassermolekül keinen Zustand zusprechen, solange wir ihn nicht gemessen haben. :D


Gegenanmerkung:
Du bringst ein Beispiel, bei dem klassische Physik (= gewohnter Anschauungsrahmen) zuständig ist, und dann gehst du ganz ungezwungen zur QM? :eek:
Beschreibe doch zunächst genau so ungezwungen den Eimer mit Wasser darin - quantenmechanisch. Bitte!
Ansonsten sehe ich auch nicht, warum sich die beiden "Eimer-Experimente" prinzipiell unterscheiden sollten. :confused:
Was hat das alles mit "Positivismus" zu tun?


dass eine Impulsänderung eine Ortsveränderung bewirkt und umgekehrt.

Dem kann ich irgendwie gar nicht folgen. Das ist ja so, als würde man sagen - Beschleunigung bewirkt Ortsänderung und umgekehrt. :confused:
=>? Keine Beschleunigung -> keine Ortsänderung.


1963 zeigte John Bell theoretisch, 1980 Allan Aspect auch empirisch, dass die Quantenmechanik nichtlokal ist.


Hmmmm....
Ich würde das Thema "Bell" doch gerne genauer/gesondert besprechen. Hab' aber im Moment nicht viel Zeit. :( (Zumindest, um damit selbst anzufangen.)


A) Quantenmechanische Objekte sind ausgedehnte Objekte und können verschiedene Formen haben. Man betrachte beispielsweise die verschiedenen Formen von Elektronen im Atom bei unterschiedlichen Energieniveaus.


Man könnte also sagen, dass das "Spielchen" mit dem Welle- (= ausgedehnt) Teilchen- (= punkt-lokalisiert) Dualismus in beide Richtungen funktioniert.
EM-Welle = Kontinuum - kann/muss in bestimmten Fällen als ein Strom diskreter Objekte betrachtet werden.
Elektron (stellvertretend) = Teilchen - kann/muss in bestimmten Fällen als ein ausgedehntes Kontinuum betrachtet werden.


Alles weitere Geschehen ergibt sich dann aus der bedingten Wellenfunktion; d.h. dem quantenmechanischen Zustand an diesem Raumpunkt.


Da verstehe ich nicht, was du meinst.


D) Die Schwierigkeit, Quantenmechanik zu verstehen, ist im Grunde eine historische. Weil - siehe Gedanke 5 - ürsprünglich das Prinzip der Lokalität als sakrosant galt, war für die Gründerväter vieles unverständlich. Und auch heute noch ist das reale Geschehen deshalb strittig, weil die Theorie (= Schrödingergleichung) das Geschehen bei einer lokalen Wechselwirkung nicht beschreibt.

Zunächst - die Schrödinger-Gleichung ist noch nichtrelativistisch ("Newtonisch"). Daher kann diese der "Lokalität", die sich mit der Relativitätstheorie ergibt, prinzipiell gar nicht gerecht werden. Mit dem "realen Geschehen" hat das imho nichts zu tun. Zu dem "realen Geschehen" an sich hätte ich eine Frage - wie soll die Antwort aussehen, die dich befriedigen würde?


Gruß, Johann

Hawkwind
11.04.12, 12:53
Zunächst - die Schrödinger-Gleichung ist noch nichtrelativistisch ("Newtonisch"). Daher kann diese der "Lokalität", die sich mit der Relativitätstheorie ergibt, prinzipiell gar nicht gerecht werden.

"Lokalität" bedeutet einfach, dass sich Wirkungen von einem Punkt aus erst in dessen Umgebung und nach und nach in immer entferntere Regionen ausbreiten. Das kann man auch ohne Relativität diskutieren.

Wenn man unter "Wirkung" tatsächlich die Wirkung versteht, wie die Physik sie definiert, dann ist auch die Quantenmechanik lokal, denn Wirkungen (Energie, Signale) breiten sich auch in der Quantentheorie mit endlicher Geschwindigkeit aus. Ansonsten hätte man auch einen Widerspruch zur SRT.

Wenn man diese Definition aber auf die quantenmechanische Zustandsfunktion erweitert, dann erhält man ein nichtlokales Feature ... zumindest in der Kopenhagener Deutung - der berüchtigte Kollaps der Wellenfunktion findet überall zugleich statt.

EMI
11.04.12, 14:18
die Ereignisse, zwischen denen der quantenmechanische Zufall entscheidet, geschehen objektiv zufällig. Es sind die Anfangsbedingungen amc.


Aber nicht alle Möglichkeiten sind auch gleich wahrscheinlich. Z.B. wird man ein Elektron im Normalfall so gut wie niemals im Atomkern antreffen, weil hierfür die Wahrscheinlichkeit sehr nahe bei Null liegt.Zur Erklärung dafür, warum Elektronen nicht auf den Kern fallen, war es ganz natürlich anzunehmen, dass sie sich auf Bahnen bewegen, ähnlich wie die Planeten um die Sonne.
Denn die Gesetzte (Newton und Coulomb) haben eine ähnliche Form, die Kraft ist umgekehrt proportional dem Quadrat der Entfernung.

Wir wissen natürlich, dass ein bewegtes Elektron auf einer Bahn um den Kern einer Beschleunigung unterliegt und jede beschleunigte Ladung el.mag.Wellen aussendet, die Energie wegführt und infolge dessen das Elektron auf den Kern fallen muss. Dieser Schluss steht jedoch im krassen Widerspruch zur Erfahrung.

Bohr fand aber einen erfolgreichen Ausweg aus dieser Lage.
Er nahm an, dass es Bahnen gibt, auf denen Elektronen keine el.mag.Strahlung aussenden. Er gab auch eine Regel an, wie man diese Bahnen findet.
Die Ursache dieser Regel konnte Bohr natürlich nicht angeben, aber er ahnte vorraus, dass diese diskreten Bahnen irgendwie mit den damals noch unbekannten Quantengesetzten der Bewegung zusammenhängen müssten.

In der QM geht die Bewegung nicht auf Bahnen vor sich, selbst der Begriff ist hier ohne Sinn. Das wird dann wieder mit der Unbestimmtheitsrelation deutlich.
Die Unbestimmtheit der kin.Energie beträgt im Wasserstoffatom rund 140 eV und die Bindungsenergie des Elektrons rund 15 eV, woran man klar erkennt, das es nicht möglich ist die Bewegungsenergie des Elektrons im Atom in kin. und pot.Energie aufzuteilen.

Die Annahme, dass es im Atom eine Elektronenbahn gibt führt zu einer Unbestimmtheit der Größen, die deren "Bindungswert" selbst um ein mehrfaches überschreitet.
Ergo kann es keine Bahnen geben.

Wenn man sich schon das Atom vorstellen will, so muss man sich den Kern mit einer verschmierten Wolke umgeben vorstellen.
Die Dichte dieser Wolke in einem gegebenen Punkt muss der Aufenthaltswahrscheinlichkeit des Elektrons entsprechen, sprich der Amplitute der Wellenfunktion.
Aber damit wird keineswegs das Elektron selbst, sondern nur seine Wellenfunktion dargestellt.

Es ist aber nicht so, dass in der QM die Wellenfunktion irgendwie die Bahn der KM ersetzt. Woher wird denn die Wellenfunktion bekannt?

Bei einer Messung wird der Zustand eines Quantensystems wesentlich geändert und ist nach der Messung nicht mehr das gleiche wie vorher.
Damit befinden sich Systeme, deren Zustand bestimmt wurde offenkundig nicht mehr in diesem Zustand.

Grobes Anschauungsbeispiel:
Ein Streichholzhersteller will die Zündfähigkeit seiner Produkte prüfen.
Wenn er dazu alle Streichhölzer abbrennt macht das wohl keinen Sinn.
Er wird also nur einige auswählen und mit dieser Probe auf die wahrscheinliche Zündfähigkeit aller schließen.
War seine Probe groß genug, erhält er eine Ausage über die Qualität seiner Streichhölzer vor dem Abbrennen!

Wenn wir den Beugungsversuch von Elektronen durchführen, ist wohl klar, dass sich ein Elektron, das schon auf den Bildschirm gefallen ist, in einem anderen Zustand befindet als vor dem Durchgang durch ein Kristall.

Mit den objektiven physikalischen Eigenschaften einer Messung in der QM verhält es sich so, dass man als Ergebnis der Messung die Wahrscheinlichkeitsamplitute für Systeme bestimmt, an denen noch keine Messung durchgeführt wurde.
Dafür ist es halt notwendig, eine bestimmte Anzahl ähnlicher Systeme durch Messung "unbrauchbar" zu machen.
Man kann aber prinzipell davon ausgehen, dass die durch noch keine Messung verfälschten Systeme weitaus zahlreicher sind.
Deshalb wird in der Gesamtheit der Systeme, die der Messung nur zum Teil unterworfen wurden, durch die Messung gerade der Zustand bestimmt, der vor ihr bestand.

Die Wellenfunktion eines Systemes charakterisiert diejenige Wahrscheinlichkeit eines bestimmten Ergebnisses, die vor Ausführung der Messung vorlag.

Die klassische Physik kennt zwei Arten der Bewegung. Die Lageveränderung von Körpern auf bestimmten Bahnen und die Ausbreitung von Wellen.
Ungeachtet des verschiedenen Wesens dieser Bewegungen stimmen die Gesetze, denen beide gehorchen, manchmal völlig überein.
Das gilt für die Fälle, in denen die Wellenlänge klein genug ist im Vergleich zu den Abmessungen des Raumes, in dem sich der Wellenvorgang ausbreitet.
Bei den Elementarteilchen (z.B. Elektronen) wissen wir allerdings nicht im voraus welche Maße als klein anzusehen sind.
Es hat sich gezeigt, dass, wenn ein Elektronenstrahl durch ein Beugungsgitter hindurchgeht, die gleichen Beugungseffekte entstehen wie bei hochfrequenten el.mag. Wellen.
Es gibt also eine Elektronenbeugung! Nun sind aber die Elektronen keine Wellen sondern Teilchen.

Der Beugungsversuch zeigt, dass jedes Elektron wie eine Welle das Gitter durchläuft, ohne dabei aufzuhören ein unteilbares Teilchen zu sein.
Wir wissen aber auch, dass sich das Elektron in manch anderen Fällen ganz wie ein Teilchen bewegt, was keinerlei Welleneigenschaften aufweist.
So fliegen die Elektronen z.B. in einer Bildröhre auf festen Bahnen, die man genau so exakt vorausberechnen kann wie die Bahnen der Planeten.

Warum verhält sich ein Elektron mal wie eine Welle, mal wie ein Teilchen?
Wir erinnern uns, dass auch Licht das gleiche zweiseitige Verhalten zeigt.
Alles hängt vom Verhältnis zwischen der Wellenlänge und den Ausmaßen des Raumes ab, in dem die Bewegung vor sich geht.

Welche Wellenlänge entspricht nun aber der Bewegung eines Elektrons?
Man kann die Wellenlänge anhand des Beugungsbildes mit der gleichen Formel ermitteln, die zur Bestimmung der Wellenlänge von Röntgenstrahlen dient.
Dabei ergibt sich, die Wellenlänge ist umgekehrt proportional dem Impuls eines Teilchens. λ = h/p
Der Proportionalitätsfaktor zwischen ihnen ist eine universelle Konstante, die Planckkonstante h!

Mit h können wir nun die Frage beantworten weshalb sich in einer Bildröhre die Welleneigenschaften des Elektrons nicht äußern, während sie es im Kristall tun.
Die Wellenlänge des Elektrons in einer Bildröhre berechnet sich zu λ≈10^-11 m, der Durchmesser des Elektronenstrahls ist ungefähr 10^-4 m.
Der Durchmesser ist 10 Millionen mal größer als die entsprechende Wellenlänge!
Hier wird deutlich, dass sich in einer Bildröhre keinerlei Welleneigenschaften bei der Bewegung von Elektronen auf Bahnen zeigen können, dass es aber unbedingt zu Beugungserscheinungen kommen muss, wenn der gleiche Elektronenstrahl durch ein Kristall geht.

In welchen Grenzen hat der Begriff der Bahn eines Strahls einen Sinn?
Der Begriff der Teilchenbahn hat dann einen vernüftigen Sinn, wenn die Amplitute der Welle, die mit der Bewegung verknüpft ist, nach beiden Seiten der Bahn schnell zu Null wird.

Wie wirkt sich nun eine seitliche Begrenzung(Spalt) der Bahn aus?
Der Strahl hat hinter dem Spalt einen bestimmten(von der Wellenlänge abhängigen) Öffnungswinkel.

Wohin ist nun die Geschwindigkeit eines den Spalt durchlaufenden Teilchen gerichtet?
Ein Teilchen weist nur dann eine genau bestimmte Geschwindigkeitsrichtung auf wenn dessen Bewegung seitlich durch nichts begrenzt ist.
Wenn nun die den Spalt durchlaufenden Teilchen nicht genau parallel aus dem Spalt heraustreten, sondern in einem bestimmten Öffnungswinkel so liegt eben innerhalb dieses Winkels auch die Richtung der Geschwindigkeit des Teilchens.
Die Geschwindigkeit ist eine vektorielle Größe und wenn sie um einen bestimmten Winkel abweicht so bedeutet das, dass sie eine senkrechte Komponente erhalten hat die gleich dem Produkt der Geschwindigkeit und diesem Winkel ist.
Folglich zeigt die Geschwindigkeit des Teilchens nach dem Spaltdurchgang eine gewisse Streuung in der Fläche des Spaltes, denn wir wissen ja nicht, um welchen Winkel das Teilchen gerade abweicht.
Die Geschwindigkeit unterliegt einer Unbestimmtheit. Auch die Koordinate x zeigt eine Unbestimmtheit Δx.
Mit der Unbestimmtheit der Geschwindigkeit hat auch der Impuls p eine Unbestimmtheit. Δp = m Δv
Nach weiteren Rechnungen (http://www.quanten.de/forum/showpost.php5?p=52192&postcount=28) kommt man zu der für die Quantenmechanik fundamentalen Beziehung:
h ≤ Δp Δx
Je genauer die Koordinate gegeben ist um so weniger genau ist der Impuls gegeben, weil Δp umgekehrt proportional zu Δx ist.

Koordinate und Impuls eines Teilchens existieren als genaue physikalische Größe nicht gemeinsam!
Es ist prinzipell unmöglich ein Verfahren anzugeben was zu ihrer genauen Bestimmung führen würde.
Das liegt nicht an einer subjektiven Unvollkommenheit sondern das ist ein objektives Naturgesetz.
Diejenigen die das Unbestimmtheitsprinzip wiederlegen möchten, erwartet das traurige Schicksal der Erfinder der Perpetuum mobile!

Die Koordinate und der Impuls eines Teilchens, als genaue physikalische Größe, existieren nicht gemeinsam.
Auch existieren der Winkel(Azimut) und das Moment eines Teilchens, als genaue physikalische Größe, nicht gemeinsam.
Das ist ein objektives Gesetz.

Gruß EMI

amc
12.04.12, 13:53
Vielen Dank für die Ausführungen EMI. Habe mir alles durchgelesen und werde es sicherlich auch noch mehrmals lesen.

Es sind die Anfangsbedingungen amc.

Was ist gemeint? Sind es die Anfangsbedingungen, welche über die Ereignisse/Messresultate entscheiden? Oder sind es die Anfangsbedingungen, welche durch den objektiven Zufall bestimmt werden? Oder beides? Oder meinst du etwas anderes?

Klingt für mich nach De Broglie - Bohm. Nach meiner bescheidenen Kenntnis ist die übliche Sicht, dass in der QM eine immer gleiche Anfangsbedingung (Präparation des Experiments) zu immer verschiedenen Messresultaten führen kann, über die der objektive Zufall entscheidet. Und nach DeBroglie-Bohm ist es so, dass unterschiedliche Messresultate auch durch unterschiedliche Anfangsbedingungen determiniert sind. Und die Anfangsbedingungen unterliegen dabei keinem objektiven Zufall, sondern sie entziehen sich lediglich unserer Kenntnis.

Der Beugungsversuch zeigt, dass jedes Elektron wie eine Welle das Gitter durchläuft, ohne dabei aufzuhören ein unteilbares Teilchen zu sein.

Reine Spekulation, oder nicht? ;)

Ein Teilchen weist nur dann eine genau bestimmte Geschwindigkeitsrichtung auf wenn dessen Bewegung seitlich durch nichts begrenzt ist.

Ja, ist schon irgendwie verrückt, dass ein Teilchen beim Durchqueren eines Spaltes abgelenkt werden kann, ohne den Spalt dabei überhaupt zu berühren. :)

Grüße, AMC

RoKo
12.04.12, 16:25
Hallo JoAx,
Gegenanmerkung:
Du bringst ein Beispiel, bei dem klassische Physik (= gewohnter Anschauungsrahmen) zuständig ist, und dann gehst du ganz ungezwungen zur QM? :eek:
Beschreibe doch zunächst genau so ungezwungen den Eimer mit Wasser darin - quantenmechanisch. Bitte!
Wo ist da das Problem? Es ist nicht nur sinnlos, sondern auch falsch, den Eimer Wasser quantenmechanisch zu beschreiben. Er verhält sich nicht wie ein quantenmechanisches System.
Ansonsten sehe ich auch nicht, warum sich die beiden "Eimer-Experimente" prinzipiell unterscheiden sollten. :confused:Der erste Eimer wird sich annähernd wie ein Festkörper verhalten. Der zweite wird auf Grund unterschiedlicher cw-Werte sich in Eimer und Wasser trennen. Das Wasser muß dann bezüglich seiner Geschwindigkeit als Vektorfeld betrachtet werden.
Was hat das alles mit "Positivismus" zu tun?Ein solches Geschwindigkeitsverktorfeld kann man nicht messen - auch im Prinzip nicht. Den hier immer von einigen mit Vehemenz vertretenen philosophischen Auffassungen zu Folge kann man dann dem herabfallenden Wasser keine Geschwindigkeit zusprechen.Dem kann ich irgendwie gar nicht folgen. Das ist ja so, als würde man sagen - Beschleunigung bewirkt Ortsänderung und umgekehrt. :confused:Ja - aber dabei die Lenzesche Regel beachten. Nur dann kommt man zur sich ausbreitenden (EM)-Welle.=>? Keine Beschleunigung -> keine Ortsänderung.Quantenmechanik >< klassische Mechanik. Beachte den Unterschied zwischen Differentialgleichungen erster und zweiter Ordnung.Hmmmm....
Ich würde das Thema "Bell" doch gerne genauer/gesondert besprechen. Hab' aber im Moment nicht viel Zeit. :( (Zumindest, um damit selbst anzufangen.)Ok. Dann später mal. Ich bin ohnehin 10.000km von meinem Buch von ihm entfernt.Man könnte also sagen, dass das "Spielchen" mit dem Welle- (= ausgedehnt) Teilchen- (= punkt-lokalisiert) Dualismus in beide Richtungen funktioniert.
EM-Welle = Kontinuum - kann/muss in bestimmten Fällen als ein Strom diskreter Objekte betrachtet werden.
Elektron (stellvertretend) = Teilchen - kann/muss in bestimmten Fällen als ein ausgedehntes Kontinuum betrachtet werden.Man kann sich mit diesen Bildern behelfen.

ich schrieb:.. Alles weitere Geschehen ergibt sich dann aus der bedingten Wellenfunktion; d.h. dem quantenmechanischen Zustand an diesem Raumpunkt.Da verstehe ich nicht, was du meinst.
Am Raumpunkt der Wechselwirkung liegt ja ein bestimmter quantenmechanischer Zustand vor. Nehmen wir mal an, die Wechselwirkung erfolgt mit einer Messsonde. Dann tritt der quantenmechanischen Zustand am Ort dieser Messonde mit der Messdynamik in Wechselwirkung und das Resultat ist ein Eigenwert des zugehörigen Messoperators. (Schrödinger [1954, "Are there Quantum Jumps?"] spricht von einem Resonanzeffekt)

Aus Sicht der BM/dBB-Theorie bedarf es keinerlei weiterer Erläuterung. Aus Sicht der KD ist es aber auch nicht anders, weil die Wellenfunktion im Falle der Wechselwirkung kollabiert und lokalisiert wird.

Dieser Kollaps der Wellenfunktion lässt sich (entgegen von mir früher vertretenen Auffassungen) doch über das Prinzip Verschränkung - Dekohärenz erklären.
Zunächst - die Schrödinger-Gleichung ist noch nichtrelativistisch ("Newtonisch"). Daher kann diese der "Lokalität", die sich mit der Relativitätstheorie ergibt, prinzipiell gar nicht gerecht werden. Mit dem "realen Geschehen" hat das imho nichts zu tun. Zu dem "realen Geschehen" an sich hätte ich eine Frage - wie soll die Antwort aussehen, die dich befriedigen würde?Darauf hat Hawkwind schon ausführlich geantwortet.

RoKo
12.04.12, 16:35
Hallo amc,

.. Was ist gemeint? Sind es die Anfangsbedingungen, welche über die Ereignisse/Messresultate entscheiden? Oder sind es die Anfangsbedingungen, welche durch den objektiven Zufall bestimmt werden? Oder beides? Oder meinst du etwas anderes?

Klingt für mich nach De Broglie - Bohm. Nach meiner bescheidenen Kenntnis ist die übliche Sicht, dass in der QM eine immer gleiche Anfangsbedingung (Präparation des Experiments) zu immer verschiedenen Messresultaten führen kann, über die der objektive Zufall entscheidet. Und nach DeBroglie-Bohm ist es so, dass unterschiedliche Messresultate auch durch unterschiedliche Anfangsbedingungen determiniert sind. Und die Anfangsbedingungen unterliegen dabei keinem objektiven Zufall, sondern sie entziehen sich lediglich unserer Kenntnis. Immer exakt gleiche Anfangsbedingungen sind unmöglich zu präparieren.Zitat von EMI
Der Beugungsversuch zeigt, dass jedes Elektron wie eine Welle das Gitter durchläuft, ohne dabei aufzuhören ein unteilbares Teilchen zu sein.
Reine Spekulation, oder nicht? ;) Wo sollte da die Spekulation sein? Nach allen bisherigen Erfahrungen ist die Elementarladung unteilbar, weshalb sie auch so heisst.Ja, ist schon irgendwie verrückt, dass ein Teilchen beim Durchqueren eines Spaltes abgelenkt werden kann, ohne den Spalt dabei überhaupt zu berühren. :)Huygensches Prinzip. Wenn man das verstanden hat, ist es auch nicht verrückt sondern logisch

fossilium
13.04.12, 00:36
Hi Emi,

Mit den objektiven physikalischen Eigenschaften einer Messung in der QM verhält es sich so, dass man als Ergebnis der Messung die Wahrscheinlichkeitsamplitute für Systeme bestimmt, an denen noch keine Messung durchgeführt wurde.

Bitte sage mir:

Ist das Deine persönliche Sicht der Dinge ?

Oder ist das die aktuell gängige Vorstellung in der etablierten Physik ?

Das Messung immer einhergeht mit einer Vorstellung von Etwas, ist mir schon klar, aber dass man etwas misst - um sich dann das Ungemessene vorstellen zu können - dieser Gedanke erscheint etwas bizarr, aber dennoch ...

Ich hab diese Interpretation jedenfalls bisher nicht gehört.

Was messe ich denn dann eigentlich, das Gemessene oder Ungemessene, etas Konkretes oder Fiktives ? Ist der Schluss vom Gemessenen auf das Ungemesse erlaubt ? Ich glaube, das ist die entscheidende Frage.


Grüsse Fossilium

EMI
13.04.12, 01:04
Ist das Deine persönliche Sicht der Dinge ? Oder ist das die aktuell gängige Vorstellung in der etablierten Physik ?Keine Ahnung fossi, das vermischt sich bei mir immer.



Das Messung immer einhergeht mit einer Vorstellung von Etwas, ist mir schon klar, aber dass man etwas misst - um sich dann das Ungemessene vorstellen zu können - dieser Gedanke erscheint etwas bizarr, aber dennoch ...
Ich hab diese Interpretation jedenfalls bisher nicht gehört.Das ist keine Interpretation. Ich schrieb von Wahrscheinlichkeit.
Sicherlich hast Du schon von Umfragen im Vorfeld von Wahlen gehört.
Das hat zwar nix mit Physik zu tun, aber da misst man etwas um auf das Ungemessene schließen zu können. Die nennen das dann Hochrechnung.



Ist der Schluss vom Gemessenen auf das Ungemesse erlaubt ? Ich glaube, das ist die entscheidende Frage.Ich denke schon.
Wenn man z.B. immer wieder die el.Elementarladung von Elektronen misst und stets das gleiche Ergebnis erhält ist der Schluss, dass alle ungemessenen Elektronen im Universum auch diese el.Ladung tragen legitim.

Gruß EMI

fossilium
13.04.12, 08:24
Hi Emi,
Wenn man z.B. immer wieder die el.Elementarladung von Elektronen misst und stets das gleiche Ergebnis erhält ist der Schluss, dass alle ungemessenen Elektronen im Universum auch diese el.Ladung tragen legitim.
Nein Emi, das ist nicht das Gleiche wie bei der Dir beschriebenen quantenmechanischen Messung.

In Deiner Beschreibung steckt ein Wurm - da bin ich sicher.

Grüsse Fossilium

JoAx
13.04.12, 12:53
Hi Uli!

"Lokalität" bedeutet einfach, dass sich Wirkungen von einem Punkt aus erst in dessen Umgebung und nach und nach in immer entferntere Regionen ausbreiten. Das kann man auch ohne Relativität diskutieren.


Diskutieren kann man das ja schon, aber ohne Relativität gibt es ja keine Einschränkung dafür. Wenn man jetzt die Wellenfunktion als ein "zusammenhängendes Objekt" betrachtet, in etwa, wie ein elastisches Band, so versteh ich Rolf's Ausführungen, dann zieht sich dieses ja auch überall zugleich zusammen, wenn man die auseinanderziehende Kraft verringert. Zumindest in der gewohnten Newtonschen Vorstellung. Die Wirkungen gebrauchen da keine Zeit zum Ausbreiten.


Wenn man unter "Wirkung" tatsächlich die Wirkung versteht, wie die Physik sie definiert, dann ist auch die Quantenmechanik lokal, denn Wirkungen (Energie, Signale) breiten sich auch in der Quantentheorie mit endlicher Geschwindigkeit aus.


Was ist Energie/Signal/... ?
Wenn wir Photon A messen, findet eine Wechslwirkung statt, und dadurch ist/wird sein Zustand determiniert. Richtig?
Ist es keine (Wechsel-) Wirkung, wenn der Zustand von Photon B durch die Messung an Photon A determiniert wird?

Aus dem http://cdsweb.cern.ch/record/142461/files/198009299.pdf
Seite 8.

Could we not be a little more clever, and devise a model which reproduces the quantum formulae completely? No. It cannot be done, so long as action at a distance is excluded.


Ansonsten hätte man auch einen Widerspruch zur SRT.


Warum gibt es dennoch keinen Widerspruch zu SRT?
Was für action geht hier ab?
Wie wird der Widerspruch zu SRT u.U. - hmmm... - umgangen?


Gruß, Johann

Hawkwind
13.04.12, 13:29
Hi Uli!



Diskutieren kann man das ja schon, aber ohne Relativität gibt es ja keine Einschränkung dafür. Wenn man jetzt die Wellenfunktion als ein "zusammenhängendes Objekt" betrachtet, in etwa, wie ein elastisches Band, so versteh ich Rolf's Ausführungen, dann zieht sich dieses ja auch überall zugleich zusammen, wenn man die auseinanderziehende Kraft verringert. Zumindest in der gewohnten Newtonschen Vorstellung. Die Wirkungen gebrauchen da keine Zeit zum Ausbreiten.



Ja, streng genommen ist die Newtonsche Gravitation nicht-lokal; Newton war ja von instantaner Ausbreitung von Feldänderungen ausgegangen.

Allerdings lassen sich durchaus Felder mit endlicher Ausbreitungsgeschwindigkeit auch nichtrelativistisch diskutieren.




Was ist Energie/Signal/... ?
Wenn wir Photon A messen, findet eine Wechslwirkung statt, und dadurch ist/wird sein Zustand determiniert. Richtig?
Ist es keine (Wechsel-) Wirkung, wenn der Zustand von Photon B durch die Messung an Photon A determiniert wird?


Nein, denn diese Reduktion der "Zustandsfunktion" bedarf keines Transportes von Informationen oder Energie von A nach B. Energie fließt nur in Richtung der Bewegung beider Teilchen, und das mit deren Geschwindigkeit.
Der Kollaps ist nur Interpretation; er findet in den Gehirnen der Menschen statt, nicht in der Natur.

RoKo
13.04.12, 13:47
Hallo JoAx,

Ist es keine (Wechsel-) Wirkung, wenn der Zustand von Photon B durch die Messung an Photon A determiniert wird?
Verschränkte Photonen sind als Doppelphoton zu betrachten, ebenso z.b. die beiden Elektronen des Heliums. Es findet daher stets eine lokale Wechselwirkung an einem System statt, die auf die verschränkten Teilsysteme gleichermaßen und gleichzeitig wirkt - unabhängig davon, wie weit die Teilsysteme voneinander räumlich entfernt sind.

JoAx
13.04.12, 15:34
Hi Rolf!


Wo ist da das Problem? Es ist nicht nur sinnlos, sondern auch falsch, den Eimer Wasser quantenmechanisch zu beschreiben.


Da bin ich anderer Meinung, wie du sicher vermutest. Auch einem Eimer mit Wasser kann man, entsprechend seiner Gesamtenergie = ERuhe + EKin eine Wellenlänge zusprechen, und somit auch den qm. Formalismus anwenden. Da gibt es imho keine Einschränkungen von der mathematischen Seite. Höchstens den guten alten "gesunden Menschenverstand". Ich schätze, dass man keine experimentellen Unterschiede feststellen wird.


Er verhält sich nicht wie ein quantenmechanisches System.


Oder - bei einem realen Experiment würden sich keine beobachtbaren Effekte ergeben, die man "quantenmechanisch" nennen könnte.


Der zweite wird auf Grund unterschiedlicher cw-Werte sich in Eimer und Wasser trennen.


Schön. Wenn wir schon einen makroskopischen Eimer mit Wasser, mit einem bsw. Elektron auf ihre qm-sche Eigenschaften vergleichen wollen, dann sollte man doch bitte möglichst gleiche Bedingungen schaffen. Mit Luft gefüllter Raum gehört sicher nicht dazu.

Mein "Problem" ist, dass dein Beispiel (in dieser Form) schlicht keinen Vergleich zwischen klassischen und quantenmechanischen Verhalten erlaubt. Und deine Feststellung:

Gemäß der positivistischen Anschauung, die hier manchmal vertreten wird, dürfen wir dem einzelnen Wassermolekül keinen Zustand zusprechen, solange wir ihn nicht gemessen haben.

geht deswegen ins Leere, weil die Messung nichts ist, was nur der Mensch "tun" kann/muss. Da sorgen andere Wassermoleküle für eine Messung. Der "Zustand" der Wassermoleküle ändert sich halt ständig. So würde ich das dar legen.


Den hier immer von einigen mit Vehemenz vertretenen philosophischen Auffassungen zu Folge kann man dann dem herabfallenden Wasser keine Geschwindigkeit zusprechen.


Keine im klassischen Sinne, wenn der Vergleich überhaupt zulässig wäre. Das ist ja der Punkt.


Ja - aber dabei die Lenzesche Regel beachten. Nur dann kommt man zur sich ausbreitenden (EM)-Welle.


Wird nicht klarer für mich.


Quantenmechanik >< klassische Mechanik. Beachte den Unterschied zwischen Differentialgleichungen erster und zweiter Ordnung.


Bringt mich jetzt auch nicht weiter. Was genau willst du sagen?
Was ist der Unterschied?


Am Raumpunkt der Wechselwirkung liegt ja ein bestimmter quantenmechanischer Zustand vor.

Welches, und von was?


Dann tritt der quantenmechanischen Zustand am Ort dieser Messonde mit der Messdynamik in Wechselwirkung


Rolf! Ich habe ganz ehrlich grösste Schwierigkeiten mit solchen Formulierungen. Was soll denn bsw. die Messdynamik sein? Und wie soll diese mit dem "quantenmechanischen Zustand" in "Wechselwirkung" treten? Das hört sich, für mich zumindest, sonderbar an.


und das Resultat ist ein Eigenwert des zugehörigen Messoperators. (Schrödinger [1954, "Are there Quantum Jumps?"] spricht von einem Resonanzeffekt)


Mit anderen Worten -> Eigenwert ist nichts, was als dem qm-schen Objekt eigen zu betrachten ist? Es ist im Grunde gar keine Eigenschaft?


Aus Sicht der BM/dBB-Theorie bedarf es keinerlei weiterer Erläuterung.


:eek::D


Gruß

EMI
13.04.12, 16:15
In Deiner Beschreibung steckt ein Wurm - da bin ich sicher.Ne, da steckt kein Wurm drin fossi,

darauf verwette ich einen halben Kasten Bier.

Gruß EMI

RoKo
13.04.12, 17:52
Hallo JoAx,

Da bin ich anderer Meinung, wie du sicher vermutest. Auch einem Eimer mit Wasser kann man, entsprechend seiner Gesamtenergie = ERuhe + EKin eine Wellenlänge zusprechen, und somit auch den qm. Formalismus anwenden. Da gibt es imho keine Einschränkungen von der mathematischen Seite. Höchstens den guten alten "gesunden Menschenverstand". Ich schätze, dass man keine experimentellen Unterschiede feststellen wird.Über diesen Punkt hatten wir hier schon einmal ein Streitgespräch. Und ich habe vor rund 40 Jahren schon mit meinem Physiklehrer darüber gestritten.

Die de Broglie-Beziehung bezieht sich auf Elektronen, Protonen und Neutronen. Dort macht sie auch Sinn, weil es sich um Objekte handelt, die durch eine Wellenfunktion beschrieben werden. Dies führt zum Atommodell stehender Wellen. Daraus ergibt sich wiederum z.B. das Kristallgittermodell für Festkörper. Da schwingt dann nichts mehr. Also ist der Begriff Wellenlänge hier physikalisch sinnlos.

Bedenke auch, dass mit zunehmender Ruhemasse die Schwingung hochfrequenter würde. Welcher physikalische Sinn sollte da bestehen?
Oder - bei einem realen Experiment würden sich keine beobachtbaren Effekte ergeben, die man "quantenmechanisch" nennen könnte.Leider lässt sich dieser Streitpunkt nicht per Experiment entscheiden. Da hilft nur logische Konsistenz des naturwissenschaftlichen Gesamtbildes.Schön. Wenn wir schon einen makroskopischen Eimer mit Wasser, mit einem bsw. Elektron auf ihre qm-sche Eigenschaften vergleichen wollen, dann sollte man doch bitte möglichst gleiche Bedingungen schaffen. Mit Luft gefüllter Raum gehört sicher nicht dazu. Ohne Tafel kann ich das leider nicht näher erläutern. Und deine Feststellung:Zitat von RoKo
Gemäß der positivistischen Anschauung, die hier manchmal vertreten wird, dürfen wir dem einzelnen Wassermolekül keinen Zustand zusprechen, solange wir ihn nicht gemessen haben. geht deswegen ins Leere, weil die Messung nichts ist, was nur der Mensch "tun" kann/muss. Womit du philosophisch keinen positivistischen sondern einen realistischen / materialistischen Standpunkt vertrittst.

Zitat von RoKo
Quantenmechanik >< klassische Mechanik. Beachte den Unterschied zwischen Differentialgleichungen erster und zweiter Ordnung.
Bringt mich jetzt auch nicht weiter. Was genau willst du sagen?
Was ist der Unterschied?Ein KM-System ist durch Ort und Impuls bestimmt; eine QM-System durch Ort oder Impuls. (Siehe Schrödinger 1935)

Mit anderen Worten -> Eigenwert ist nichts, was als dem qm-schen Objekt eigen zu betrachten ist? Es ist im Grunde gar keine Eigenschaft? Genau. Ein QM-System hat die Eigenschaften Masse, Ladung, Spin-Zahl (Konstant) sowie Energie (variabel). Strittig, da experimentel nicht beweisbar, ist die Eigenschaft Ort (variabel). [Ist auch nicht wichtig, da wir ihn vor einer Messung ohnehin nicht kennen.] Mehr Eigenschaften kann man an einem QM-System nicht messen (im ursprünglichen Sinne des Wortes).

----gute Nacht.

fossilium
13.04.12, 22:44
Hi Emi,

Doch, es gibt einen Wurm.
Sogar mehrere.
Wart ab.

Gruß Fossilium

JoAx
14.04.12, 09:14
Die de Broglie-Beziehung bezieht sich auf Elektronen, Protonen und Neutronen.


Nein. Diese Einschränkung ist so völlig willkürlich. So geht das einfach nicht.


Dort macht sie auch Sinn, weil es sich um Objekte handelt, die durch eine Wellenfunktion beschrieben werden.


Dann beschreibe doch auch dein Eimer mit der Wellenfunktion.


Da schwingt dann nichts mehr. Also ist der Begriff Wellenlänge hier physikalisch sinnlos.


Die Wellenfunktion ist doch eine Wahrscheinlichkeits-"welle". Da braucht von sich aus nix zu schwingen. Und ausserdem schwingen die Atome im Kristallgitter sehr wohl.


Bedenke auch, dass mit zunehmender Ruhemasse die Schwingung hochfrequenter würde.


Da ist nicht die Frequenz entscheidend, die man auch zusprechen könnte, sondern die Wellenlänge. Kürzere Wellenlänge - Das Wellenpaket zerfliesst langsamer. (Oder irgendwann vlt. auch gar nicht mehr = "Das Ganze ist weniger als die Summe seiner Teile!" :D)


Welcher physikalische Sinn sollte da bestehen?


Den gleichen, wie bei jeder physikalischen Berechnung. -> Eine zutreffende Voraussage treffen zu können.

Wenn du sagen würdest - "Es ist nicht gerade zweckmässig, die QM auf ein gewöhnliches Eimer mit Wasser anzuwenden, nur um zu berechnen, wo es landet." - dann würde ich dir sofort zustimmen. Es ist genau so unzweckmässig, wie die ART auf gewöhnlichen senkrechten Wurf anzuwenden. Aber das ist dann auch alles.


Leider lässt sich dieser Streitpunkt nicht per Experiment entscheiden.


Doch. Natürlich. Dazu muss man lediglich die Voraussagen der KM und der QM vergleichen. Ich wette, dass die Unterschiede nicht messbar sein werden.


Da hilft nur logische Konsistenz des naturwissenschaftlichen Gesamtbildes.


Das klingt für mich zu sehr nach dem "gesunden Menschenverstand". Beeindruckt mich nicht wirklich.


Womit du philosophisch keinen positivistischen sondern einen realistischen / materialistischen Standpunkt vertrittst.


Keine Ahnung, welchen Standpunkt ich vertrete. Dieses Schubladendenken - positivistisch, realistisch, was weiss ich noch was - geht mir aber langsam schlicht auf den Keks. Das muss ich jetzt ein Mal raus lassen.


Ein KM-System ist durch Ort und Impuls bestimmt; eine QM-System durch Ort oder Impuls. (Siehe Schrödinger 1935)


Du sprichst von den Anfangsbedingungen. Und weiter? Es ist ja nicht alles vom QM-System bestimmt, sondern nur "Orts"-Zustand, oder nur "Impuls"-Zustand, oder ... . Was ja nach der Unbestimmtheitsrelation auch logisch ist.

Genau. ...


Das ist zumindest ein interessanter Gedanke! Die Messergebnisse auf ihre "Eigenschaftlichkeit" zu untersuchen.


Gruß, Johann

RoKo
14.04.12, 18:15
Hallo JoAx,

es gibt einige Punkte, da haben wir beide eine unterschiedliche Meinung, zwischen denen nicht so einfach entschieden werden kann. Mit unserer Debatte drehen wir uns im Kreis. Ich lasse das deshalb jetzt.

Zu zwei Punkten möchte ich jedoch die Debatte fortführen, weil sie etwas anderer Natur ist.

.. Du sprichst von den Anfangsbedingungen. Und weiter? Es ist ja nicht alles vom QM-System bestimmt, sondern nur "Orts"-Zustand, oder nur "Impuls"-Zustand, oder ... . Was ja nach der Unbestimmtheitsrelation auch logisch ist.Nein. Ich meine nicht Anfangsbedigungen. Eine Differentialgleichung zweiter Ordnung führt zu einer Funktion mit zwei unabhängigen Variablen. F=m*a; a= 2.Ableitung des Weges nach der Zeit ist eine solche DGL. Ort und Impuls (oder zwei andere variable, aus denen sich beides ergibt) sind unabhängig voneinander. Du must beide kennen, um das weitere Verhalten des Systems berechnen zu können.

Die Schrödinger-Gleichung hingegen ist eine partielle Differentialgleichung und führt zu einer Funktion mit nur einer unabhängigen Variablen. Schrödinger diskutiert diesen Unterschied ausführlich in seinem Aufsatz von 1935. (Vor zwei Wochen hatte Petruska auch einen Link dazu eingestellt.)

Das ist zumindest ein interessanter Gedanke! Die Messergebnisse auf ihre "Eigenschaftlichkeit" zu untersuchen.
Die Schrödinger-Gleichung ist eine Energie-Gleichung. Der quantenmechanische Zustand ist daher eín energetischer Zustand. Und das ist die einzige variable Eigenschaft eines QM-Systems.

Betrachtet man nun Energie als Wirkungspotenz, also als Möglichkeit, eine Wirkung zu erzielen, und berücksichtigt, das ein Quantum eben nur einmal eine Wirkung erzielen kann - und damit selbst verändert oder sogar absorbiert wird, dann wird klar, dass die Wahrscheinlichkeit, tatsächlich Wirkung zu erzielen, proportional zur Energiedichte ist.

Timm
15.04.12, 09:45
chi-fan-le-ma RoKo,

Betrachtet man nun Energie als Wirkungspotenz, also als Möglichkeit, eine Wirkung zu erzielen, und berücksichtigt, das ein Quantum eben nur einmal eine Wirkung erzielen kann - und damit selbst verändert oder sogar absorbiert wird, dann wird klar, dass die Wahrscheinlichkeit, tatsächlich Wirkung zu erzielen, proportional zur Energiedichte ist.
Betrachten wir ein Quantenobjekt, sei es ein Photon oder ein C-70 Fulleren, als eine sich ungestört ausbreitende Kugelwelle. Dann hätte demnach die Energiedichte zu einem bestimmten Zeitpunkt an einem beliebigen Ort auf der Kugelschale denselben Wert. Habe ich Dich soweit richtig verstanden?

Gruß, Timm

RoKo
15.04.12, 16:26
Hallo Timm,

.Betrachten wir ein Quantenobjekt, sei es ein Photon oder ein C-70 Fulleren, als eine sich ungestört ausbreitende Kugelwelle. Dann hätte demnach die Energiedichte zu einem bestimmten Zeitpunkt an einem beliebigen Ort auf der Kugelschale denselben Wert. Habe ich Dich soweit richtig verstanden? Nein und Ja. Die Energien dieser beiden unterschiedlichen Objekte sind natürlich unterschiedlich. Aber die jeweilige Energiedichte hat zu einem bestimmten Zeitpunkt an einem beliebigen Ort auf der Kugelschale denselben Wert.

Ob die Beschreibung als Kugelwelle richtig ist, ist bereits bei einem Photon fraglich. Das kann man in der Theorie leicht annehmen, setzt aber ein ziemlich einsames Atom irgendwo im Weltraum voraus. Ein C-70 Fulleren kann man da aber nicht einfach aus dem Hut zaubern. Mal davon abgesehen, dass man die chemischen Bindungskräfte berücksichtigen muß. Ein C-70 Fulleren kann sich daher nicht wie eine Kugelwelle ausbreiten.

Timm
15.04.12, 16:57
Hallo RoKo,

Nein und Ja. Die Energien dieser beiden unterschiedlichen Objekte sind natürlich unterschiedlich. Aber die jeweilige Energiedichte hat zu einem bestimmten Zeitpunkt an einem beliebigen Ort auf der Kugelschale denselben Wert.

Ok, zieht sich nun die auf beliebige Orte verteilte Energie des Objektes im Augenblick der Messung instantan auf einen Ort zusammen?

Ein C-70 Fulleren kann man da aber nicht einfach aus dem Hut zaubern. Mal davon abgesehen, dass man die chemischen Bindungskräfte berücksichtigen muß. Ein C-70 Fulleren kann sich daher nicht wie eine Kugelwelle ausbreiten.
Laß Dich von präperativen Herausforderungen nicht irritieren, darum geht es nicht. Nebenbei, die chemischen Bindungskräfte von C-70 Fulleren muß man nicht berücksichtigen, genausowenig wie die innere Struktur von Neutronen. Entscheidend ist die Präparation als Quantenobjekt. Zeilinger hat ja Fullerene in eine Superposition gebracht.

Gruß, Timm

RoKo
15.04.12, 18:52
Hallo Timm,

Ok, zieht sich nun die auf beliebige Orte verteilte Energie des Objektes im Augenblick der Messung instantan auf einen Ort zusammen?Der (Standard)Theorie nach ja. Bisher ist dieser "Kollaps" der Wellenfunktion leider nur als Postulat eingeführt.
.. Nebenbei, die chemischen Bindungskräfte von C-70 Fulleren muß man nicht berücksichtigen, genausowenig wie die innere Struktur von Neutronen. Entscheidend ist die Präparation als Quantenobjekt. Da hätte ich dann doch gerne die Wellenfunktion..Zeilinger hat ja Fullerene in eine Superposition gebracht.In eine Superposition von was?
Zeilinger hat Beugungserscheinungen nachgewiesen. Mehr nicht.

Ich
15.04.12, 21:12
Ok, zieht sich nun die auf beliebige Orte verteilte Energie des Objektes im Augenblick der Messung instantan auf einen Ort zusammen?
Der (Standard)Theorie nach ja.
Sehe ich nicht so, und fände ich auch nicht vertretbar.
Es ist unser Wissen, das im Augenblick der Messung neu definiert wird, nicht das Objekt. Deswegen kann da auch "instantan" was passieren, trotz SRT.
Ich will damit explizit nicht sagen, dass das Objekt immer einen wohldefinierten Zustand (Ort, Impuls) hätte, von dem wir erst im Augenblick der Messung erfahren. Sondern nur, dass der Zustand mit dem gemessenen Wert konsistent sein muss. Was dann natürlich die Projektionen in die Zukunft beeinflusst.

Ich will mich da gar nicht beliebig weit aus dem Fenster lehnen mit Interpretationen, aber sogar von Weizsäcker sagt in seinen philosophischen Exkursionen deutlich, dass man sich die Wellenfunktion keineswegs als "etwas Seiendes" vorstellen dürfe. Da hat er recht. Was da kollabiert, war nie, ist nicht, und wird nie sein. Es ist nur eine Beschreibung - wenn auch, und das macht es schwierig, eine absolut ausreichende, alles was wir kriegen können.

RoKo
16.04.12, 03:39
Hallo Du,
Sehe ich nicht so, und fände ich auch nicht vertretbar.
Es ist unser Wissen, das im Augenblick der Messung neu definiert wird, nicht das Objekt. Deswegen kann da auch "instantan" was passieren, trotz SRT.
..
Ich will mich da gar nicht beliebig weit aus dem Fenster lehnen mit Interpretationen, aber sogar von Weizsäcker sagt in seinen philosophischen Exkursionen deutlich, dass man sich die Wellenfunktion keineswegs als "etwas Seiendes" vorstellen dürfe. Da hat er recht. Was da kollabiert, war nie, ist nicht, und wird nie sein. Es ist nur eine Beschreibung - wenn auch, und das macht es schwierig, eine absolut ausreichende, alles was wir kriegen können.
eine philosophische Position legt auch Verpflichtungen auf. Einige Fragen must du dann schon beantworten können:
- Wie kommt es, dass das geistige Konstrukt Wellenfunktion materielle Realität verändern kann?
- Wie kommt die Energie von der präparierten Ausgangssituation zum Messgerät?
- Wie erkärtst du die verschiedenen chemischen Bindungskräfte von Atomen?
- Wie kommt es, dass angeregte Atome portionierte elektromagnetische Wellen abstrahlen?
- Was kann man mit einem Rastertunnelmikroskop sehen?
- Wie sind solche Fälle zu beurteilen, wo man die Wellenfunktion nicht kennt, sich die Natur aber so verhält als gäbe es eine?

Ich
16.04.12, 09:59
Hi,

- Wie kommt es, dass das geistige Konstrukt Wellenfunktion materielle Realität verändern kann?
Kann es nicht. Es beschreibt sie nur.

Ich hab nur was gegen die verteilte Energie, die sich "instantan" auf einen Ort zusammenzieht. Wenn sowas in irgendeiner Weise messbar (also "real") wäre, dann wäre die SRT widerlegt. Man sollte den Kollaps der Wellenfunktion nicht mit einer Veränderung der realen Welt gleichsetzen. Das ist m.E. nur eine Metapher für das Einarbeiten neuer Information in unsere Beschreibung.

Bauhof
16.04.12, 10:00
Ich will mich da gar nicht beliebig weit aus dem Fenster lehnen mit Interpretationen, aber sogar von Weizsäcker sagt in seinen philosophischen Exkursionen deutlich, dass man sich die Wellenfunktion keineswegs als "etwas Seiendes" vorstellen dürfe. Da hat er recht. Was da kollabiert, war nie, ist nicht, und wird nie sein. Es ist nur eine Beschreibung - wenn auch, und das macht es schwierig, eine absolut ausreichende, alles was wir kriegen können.
Hallo Ich,

so ist es.
Viel früher stellte dies bereits Max Born fest, und der war Physiker und kein Philosoph. Und viel später dann auch Anton Zeilinger:

Ich bin nicht ein Anhänger des Konstruktivismus, sondern ein Anhänger der Kopenhagener Interpretation. Danach ist der quantenmechanische Zustand die Information, die wir über die Welt haben. … Es stellt sich letztlich heraus, dass Information ein wesentlicher Grundbaustein der Welt ist. Wir müssen uns wohl von dem naiven Realismus, nach dem die Welt an sich existiert, ohne unser Zutun und unabhängig von unserer Beobachtung, irgendwann verabschieden. (http://de.wikipedia.org/wiki/Anton_Zeilinger)

M.f.G Eugen Bauhof

Timm
16.04.12, 10:38
Hallo RoKo,

Aussagen wie diese (http://books.google.de/books?id=1czk0wXi6AcC&pg=PA98&lpg=PA98&dq=kugelwelle+quantensystem&source=bl&ots=VqZl6ieyt4&sig=P_2DlyY4sBuc_p_4ZgrKe5f_uEs&hl=de&sa=X&ei=hoWKT-bOPMvLsgaGh4TdDQ&sqi=2&ved=0CCsQ6AEwAg#v=onepage&q=kugelwelle%20quantensystem&f=false)

Die quantenmechanische Wellenfunktion bezieht sich auf reine Möglichkeit:
Verstünde man sie als Beschreibung einer Wirklichkeit, also einer wirklichen Welle, hätte das absurde Konsequenzen. ... Diese rein potentielle Welle kollabiert, man könnte sagen: wird real in einem bestimmten Meßereignis.
sind typisch.
Offensichtlich hat sich dazu von Weizäcker nach 'Ich's Erläuterung
Zitat von 'Ich'
... aber sogar von Weizsäcker sagt in seinen philosophischen Exkursionen deutlich, dass man sich die Wellenfunktion keineswegs als "etwas Seiendes" vorstellen dürfe. Da hat er recht. Was da kollabiert, war nie, ist nicht, und wird nie sein.

ähnlich geäußert.

Es ist für mich völlig akzeptabel, daß Du es anders siehst. Ich würde nur gerne den Hintergrund besser verstehen. Deshalb drei Fragen:

1. Wie begründest Du mit der Quantentheorie, daß die Wellenfunktion etwas "Reales", bzw. "etwas Seiendes" ist? Oder anders, worauf beruht der Denkfehler von Leuten wie von Weizäcker, Zeilinger und anderen?

2. Kannst Du Textstellen benennen, die von Weizäcker, Zeilinger und andere widerlegen?

3. Bleibst Du bei Deiner Vorstellung einer über die Kugelschale verteilten Energiedichte? Du hast zugestimmt, daß diese beim Messereignis instantan kollabiert. Bist Du etwa der Meinung, daß diese Vorstellung nicht der SRT widerspricht?

Da hätte ich dann doch gerne die Wellenfunktion..In eine Superposition von was?
Zeilinger hat Beugungserscheinungen nachgewiesen. Mehr nicht.
Zeilger hat nacheinander Fulleren Moleküle im Mach-Zehnder-Interferometer in die Superposition linker Weg + rechter Weg gebracht. Dazu gibt es paper und sein Buch "Einsteins Schleier".

Gruß nach HK, Timm

P.S. Noch eine Bitte: rede die user mit ihrem user Namen an.

JoAx
16.04.12, 12:19
Hi Eugen!

Du hast folgendes zitiert:

Danach ist der quantenmechanische Zustand die Information, die wir über die Welt haben. … Es stellt sich letztlich heraus, dass Information ein wesentlicher Grundbaustein der Welt ist.


Das, was ich fett hervorgehoben habe, scheint mir doch nicht so gut zu der Einstellung zu passen, dass die QM nur eine Beschreibung ist. IMHO geht er hier eindeutig weiter.
Und so auch ein Gruß und Fragen an Timm:
Von welcher Welt spricht da Zeilinger im Zusammenhang mit Information als ihr Grundbaustein?
Müsste die Wellenfunktion danach nicht etwas Seiendes sein?


Gruß, Johann

Bauhof
16.04.12, 12:55
Zitat von Zeilinger: Danach ist der quantenmechanische Zustand die Information, die wir über die Welt haben. … Es stellt sich letztlich heraus, dass Information ein wesentlicher Grundbaustein der Welt ist.
Das, was ich fett hervorgehoben habe, scheint mir doch nicht so gut zu der Einstellung zu passen, dass die QM nur eine Beschreibung ist. IMHO geht er hier eindeutig weiter.
Hallo Johann,

man könnte das z.B. wie folgt interpretieren:
Information ist ein wesentlicher Grundbaustein zur Beschreibung der Natur.
Von welcher Welt spricht da Zeilinger im Zusammenhang mit Information als ihr Grundbaustein?
Ich denke, nur zur Beschreibung der physikalischen Welt.
Müsste die Wellenfunktion danach nicht etwas Seiendes sein?
Gruß, Johann
Die Wellenfunktion ist nach Max Born eine "mathematische Wahrscheinlichkeitswelle", mit der man statistische Vorhersagen machen kann. Sie ist im physikalischen Sinn nicht 'etwas Seiendes' (was auch immer 'etwas Seiendes' sein soll).

M.f.G Eugen Bauhof

JoAx
16.04.12, 13:10
man könnte das z.B. wie folgt interpretieren:
Information ist ein wesentlicher Grundbaustein zur Beschreibung der Natur.


In dem Falle hätte Zeilinger aber eine echt unglückliche Formulierung gewählt. Denn als erstes und natürliches denkt man an die Natur, und nicht die "Welt" der Theorie, wenn man einfach nur - Welt - liest.

Aber dann. Können diese Worte:

Wir müssen uns wohl von dem naiven Realismus, nach dem die Welt an sich existiert, ohne unser Zutun und unabhängig von unserer Beobachtung, irgendwann verabschieden.

dazu passen, dass es sich bei "Zeilingers Welt" um das Theoriegebäude handelt?

Ich sehe da kein Weg hin, ehrlich gesagt. Ich ersetze mal "Welt" mit "Theoriegebäude":

Wir müssen uns wohl von dem naiven Realismus, nach dem das Theoriegebäude an sich existiert, ohne unser Zutun und unabhängig von unserer Beobachtung, irgendwann verabschieden.Würde das irgendeinen vernünftigen Sinn ergeben? :confused:


Gruß, Johann

amc
16.04.12, 13:25
Moin JoAx, Moin Eugen,

Von welcher Welt spricht da Zeilinger im Zusammenhang mit Information als ihr Grundbaustein?
Müsste die Wellenfunktion danach nicht etwas Seiendes sein?

ich glaube, da muss man grundsätzlich erstmal vorsichtig sein. Obwohl mir Zeilingers Überlegungen auch manchmal leicht in diese Richtung zu gehen scheinen. Jedenfalls können solch Zitate aber, je nach Blickwinkel und Kontext, schnell unteschiedlich interpretiert werden.

Nach allem was ich bisher von Zeilinger gehört/gelesen habe, habe ich konkrete Information eigentlich immer als Gegenspieler des undefinierten "Zustandes" verstanden. Somit würde ich Information insofern als Grundbaustein verstehen, als das, wann immer Interferenzerscheinungen verringert werden, dies zwingend mit dem Vorhandensein bzw. einer Zunahme von konkreter Information einhergeht.

Dazu aus "Einsteins Schleier" (Goldmann Taschenbuch 3.Auflage S. 59):

Es ist also unerheblich, ob ein Beobachter über eine Messung Notiz davon nimmt, welchen Weg das Teilchen nimmt. Allein dass man diesen Weg bestimmen könnte, reicht aus, um die Interferenz zum Verschwinden zu bringen.
Man kann dies etwa in folgender Weise auf den Punkt bringen. Solange Information etwa darüber, welchen Weg das Teilchen durch den Spalt nimmt, irgendwo vorhanden ist - und das kann irgendwo im Universum sein - kann die komplementäre Größe, nämlich das Interferenzbild, nicht wohldefiniert sein.

Ich denke dies passt ganz gut. Auch in Bezug auf die Frage, ob Zeilinger einen Konstruktivismus vertritt. Offensichtlich nicht.

Grüße, AMC

Timm
16.04.12, 14:19
Und so auch ein Gruß und Fragen an Timm:
Von welcher Welt spricht da Zeilinger im Zusammenhang mit Information als ihr Grundbaustein?
Müsste die Wellenfunktion danach nicht etwas Seiendes sein?

Das glaube ich nicht, Johann. Zeilinger ist ja der Auffassung, daß die Wellenfunktion die Information einer statistischen Wahrscheinlichkeit bereitstellt, also Kopenhagerner Deutung. Er macht, wie viele andere auch, darauf aufmerksam, daß die Annahme, die Wellenfunktion sei etwas Reales (Seiendes), physikalisch unsinnig ist. Etwas "Reales", wie etwas RoKo's Energiedichte, gehorcht physikalischen Gesetzen, kollabiert also nicht instantan.

Gruß, Timm

JoAx
16.04.12, 14:27
Zeilinger ist ja der Auffassung, daß die Wellenfunktion die Information einer statistischen Wahrscheinlichkeit bereitstellt, [...] daß die Annahme, die Wellenfunktion sei etwas Reales (Seiendes), physikalisch unsinnig ist.

Dann frage ich mich, Timm, wie man da zur Aussage kommen kann, dass "die Information ein Grundbaustein der Welt" sein soll. ???
Was ist damit gemeint?


Gruß

Timm
16.04.12, 16:25
Dann frage ich mich, Timm, wie man da zur Aussage kommen kann, dass "die Information ein Grundbaustein der Welt" sein soll. ???
Was ist damit gemeint?

Johann, wenn ich das so genau wüßte. Ich denke aber, er bezieht sich auf die Bedeutung der Information auf der Quantenebene. Was man beobachtet, ist eng mit Information verknüpft. Das Interferenzbild hängt davon ab, ob oder ob nicht eine "welcher Spalt" Information existiert, wieviel Information darüber in die Umwelt gelangt ist usw. Dann fehlt noch die Brücke die Quantenebene, die ja nun wirklich fundamental ist, als Grundbaustein der Welt zu sehen.

Gruß, Timm

RoKo
16.04.12, 17:19
Hallo Timm,
.. Es ist für mich völlig akzeptabel, daß Du es anders siehst. Ich würde nur gerne den Hintergrund besser verstehen. Deshalb zwei Fragen:
Das ist ein mehr als abendfüllendes Programm. Ich versuche es trotzdem.

1. Wie begründest Du mit der Quantentheorie, daß die Wellenfunktion etwas "Reales", bzw. "etwas Seiendes" ist? Oder anders, worauf beruht der Denkfehler von Leuten wie von Weizäcker, Zeilinger und anderen?

A. Naturphilosophischer Ausgangspunkt:
A1. Es gibt eine von der Existenz intelligenter Wesen unabhängige objektive Realität.
A2. Diese objektive Realität ist materiell; wobei ich unter Materie mit Mario Bunge [1] all das verstehe, was verändert werden kann und selbst anderes verändert. (Der Physikalische Feldbegriff ist damit abgedeckt.)
(Natur, Universum, Kosmos sind mehr oder weniger Synonyme Begriffe für die materielle objektive Realität.)
A3. Die Natur befindet sich in einem ständigen Prozess der Selbstveränderung, den wir mit Evolution bezeichnen.
A4. Menschen sind ein Ergebnis dieses Evolutionsprozesses. Menschen sind materielle informationsverarbeitende Wesen.

B. Was ist Information?
B1. Die Natur ist in sich unterschiedlich, zeitlich, räumlich, energetisch etc. Das ist die materielle Basis jeglicher Information. Ebeling/Feistel [2] nennen es gebundene Information.
B2. Damit aus materiell gebundender Information freie Information wird, braucht es einen materiellen Informationsverarbeitungsprozess. Regenwürmer z.B. oder einen Sensor, der eine physikalische Größe in einen digitalen Wert wandelt, oder eben auch Menschen.
B3. Wenn Menschen Informationen haben, dann sind es Informationen über etwas; z.B. über Unterschiede oder Verhaltensweisen in der Natur. Sie haben nicht einfach nur Information. Letztere wird nämlich in Bit gemessen - und es nützt niemanden, wenn ich sage, auf der Festplatte meines Computers befinden sich jetzt 16 Gigabyte Information.

C. Gibt es ein Apriori jeglicher Erkenntnis?
C1. Diese Frage hat sich bereits Kant gestellt (und der ist mit seiner Antwort letztlich gescheitert.) Aber diese Frage ist nicht dumm oder gar falsch, weil sie herauszufinden versucht, ob es eine unbezweifelbare Grundlage unserer Wissenschaft gibt.
C2. Weizäcker greift diese Frage wieder auf und sein Schluss ist: Information ist das Apriori jeglicher Erkenntnis. Sein Schüler Lyre führt diesen Ansatz fort und hat dazu auch ein dickes Buch über Informationstheorie geschrieben. Diese Antwort ist ebenfalls nicht dumm! Diese Antwort greift aber aus zwei Gründen zu kurz.
C3. Erstens führt sie damit den Menschen (das informationsverarbeitende Wesen) als notwendige Voraussetzung einer solchen Wissenschaftsbegründung ein und verbaut sich damit die Möglichkeit, die Wissenschaft über die Evolution auf eine gleichberechtigte Basis zu stellen.
C4. Zweitens wird, siehe B1, die materielle Basis von Information übersehen.
C5. Meine Schlussfolgerung ist deshalb, an Begriff der materiellen objektiven Realität festzuhalten.

D. Kritik an der Ur-Kopenhagener Deutung
D1. Mit "Ur-KD" meine ich das, was von Bohr und Heisenberg, in der Solvey-Konferenz 1927 durchgesetzt wurde.
D2. Diese "Ur-KD" ist zu unterscheiden von der "Princeton"-Schule, die von der objektiven Existenz einer Quantenwelt ausgeht.
D3. Die philosophischen Grundlagen sind denen unter C kritisierten Auffassungen sehr ähnlich. Entgegen den immer wieder hier im Forum vorgetragenen Ansichten von Bauhof (z.B. heute 11:00) geht es nicht um Physik und Max Born würde sich möglicherweise im Grabe umdrehen, wenn hier sein Name genannt wird.
D4. Die "Ur-KD" ist wissenschaftstheoretisch zu kritisieren, weil sie keine Aussage über die offensichtlich unabhängig vom Menschen existierende Quantenwelt macht, sondern nur über das, was man mittels klassischer Messgeräte über sie herausfinden kann. Beobachter, Beobachtungsmittel und Beobachtungsgegenstand sind in dieser untrennbar miteinander verquickt. So kann nicht einmal aussagen, ob und wie weit die Beobachtungen zum Beobachtungsgegenstand gehören oder mehr dem Beobachtungsmittel geschuldet sind. Und als Basis für eine naturwissenschaftliche Evolutionstheorie taugt sie schon garnicht.
D5. Auch wenn es einige (Bauhof, "Ich") noch nicht ganz mitbekommen haben: die Standard-QM ist nicht die "Ur-KD". In der Standard-QM geht man davon aus, dass die Wellenfunktion einen Zustand eines real in der Natur vorkommenden Objektes beschreibt und der Kollaps der Wellenfunktion wird postuliert. Ansonsten geht man mit den Unzulänglichkeiten der QM höchst pragmatisch um.

E. Zur Wellenfunktion als Tool
E1. Die Wellenfunktion als reines Tool zur Berechnung von Messwerten bzw. als reine Information zu betrachten, ist logisch nur durchhaltbar, wenn man sich auf den bereits kritisierten philosophischen Standpunkt der "Ur-KD" stellt.
E2. Damit muss man zugleich den von Schrödinger immer wieder kritisierten Quantensprung annehmen und begibt sich physikalisch in Widerspruch zur Elektrodynamik. Eine ausführliche grafische Kritik findet sich hier. [3]
E3. Die Interpretation der Elektronenorbits (bzw. |Psi|²) im Atom als "Aufenthaltswahrscheinlichkeit" steht im Widerspruch zur Chemie. Diverse Bindungen lassen sich nur über die Interpretation als Ladungsdichte beschreiben.
E4. Nicht klassische Messgeräte wie ein Rastertunnelmikroskop oder nicht klassische Messungen oder unscharfe Messungen - das alles darf es nach der "Ur-KD" garnicht geben.
E5. Völlig unerklärt bleibt, wie den die Energie von dem zwecks Messung präparierten Objekt zum Messgerät fliesst.

F. Elektrodynamischer Exkurs
F1. Im EM-Feld steckt Energie - unabhängig, ob sie Wirkung erzielt oder nicht.
Energie kann deshalb als Wirkungspotenz, als Wirkungsmöglichkeit interpretiert werden. (Achtung, Timm - man muss also nicht, wie Stefan Kunz es tut, "Möglichkeit" als nicht real bezeichnen.)
F2. In der Elektrodynamik unterscheidet man zwischen Energiedichte und Energiestrom bzw. Energiestromdichte. Die Energiestromdichte muss dabei nicht proportional zur Energiedichte sein. Einzig der zum Ort der Wirkung führende Energiestrom wirkt tatsächlich (wenn man von Nebenwirkungen wie Wärmeentwicklung im Leiter o.ä. absieht.)
F3. Es gibt eine auffallende Analogie zwischen Quantenoptik und Quantenmechanik.
F4. In der Quantenoptik kann man alles klassisch erklären ausser der Absorbition eines Quantums Licht an einem Ort.
F5. Die klassische Elektrodynamik kann per se nicht im Widerspruch zur SRT stehen.

G. Nichtlokalität
G1. Das Prinzip der Verschränkung führt zwangsläufig zur Nichtlokalität.
G2. Es war zunächst Einstein mit seinen Assistenten, der im berühmten EPR-Papier darauf hingewiesen hat.
G3. Es war Schrödinger, der das Prinzip der Verschränkung im Detail ausgearbeitet hat und dabei die "Ur-KD" und die von Neumansche Simplifizierung ad absurdum geführt hat. (zu nichts anderem diente die Katze).
G4. Es war Bohm, der aus Einststeins Kritik einen praktikablen Versuch entwickelte.
G5. Es war Bell, der daraus seine Ungleichung entwickelte.
G6. Und es war Aspect, der das experimentell nachwies.
G7. Die Werke Einsteins, Schrödingers, Bohms und Bells sollte man schon zur Kenntnis genommen haben, wenn man nicht geistig im Jahre 1927 stehen bleiben will. (Ein bischen Polemik muß erlaubt sein. Herr Moderator.)

H. Mathematisches zur Wellenfunktion
H1. Die Schrödingergleichung ist eine Energiegleichung.
H2. Die Wellenfunktion beschreibt deshalb einen Energieausbreitungsprozess.
H3. Psi = energetischer Zustand
H4. Die Wellenfunktion lässt sich in Real- und Imagiärteil trennen; analog lässt sich ein Quantum EM-Welle auch als komplexe Funktion darstellen.
H5. Der Imaginärteil der Wellenfunktion beschreibt den Ort, der Realteil die Geschwindigkeit.
H6. Im Atom ist die Geschwindigkeit null; trotzdem gibt es noch einen Impuls. Hier steht die Welle.

S. Schlussfolgerungen
Die Punkte A-H bedenkend komme ich zum Schluss:
S1. Die Wellenfunktion ist etwas materielles, weil sie durch andere Materie veränndert werden kann und selbst andere Materie verändern kann.
S2. |Psi|² beschreibt die Energiedichte. Ihr proportional ist die Ladungsdichte und (ggf.) die Wahrscheinlichkeit, eine Wirkung zu erzielen.
S3. Da die QM nichtlokal ist, ist auch der nichtlokale Kollaps kein Problem.
S4. Wem der Kollaps nicht gefällt, der kann Anfangsbedingungen annehmen.

2. Kannst Du Textstellen benennen, die von Weizäcker, Zeilinger und andere widerlegen?

Morgen. Gute nacht.

JoAx
16.04.12, 17:45
Johann, wenn ich das so genau wüßte. Ich denke aber, ...

Timm, verstehe mich bitte nicht falsch, aber das geht am Kern meiner Frage(n) etwas vorbei.

Versteht man deine Ausführungen lediglich als "Hilfsbildchen" (nicht herablassend gemeint, und mit Stichwort "Vakuumfluktuationen"), um die Mathematik intuitiv nachvollziehen zu können, dann ist es völlig Ok., es geht imho aber nicht, dann von diesen, auf die Realität rück zu schließen.

Will man dahinter mehr sehen, was ich halt hinter Zeilingers Worten zu sehen meine (und nicht nur ich, richy bsw. auch, denke ich), ist es (wieder imho) nicht mehr lediglich statistische "Minimal"-Interpretation. Und dann kan man nicht Max Born im gleichen Atemzug zitieren.

Das ist meine Schwierigkeit. Ich meine - man kann die "Minimal"-Interpretation nicht beliebig mit zusätzlichen "Features" ausstatten und davon ausgehen, dass diese immer "Minimal" bleibt. Irgendwann ist sie das einfach nicht mehr. Ob man es nun wahr haben will oder nicht.


Gruß, Johann

Timm
16.04.12, 18:02
Hallo Johann,

Will man dahinter mehr sehen, was ich halt hinter Zeilingers Worten zu sehen meine (und nicht nur ich, richy bsw. auch, denke ich), ist es (wieder imho) nicht mehr lediglich statistische "Minimal"-Interpretation. Und dann kan man nicht Max Born im gleichen Atemzug zitieren.

Zeilinger outet sich bei diversen Gelegenheiten als Vertreter der minimalistischen Interpretation. Er ist voll bei Max Born.

Wenn ich Dich richtig verstehe, interpretierst Du Zeilinger anders als er sich äußert. Wozu soll das gut sein? Da kannst Du auch Grass unterstellen, er hätte kürzlich ein Liebesgedicht verfasst.

Gruß, Timm

RoKo
17.04.12, 00:42
Hallo Bauhof,

..
Die Wellenfunktion ist nach Max Born eine "mathematische Wahrscheinlichkeitswelle", mit der man statistische Vorhersagen machen kann. Sie ist im physikalischen Sinn nicht 'etwas Seiendes' (was auch immer 'etwas Seiendes' sein soll)...

Max Born & "mathematische Wahrscheinlichkeitswelle" ergibt bei Google genau einen Treffer -deinen Beitrag.

Timm
17.04.12, 09:57
Hallo RoKo,

Mit Deine Folgerungen


S1. Die Wellenfunktion ist etwas materielles, weil sie durch andere Materie veränndert werden kann und selbst andere Materie verändern kann.
S2. |Psi|² beschreibt die Energiedichte. Ihr proportional ist die Ladungsdichte und (ggf.) die Wahrscheinlichkeit, eine Wirkung zu erzielen.
S3. Da die QM nichtlokal ist, ist auch der nichtlokale Kollaps kein Problem.

stehst Du im Widerspruch zu KD.

Dieses Zitat aus wikipedia (http://de.wikipedia.org/wiki/Lokalit%C3%A4t_(Physik))

Dabei zeigt sich, dass zwar der Kollaps der Wellenfunktion instantan erfolgt, jedoch keine echten Informationen übertragen werden können, sodass die Einstein-Kausalität dennoch erhalten bleibt. Diese Ergebnisse entsprechen der Kopenhagener Interpretation der Quantenmechanik, die der Wellenfunktion keine unmittelbare physikalische Realität zuschreibt, sondern nur den Messergebnissen. Der "Kollaps" der Wellenfunktion ist daher kein physikalisches Phänomen, das mit Lichtgeschwindigkeit "übertragen" werden müsste.
mag als Beispiel für viele ähnlich lautende Beschreibungen dienen.

Mit Deiner privaten Deutung (Wellenfunktion -> materiell/Energiedichte) ist der nicht-lokale Kollaps sehr wohl ein Problem, denn er verletzt die SRT, wie schon weiter oben angemerkt. Vielleicht ist Dir das entgangen.

Gruß, Timm

Hawkwind
17.04.12, 10:12
Hallo,

mal keine Zitate großer Physiker, nur ein paar Gedanken von mir: :)

was in der Diskussion vielleicht etwas untergeht: die Wellenfunktion ist ein Konzept zur Ermöglichung von Vorhersagen statistischer Natur. Sie entfaltet ihren Sinn erst, wenn man ein ganzes Ensemble unabhängiger und gleicher Experimente durchführt.

Es ist die Frage, ob die Diskussion von Details, was mit der Wellenfunktion denn bei einer einzelnen Messung passiert, überhaupt allzu sinnvoll sind.

Klar ist, will man nun nach dieser Messung wiederum ein Ensemble gleichartiger Messungen dieses bereits vermessenen Systems durchführen, so braucht man eine andere Wellenfunktion als vor der ersten Messung, denn an dem bereits vermessenen System wird man nun immer wieder den bei der 1. Messung festgestellten Wert reproduzieren. Es macht also Sinn, eine Eigenfunktion zum gemessenen Wert zu wählen; das ist der "Kollaps" - eine Wahl von uns, ein nützliches Konstrukt, wenn man wie in der Kopenhagener Deutung (eigentlich wider besseres Wissen) darauf besteht die Entwicklung der Wellenfunktion "nicht-statistisch" (an einer einzelnen Messung) zu diskutieren.

Viele Welten umschifft diese Kopenhagener Schwäche, indem angenommen wird, es gibt gar kein einzelnes Experiment: es wird vielmehr sowieso immer alles gemessen, was möglich ist - halt nur von Beobachtern, die sich samt ihrer Ergebnissse duplizieren. Dadurch hat man hier diesen Kollaps nicht so als Bruch, zahlt aber einen gewissen "Preis" für die m.E. grotesk anmutenden Duplizierungen der Welten.


Gruß,
Hawkwind

JoAx
17.04.12, 10:20
Wozu soll das gut sein?


Timm, ich hatte doch nur paar Fragen. Und nebenbei habe ich "etwas" dazu geschrieben, damit es verständlich wird, warum ich diese Fragen habe. Das ist doch keine böse Absicht von mir. :D Und aufs Wort glauben ... das muss ja auch nicht sein.

... jedoch keine echten Informationen übertragen werden können ...

Muss man dazu etwas sagen? Baut Zeilinger seine philosophischen Aussagen auf diesen unechten Informationen, als Grundbaustein der echten Welt?

Ich frage nur.


Gruß, Johann

JoAx
17.04.12, 10:25
die Wellenfunktion ist ein Konzept zur Ermöglichung von Vorhersagen statistischer Natur.


d'accord

Gruß

Bauhof
17.04.12, 12:56
Zitat von Hawkwind
die Wellenfunktion ist ein Konzept zur Ermöglichung von Vorhersagen statistischer Natur.

d'accord
Hallo Johann,

ich denke dir ist klar, dass in der Quantenmechanik diese "Vorhersagen statistischer Natur" prinzipiell anderer Natur sind als die statistischen Vorhersagen in der klassischen Physik. Aber ich will mich nur vergewissern, ob wir dasselbe dabei meinen:

In der Quantenmechanik ist der tiefere Grund für das Auftreten statistischer Zusammenhänge die Heisenbergsche Unbestimmtheit. Einzelne Ereignisse in der Gegenwart und der Zukunft sind durch die Vergangenheit nicht vollkommen vorherbestimmt.

Max Born führte die Annahme ein, dass die Wahrscheinlichkeitswellen nicht Felder einer Art Materie, die sich im Raum ausdehnt, darstellten, sondern dass sie nur ein mathematisches Hilfsmittel seien, das statistische Verhalten von Teilchen auszudrücken.

Lew Tarassow formuliert das in seinem Buch [1] auf Seite 164 wie folgt:

Nachdem sich die Physiker davon überzeugt hatten und die Versuche gescheitert waren, die "de Brogliesche Welle" mit einem materiellen Wesen zu versehen, mussten die Forscher anerkennen, dass diese "Wellen" nichts Gemeinsames mit wirklich existierenden Wellen haben. Nicht von ungefähr entstand eine recht beeindruckende Bezeichnung - die Wahrscheinlichkeitswellen. Allmählich wurde der Begriff Wellenmechanik durch Quantenmechanik von allen seinen Positionen verdrängt, die Wellenfunktion wurde durch Wahrscheinlichkeitsamplitude ersetzt.


M.f.G. Eugen Bauhof

[1] Tarassow, Lew
Wie der Zufall will?
Vom Wesen der Wahrscheinlichkeit. (http://www.amazon.de/Wie-Zufall-will-Wesen-Wahrscheinlichkeit/dp/3860253069/ref=sr_1_1?s=books&ie=UTF8&qid=1334663605&sr=1-1)
Heidelberg 1993
ISBN=3-86025-306-9

RoKo
17.04.12, 13:29
Hallo Timm,

Mit Deine Folgerungen .. stehst Du im Widerspruch zu KD... mit der ich, wie zuvor in D und E dargelegt, unzufrieden bin.

[QUOTE].. mag als Beispiel für viele ähnlich lautende Beschreibungen dienen... die mir selbstverständlich bekannt sind.

Mit Deiner privaten Deutung (Wellenfunktion -> materiell/Energiedichte) ..hier habe wohl nicht deutlich genug zum Ausdruck gebracht, dass es sich um die Energie- und Ladungsdichte eines unteilbaren Objektes handelt, das nur in seiner Gesamtheit eine Wirkung erzielen kann - eben das Prinzip der kleinsten Wirkung. Bekanntermaßen ist eine Elementarladung auch unteilbar.

von wikipedia: Dabei zeigt sich, dass zwar der Kollaps der Wellenfunktion instantan erfolgt, jedoch keine echten Informationen übertragen werden können, sodass die Einstein-Kausalität dennoch erhalten bleibt. Das sehe ich nicht anders, von daher habe ich auch kein Problem mit der SRT. Mal davon abgesehen, dass instantan ebenso eine Abstraktion ist wie Punktförmigkeit.

RoKo
17.04.12, 14:12
Hallo Bauhof,

Max Born führte die Annahme ein, dass die Wahrscheinlichkeitswellen nicht Felder einer Art Materie, die sich im Raum ausdehnt, darstellten, sondern dass sie nur ein mathematisches Hilfsmittel seien, das statistische Verhalten von Teilchen auszudrücken.Aus dem Originaltext geht das nicht hervor.

RoKo
17.04.12, 14:20
In der Quantenmechanik ist der tiefere Grund für das Auftreten statistischer Zusammenhänge die Heisenbergsche Unbestimmtheit. Einzelne Ereignisse in der Gegenwart und der Zukunft sind durch die Vergangenheit nicht vollkommen vorherbestimmt.
..wobei sich die Heisenbergsche Unbestimmtheit wiederum aus dem wellenartigen Charakter der Quantenobjekte ergibt und von deren Wellenfunktion "automatisch" (Schrödinger, 1935) erfüllt wird.

Hawkwind
17.04.12, 14:53
Hallo Bauhof,

Aus dem Originaltext geht das nicht hervor.

Born erhielt 1954 den Nobelpreis für die statistisch mathematische Interpretation Wellenfunktion.

Danach ist die Wellenfunktion ein "Tool" um Vorhersagen probabilistischer Natur zu machen. Selbst wenn das nirgends wortwörtlich so steht. :)

Gruß,
Hawkwind

Timm
17.04.12, 15:20
was in der Diskussion vielleicht etwas untergeht: die Wellenfunktion ist ein Konzept zur Ermöglichung von Vorhersagen statistischer Natur. Sie entfaltet ihren Sinn erst, wenn man ein ganzes Ensemble unabhängiger und gleicher Experimente durchführt.
Sehe ich auch so, die Ensemble Interpretation (http://en.wikipedia.org/wiki/Ensemble_interpretation) hatten wir übrigens vor nicht allzu langer Zeit schon mal diskutiert. Ein sehr ernst zunehmender Denkansatz, wie ich finde. Beschränkt man die Anwendung der Wellenfunktion auf das Ensemble, ist man den Kollaps beim einzelnen Teilchen und damit die Spekulation der Wellenfunktion einen materiellen Charakter zuzubilligen auf elegante Weise los.

Viele Welten umschifft diese Kopenhagener Schwäche, indem angenommen wird, es gibt gar kein einzelnes Experiment: es wird vielmehr sowieso immer alles gemessen, was möglich ist - halt nur von Beobachtern, die sich samt ihrer Ergebnissse duplizieren. Dadurch hat man hier diesen Kollaps nicht so als Bruch, zahlt aber einen gewissen "Preis" für die m.E. grotesk anmutenden Duplizierungen der Welten.

Genau, in der VWI gibt es keinen Kollaps, da ja alle Möglichkeiten ausgeschöpft werden. Hingegen schon eine "materielle" (um bei RoKo zu bleiben) Bedeutung der Wellenfunktion, Zeh: "Wozu braucht man viele Welten in der Quantentheorie". Mir scheint das die Gretchenfrage zu sein, die wir hier ja auch im Fokos haben.

Hier der Versuch den Deutungen Bausteine zuzuordnen:

----------------------------- Wellenfunktion materiell ----- Kollaps
Kopenhagener Deutung ............... nein ........................ ja
Ensemble Deutung ...................... nein ....................... nein
VWI ........................................... ja .......................... nein
RoKo Deutung ............................. ja .......................... ja

So zumindest habe ich RoKo verstanden. Damit wären alle Baustein-Möglichkeiten ausgeschöpft.

Gruß, Timm

Bauhof
17.04.12, 16:24
Aus dem Originaltext geht das nicht hervor

Hallo RoKo,

hier einige Textstellen aus den Schriften von Max Born und eine Textstelle bei Hans Reichenbach, aus dem das indirekt hervorgeht:

Aus [1], Seite 33:
Fragen wir also nicht, wo ist ein Teilchen genau, sondern begnügen wir uns zu wissen, dass es in einem bestimmten größeren Raumteil ist: dann verschwindet der Widerspruch zwischen Wellen- und Korpuskulartheorie.
Man sieht dies am leichtesten ein, wenn man der Welle die Funktion zuschreibt, die Wahrscheinlichkeit für das Auftreten eines Teilchens zu bestimmen, wobei die Energie des Teilchens mit den in der Welle vorhandenen Schwingungszahlen durch die Plancksche Relation verknüpft ist.

Aus [1], Seite 10:
Dies legt den Gedanken nahe, dass die Quantenmechanik ebenfalls nur Antwort gibt auf richtig gestellte statistische Fragen, aber im allgemeinen die Frage nach dem Ablauf eines Einzelprozesses unbeantwortet lässt.

Aus [1], Seite 17:
Wentzel und Oppenheimer haben gezeigt, dass man tatsächlich die Rutherfordsche Formel für die Anzahl der gestreuten Teilchen bekommt, wenn man die Intensität der Schrödinger-Welle als Maß der Wahrscheinlichkeit annimmt.

Aus [2], Seite 93:
Die neue Mechanik beantwortet nicht, wie die alte, die Frage: "wie bewegt sich ein Teilchen", sondern die Frage: "wie wahrscheinlich ist es, dass ein Teilchen sich in gegebener Weise bewegt".

Aus [3], Seite 33:
Ein Versuch, die beiden Interpretationen zu vereinigen, wurde von Born gemacht, der die Annahme einführte, dass die Wellen nicht Felder einer Art Materie, die sich im Raum ausdehnt, darstellten, sondern dass sie nur ein mathematisches Hilfsmittel seien, das statistische Verhalten von Teilchen auszudrücken.

Mit freundlichen Grüßen
Eugen Bauhof

[1] Born, Max
Physik im Wandel meiner Zeit.
(http://www.amazon.de/Physik-im-Wandel-meiner-Zeit/dp/3528085398/ref=sr_1_1?s=books&ie=UTF8&qid=1334675321&sr=1-1)Braunschweig 1983. ISBN=3-528-08539-8

[2] Born, Max
Zur statistischen Deutung der Quantentheorie.
(http://www.amazon.de/statistischen-Deutung-Quantentheorie-Redaktion-Hermann/dp/B006C5EIIM/ref=sr_1_18?s=books&ie=UTF8&qid=1334675440&sr=1-18)Stuttgart 1962

[3] Kamlah, Andreas (Hrsg.)
Reichenbach, Maria (Hrsg.)
Hans Reichenbach. Gesammelte Werke.
Band 5: Philosophische Grundlagen der Quantenmechanik und Wahrscheinlichkeit.
(http://www.amazon.de/Gesammelte-Werke-B%C3%A4nden-Quantenmechanik-Wahrscheinlichkeit/dp/3528083654/ref=sr_1_1?s=books&ie=UTF8&qid=1334675368&sr=1-1)Braunschweig 1979. ISBN=3-528-08365-4

RoKo
17.04.12, 17:17
Hallo,

ich hatte gerade die Lobreden (http://www.dpg-physik.de/veroeffentlichung/reden/urban/red_max_born_50.html) zum 50.Jahrestag der Nobelpreisverleihung an Max Born durchgesehen und bin dort auf folgendes Zitat aus seiner Rede gestossen:

von Max Born: "Ich wurde dabei von einer Bemerkung Einsteins über die Bedeutung der Lichtintensität (d.h. einer elektromagnetischen welle) unter dem Photonenaspekt geleitet: Diese Intensität muß die Anzahl der Photonen darstellen, aber letztere war natürlich statistisch als Mittel einer gewissen Photonenverteilung aufzufassen. Einstein hatte einige tiefsinnige Betrachtungen über die statistische Natur dieser Verteilung angestellt …. Diese Überlegungen waren mir wohlbekannt und führten mich unmittelbar zu der Vermutung, daß die Intensität der de Broglie-Welle, d.h. das Quadrat von Schrödingers Wellengleichung, als die Wahrscheinlichkeitsdichte angesehen werden mußte, als die Wahrscheinlichkeit, ein Teilchen in einer Volumeneinheit zu finden."

Fairerweise muss ich zugeben, dass sich das etwas anders liest, als in o.a. erster Mitteilung. Dennoch möchte ich auf den Aspekt hinweisen, dass bei Born, anders als in der "Ur-KD", Born kein "Anti-Realist" (nach dem Motto "alles ist Information") war. Wenn man die klassische Vorstellung vom Massepunkt retten will, dann kommt man nur zu einer statistischen Deutung der Wellenfunktion oder über die Annahme von Anfangswerten zur Bohmschen Mechanik.

Ich halte aber die Annahme von Punktteilchen für problematisch, weil sie zu anderen Problemen führt. Deshalb suche ich nach anderen Deutungen. Die VWI habe ich abgehakt, weil sie zu sehr auf simplizierten Annahmen beruht.

Das die Probleme noch lange nicht gelöst sind, machte im übrigen auch der Lobredner deutlich:
von Prof. Dr. Knut Urban, Präsident der Deutschen Physikalischen Gesellschaft:
Dennoch ist die Interpretation der Wellenfunktion bis heute ein physikalisch und vor allem erkenntnistheoretisch nicht gelöstes Problem, das wir von der Physik des zwanzigsten in die des einundzwanzigsten Jahrhunderts herübergenommen haben. Kollaps- und Everitt-Interpretationen sind aktuelle Versionen dieser Versuche, unser an der Klassischen Physik orientiertes Vorstellungsvermögen mit den Aussagen der Quantentheorie zu synchronisieren.

Im übrigen sind meine Überlegungen nicht so ganz "privat". Am Karlsruher Institut für Didaktik der Physik stellt man ähnliche Überlegungen an. Z.b. hier. (http://www.pohlig.de/Physik/PantaRhei/Vortraege/Stroeme_in_der_Atomhuelle.pdf)

Bauhof
17.04.12, 18:20
Hallo, ich hatte gerade die Lobreden (http://www.dpg-physik.de/veroeffentlichung/reden/urban/red_max_born_50.html) zum 50.Jahrestag der Nobelpreisverleihung an Max Born durchgesehen und bin dort auf folgendes Zitat aus seiner Rede gestossen:
Hallo RoKo,

und mir erschient dieses Zitat wichtig, denn Wolfgang Pauli hatte fast immer recht behalten:

von Prof. Dr. Knut Urban, Präsident der Deutschen Physikalischen Gesellschaft:
Auf alle Fälle hat Wolfgang Pauli rechtbehalten, als er an Max Born schrieb "Ich bin gewiß, daß der statistische Charakter der Naturgesetze - auf dem Sie von Anfang an gegen Schrödingers Widerstand bestanden haben - den Stil der Gesetze wenigstens für einige Jahrhunderte bestimmen wird."

M.f.G. Eugen Bauhof

RoKo
18.04.12, 08:01
Hallo Bauhof,

gegen den statistischen Charakter der quantenmechanischen Gesetze habe ich auch nie argumentiert, sondern lediglich gegen ihre antirealistische Deutung.

JoAx
18.04.12, 08:19
Hi, AMC!

Es geht mir nicht um Konstruktivismus bei Zeilinger. (Maaan! Es gibt wohl für alles ein Wörtchen. :D ) Das ist mir auch gar nicht in den Sinn gekommen. Es geht mir darum, tatsächlich 100%-ig belastbare Aussagen von solchen zu trennen, die aus dem Blickwinkel einer Interpretation getroffen werden oder wurden, und damit vlt. nur eine Angelegenheit der Auslegung sind. In etwa - man kann es so formulieren, muss man aber nicht.

Z.B. - die Wellenfunktion ist nur ein Instrument, mathematisches Werkzeug.
Gibt es Experimente, die gegen diese Annahme sprechen würden?
Kann so etwas durch ein Experiment überhaupt entschieden werden?

Warum ich das frage? - Weil jede Interpretation (ausser der Ur-KD vlt., wie RoKo es ausdrückte) imho davon ausgeht, dass es mehr ist, als das.


Gruß, Johann

Hawkwind
18.04.12, 08:42
Z.B. - die Wellenfunktion ist nur ein Instrument, mathematisches Werkzeug.
Gibt es Experimente, die gegen diese Annahme sprechen würden?
Kann so etwas durch ein Experiment überhaupt entschieden werden?

Warum ich das frage? - Weil jede Interpretation (ausser der Ur-KD vlt., wie RoKo es ausdrückte)


und v.a. die minmale Interpretation (siehe z.B. Timms Link oben, dort "Ensemble-Interpretation" genannt).



imho davon ausgeht, dass es mehr ist, als das.


Gruß, Johann

Naja, wir befinden uns hier schon lang in einem "Wischiwaschi"-Bereich.
Welche Anforderungen muss denn die Wellenfunktion erfüllen, um mehr als ein mathematisches Werkzeug zu sein? Reicht es, Element eines etablierten Modells zu sein oder forderst du Beobachtbarkeit?

Fest steht, die Wellenfunktion als Funktion von zeitlichen und räumlichen Koordinaten (in der klassischen Physik nennt man so etwas ja "Feld") ist kein
beobachtbares Feld.

Es drängt sich immer die Analogie zwischen Maxwell-Gleichungen und Schrödingergleichung auf - beides partielle Differentialgleichungen, deren Lösungen Wellennatur haben. Bei den Maxwellgleichungen ist die Lösung das elm. Feld; es kann unmittelbar an jedem Raumzeitpunkt über die Kraft auf eine Probeladung etc. vermessen werden. In diesem Sinne "existiert" es (Realität würde ich in der Physik über Beobachtbarkeit/Messbarkeit definieren). In demselben Sinne "existiert" die quantenmechanische Wellenfunktion nicht: sie ist lediglich Teil eines "Kochrezepts", um Vorhersagen machen zu können. Ich kann höchstens mittels eines ganzen Ensembles gleichwertiger Messungen ihr über einen Raumbereich integriertes Betragsquadrat vermessen.

Man mag "Realität" aber vielleicht auch weniger streng definieren, etwa in dem Sinne, dass man sagt, Elemente eines überwältigend gestützten Modells betrachten wir als "real existierend".

Sind die Interpretationen nicht letztlich ein Versuch, die vertrauten Paradigmen der klassischen Physik auch der Quantentheorie "aufzudrängen"?

Gruß,
Hawkwind

amc
18.04.12, 13:14
Hi,

Es geht mir nicht um Konstruktivismus bei Zeilinger. (Maaan! Es gibt wohl für alles ein Wörtchen. :D ) Das ist mir auch gar nicht in den Sinn gekommen.

ok, ich habe nicht deutlich gemacht, dass ich dich auch gar nicht so verstanden hatte. Ich habe es deshalb erwähnt, weil die Frage, ob Zeilinger einen Konstruktivismus vertritt, in Bezug auf genau jenes Zitat, hier in den letzten Wochen bereits schon mal aufgekommen ist. Und es eignet sich ja auch hervorragend, um einen Konstruktivismus zu propagieren. Darum der Hinweis von mir.

Z.B. - die Wellenfunktion ist nur ein Instrument, mathematisches Werkzeug.
Gibt es Experimente, die gegen diese Annahme sprechen würden?
Kann so etwas durch ein Experiment überhaupt entschieden werden?
Man mag "Realität" aber vielleicht auch weniger streng definieren, etwa in dem Sinne, dass man sagt, Elemente eines überwältigend gestützten Modells betrachten wir als "real existierend".

Ich denke, da gibt es im Grunde letztendlich wenig Spielraum. Etwas, das Anspruch auf Realität haben möchte, das muss auch immer durch etwas materielles repräsentiert sein. Wobei die Frage danach, was Materie eigentlich genau ist/sein kann, noch unbeantwortet bleibt. Vielleicht kommt man auch irgendwann zu dem Schluss, dass es auch nichtlokal real existierende Materie gibt, möglicherweise zusammengefasst durch den Begriff Information. Wäre sicherlich äußerst spannend dies zu erleben, auch gerade wie sich dies dann mit der RT verträgt. Da kann ich mir grundsätzlich viel vorstellen, bin dabei aber möglichst zurückhaltend - eben nur das annehmen, was unbedingt nötig erscheint.

In jedem Fall muss sich "Reales" nachweislich bemerkbar machen, es muss Energie tragen, es muss irgendwie zu bemessen sein. Richy verwies hierbei schon mal auf den Begriff der "Wägbarkeit". Finde ich recht zutreffend. Ich sage mal ganz laienhaft: Wenn die Wahrscheinlichkeitswelle physikalisch real ist, dann muss sich das z.B. in den Energieerhaltungsätzen bzw. in den Berechnungen zur Energieerhaltung wiederfinden lassen. IMHO - DANKE TSCHÜSS :)

Grüße, AMC

JoAx
18.04.12, 13:55
und v.a. die minmale Interpretation (siehe z.B. Timms Link oben, dort "Ensemble-Interpretation" genannt).


Interessant, dass die Ensemble-Interpretation sich auch auf Max Born beruft:

At its heart, it takes to the fullest extent the statistical interpretation of Max Born ...

die ich aber als Gegenspielerin jeder Interpretation verstehe, die davon ausgeht, dass die Wellenfunktion auch ein einzelnes Objekt vollständig beschreibt. KD/KI gehört auch dazu. Überhaupt, versteht sie ja die Statistik der QM klassisch, wenn ich das richtig verstehe. Und wenn man den kleinen Auszug aus Max Borns Arbeit betrachtet (was Rolf eingestellt hat), ich denke aber, es geht auch aus der ganzen Arbeit hervor, dann hat er das wohl auch klassisch gemeint. Heute würde man da verborgene Variable - klar definierte Ort/Impuls - sehen, und aus diesem Grund diese Haltung ablehnen. Oder?
Wie beschreibt bsw. die Ensemble-Interpretation das Elitzur-Vaidman-Schema (http://de.wikipedia.org/wiki/Wechselwirkungsfreie_Quantenmessung#Elitzur-Vaidman-Schema)?


Naja, wir befinden uns hier schon lang in einem "Wischiwaschi"-Bereich.


Kann man zumindest klären, ob dieses "Wischiwaschi" sein muss?
"Ob Max Born sein Artikel Heute immer noch so verfasst hätte" - so zu sagen?


Welche Anforderungen muss denn die Wellenfunktion erfüllen, um mehr als ein mathematisches Werkzeug zu sein? Reicht es, Element eines etablierten Modells zu sein oder forderst du Beobachtbarkeit?


Deswegen frage ich ja.


Fest steht, die Wellenfunktion als Funktion von zeitlichen und räumlichen Koordinaten ist kein
beobachtbares Feld.


Fest steht aber auch, dass man ohne dieses unbeobachtbare Feld nicht zu korrekten Vorhersagen kommen kann. Oder doch?


Ich kann höchstens mittels eines ganzen Ensembles gleichwertiger Messungen ihr über einen Raumbereich integriertes Betragsquadrat vermessen.


Ja. Aber kann das Bild, das durch ein ganzes Ensemble gleichwertiger Messungen entsteht, auch entstehen, wenn nicht jede einzelne Messung dem "selben Algorithmus" folgt?


Sind die Interpretationen nicht letztlich ein Versuch, die vertrauten Paradigmen der klassischen Physik auch der Quantentheorie "aufzudrängen"?


Diese frage schwebt auch immer über mir. :)
Und trotz des ganzen Schleiers über der QM, lässt sich doch einiges darüber aussagen. Und wenn es nur die Form hat - "das ist die QM nicht".


Gruß, Johann

Timm
18.04.12, 13:56
Wenn die Wahrscheinlichkeitswelle physikalisch real ist, dann muss sich das z.B. in den Energieerhaltungsätzen bzw. in den Berechnungen zur Energieerhaltung wiederfinden lassen.
Insbesondere würde dann ein instantaner Kollaps eine Verletzung der SRT bedeuten.

Gruß, Timm

RoKo
18.04.12, 15:20
Hallo zusammen,

... (Realität würde ich in der Physik über Beobachtbarkeit/Messbarkeit definieren).
..Man mag "Realität" aber vielleicht auch weniger streng definieren, etwa in dem Sinne, dass man sagt, Elemente eines überwältigend gestützten Modells betrachten wir als "real existierend".
Ich sehe das so: Eine Theorie muss Beobachtbares widerspruchsfrei vorhersagen und erklären können. Welche Elemente der Theorie dann als "real existierend" anzunehmen sind, sollte sich dann aus der Theorie ergeben. Diese Elemente müssen nicht selbst zwingend beobachtbar sein. Ihre empirische Relevanz kann sich auch dadurch zeigen, dass ohne diese Elemente Beobachtbares nicht mehr vorhergesagt und erklärt werden kann. (Man möge mal versuchsweise die Wellenfunktion aus der Theorie entfernen!).

Das Problem der Standard-QM ist nun, dass sie derart pragmatisch (durch ihre Postulate) formuliert ist, dass sich viel Interpretationsspielraum ergibt.

Aber: Wer A sagt, der muss auch B sagen. Wenn Quantenobjekte bzw. Systeme als "real existierend" angenommen werden, wenn wir also davon ausgehen, dass es einen abgrenzbaren Teilbereich der Natur gibt, der sich entsprechend unserer Theorie verhält, dann hat das weitere Konsequenzen.

Wenn wir also ein "real existierendes" Quantenobjekt präparieren, dann bringen wir es in einen Zustand. Das ist dann eine "real existierende" Eigenschaft des "real existierenden" Quantenobjektes. Wenn wir das Quantenobjekt nun eine Weile sich selbst und dem Einfluss seiner Umgebung überlassen, dann verändert es sich. Das ist ein stets überprüfbarer Fakt! Unsere Theorie sagt uns, dass sich das Objekt gemäß einer Wellenfunktion verändert. Das ist ein unbestrittener Bestandteil der Theorie! Folglich ist die Wellenfunktion nicht nur ein mathematisches Tool, sondern mindestens in dem Sinne "real existierend", wie Ort, Impuls und Kräfte in der klassischen Mechanik als "real existierend" angenommen werden. Die Wellenfunktion spielt genau diese Rolle - aber eben auf eine nicht klassisch zu beschreibende Weise).

Die Krux ist nun, dass sich das Wirken seiner Wellenfunktion auf ein Quantenobjekt nur statistisch erfassen lässt. Die Theorie sagt uns nun aber auch, warum das so ist:
- Der (energetische) Zustand und sein Absolutquadrat sind die einzigen Eigenschaften, die das Objekt real hat. Alle "messbaren" Eigenschaften sind kontextuelle Eigenschaften.
- Ein Quantenobjekt kann nur als Ganzes verändert werden oder wirken.
- Jede (energetische) Wirkung bedeutet Zu- oder Abnahme von ganzzahligen Wirkungsquanten.
- Jede (energetische) Wirkung verändert damit den (energetischen) Zustand des ganzen Quantenobjektes. Da das Objekt in der Regel räumlich ausgedehnt ist, ist dies ein "nichtlokales Feature".



Sind die Interpretationen nicht letztlich ein Versuch, die vertrauten Paradigmen der klassischen Physik auch der Quantentheorie "aufzudrängen"?
Nein. Es geht eher um eine Justierung der verschiedenen Modellvorstellungen, so dass sie ein widerspruchsfreies Gesamtbild ergeben.

JoAx
18.04.12, 15:51
Hi, Eugen!


ich denke dir ist klar, dass in der Quantenmechanik diese "Vorhersagen statistischer Natur" prinzipiell anderer Natur sind als die statistischen Vorhersagen in der klassischen Physik.


Wie siehst du Max Born's Aussagen über den statistischen Charakter der Wellenfunktion?
Sind sie "klassischer" oder "prinzipiell anderer" Natur?


Gruß, Johann

Hawkwind
18.04.12, 17:04
Hallo zusammen,

Ich sehe das so: Eine Theorie muss Beobachtbares widerspruchsfrei vorhersagen und erklären können. Welche Elemente der Theorie dann als "real existierend" anzunehmen sind, sollte sich dann aus der Theorie ergeben. Diese Elemente müssen nicht selbst zwingend beobachtbar sein. Ihre empirische Relevanz kann sich auch dadurch zeigen, dass ohne diese Elemente Beobachtbares nicht mehr vorhergesagt und erklärt werden kann. (Man möge mal versuchsweise die Wellenfunktion aus der Theorie entfernen!).


Keine Ahnung, ich bin kein Experte; aber meines Wissens gibt es Formulierungen der Quantentheorie ohne Schrödingergleichung und Wellenfunktion.

In Heisenbergs Matrizenmechanik, z.B.
http://en.wikipedia.org/wiki/Matrix_mechanics
kommst du ohne explizite Wellenfunktion und Schrödingergleichung aus. Schau dir an, wie du den harmonischen Oszillator ohne explizite Berechnung der Wfkt. mittels Schrödingergleichung rein algebraisch berechnen kannst. Heisenbergs Matrizenmechanik entstand ja noch vor Schrödingers Formulierung, die dann Heisenbergs Matrizenmechanik in der Beliebtheit schnell überholt hatte. Die Energie-Eigenzustände werden in diesem Formalismus als Zustandsvektoren und weniger als Funktionen gesehen.

Meines Wissens benötigst du auch in Feynmans Pfadintegral-Formulierung
http://www.quantumfieldtheory.info/Path_Integrals_in_Quantum_Theories.htm der Quantenmechanik keine Wellenfunktion. Dort werden unmittelbar Übergangswahrscheinlichkeiten bzw. Übergangsamplituden für Prozesse berechnet.

Man kann in beiden Fällen jedoch Äquivalenz zur Schrödingerschen Formulierung nachweisen. Die Wellenfunktion "existiert" also in einer bestimmten Formulierung der Quantenmechanik. Mir scheint, sie ist doch ein sehr vom Modell abhängiges Feature; "da draußen" gibt es sie nicht und da kollabiert sie auch nicht.

Gruß,
Hawkwind

Philipp Wehrli
18.04.12, 21:00
Man mag "Realität" aber vielleicht auch weniger streng definieren, etwa in dem Sinne, dass man sagt, Elemente eines überwältigend gestützten Modells betrachten wir als "real existierend".

Sind die Interpretationen nicht letztlich ein Versuch, die vertrauten Paradigmen der klassischen Physik auch der Quantentheorie "aufzudrängen"?

Der Punkt ist, dass du das Element 'Wellenfunktion' nicht aus dem Modell weglassen kannst, ohne dass die ganze Theorie zusammen bricht. Wenn mir jemand sagen kann, wie man den Doppelspaltversuch ohne Wellen oder etwas Analoges korrekt beschreiben kann, und wenn diese Beschreibung nicht wesentlich komplizierter ist, als die mit Wellen, dann glaube ich auch nicht mehr, dass die Wellen real sind. Aber wenn Wellen zwingend nötig sind, wenn wir die Sache ausrechnen wollen, dann braucht die Natur sie auch. Woher soll denn die Natur sonst das Resultat kennen?

Philipp Wehrli
18.04.12, 21:09
Wenn die Wahrscheinlichkeitswelle physikalisch real ist, dann muss sich das z.B. in den Energieerhaltungsätzen bzw. in den Berechnungen zur Energieerhaltung wiederfinden lassen.

Wie willst du denn die Energieverteilung beim Doppelspaltversuch beschreiben, wenn du nicht irgendwo eine Wellenfunktion eingibst?

Philipp Wehrli
18.04.12, 21:20
Keine Ahnung, ich bin kein Experte; aber meines Wissens gibt es Formulierungen der Quantentheorie ohne Schrödingergleichung und Wellenfunktion.

In Heisenbergs Matrizenmechanik, z.B.
http://en.wikipedia.org/wiki/Matrix_mechanics
kommst du ohne explizite Wellenfunktion und Schrödingergleichung aus. Schau dir an, wie du den harmonischen Oszillator ohne explizite Berechnung der Wfkt. mittels Schrödingergleichung rein algebraisch berechnen kannst. Heisenbergs Matrizenmechanik entstand ja noch vor Schrödingers Formulierung, die dann Heisenbergs Matrizenmechanik in der Beliebtheit schnell überholt hatte. Die Energie-Eigenzustände werden in diesem Formalismus als Zustandsvektoren und weniger als Funktionen gesehen.

Meines Wissens benötigst du auch in Feynmans Pfadintegral-Formulierung
http://www.quantumfieldtheory.info/Path_Integrals_in_Quantum_Theories.htm der Quantenmechanik keine Wellenfunktion. Dort werden unmittelbar Übergangswahrscheinlichkeiten bzw. Übergangsamplituden für Prozesse berechnet.

Man kann in beiden Fällen jedoch Äquivalenz zur Schrödingerschen Formulierung nachweisen. Die Wellenfunktion "existiert" also in einer bestimmten Formulierung der Quantenmechanik. Mir scheint, sie ist doch ein sehr vom Modell abhängiges Feature; "da draußen" gibt es sie nicht und da kollabiert sie auch nicht.

Gruß,
Hawkwind
Natürlich ist die 'Welle' nicht in allen Beschreibungen gleich gut sichtbar. Aber in allen brauchbaren Theorien gibt es Elemente, die zur Wellenfunktion äquivalent sind. Wie sonst willst du auf die Interferenzmuster kommen?
Ich sage ja nicht, die Mathematik sei die Realität. Ich sage, sie sei isomorph zur Realität, wenigstens in guter Näherung. Wir können nichts daraus weglassen, ohne es durch etwas Äquivalentes zu ersetzen. Sonst stimmen die Rechnungen nicht mehr. Die Natur kann auch nichts davon weglassen, sonst funktioniert die Welt nicht mehr.

RoKo
19.04.12, 00:54
Hallo Hawkwind,

..Keine Ahnung, ich bin kein Experte; aber meines Wissens gibt es Formulierungen der Quantentheorie ohne Schrödingergleichung und Wellenfunktion...
ja, ja .. und es gibt auch noch die Bohmsche Mechanik
Man kann in beiden Fällen jedoch Äquivalenz zur Schrödingerschen Formulierung nachweisen. .. was auf jeden für die BM gilt. Für die Heisenbergsche Matritzenvorstellung wäre anzumerken, dass mir keine ausgearbeitete QM-Theorie auf ihrer Basis bekannt ist. Gleiches gilt für die Feynmann-Diagramme.Die Wellenfunktion "existiert" also in einer bestimmten Formulierung der Quantenmechanik. Mir scheint, sie ist doch ein sehr vom Modell abhängiges Feature;..noch sind unsere Modelle ja auch noch vorläufige Modelle; die QM ist ja wohl noch nicht als abgeschlossenes Kapitel der Physik zu betrachten.
"da draußen" gibt es sie nicht und da kollabiert sie auch nicht.
Darauf hat Philipp Wehrli schon wunderschön geantwortet. Meine Frage an dich wäre dann, was gib es denn "da draußen"?

RoKo
19.04.12, 03:33
Hallo zusammen,

der "Kollaps" ist selbstverständlich ein im Rahmen der QM erklärbarer Prozess. Es handelt sich um eine verwickelte Kombination aus Verschränkung, dem Austausch von Wirkungsquanten und dem Zerfall des verschränkten Quantenobjektes in unabhängige Objekte. Die Lokalisierung, die i.d.R. dabei erfolgt, ist ein Prozess der Verstärkung und Dämpfung von Schwingungskomponenten. Die SRT ist davon überhaupt nicht berührt, weil ein Quantenobjekt kein klassisches Feld von gekoppelten Raumpunkten ist.

Hawkwind
19.04.12, 09:26
Hallo Hawkwind,

ja, ja .. und es gibt auch noch die Bohmsche Mechanik.. was auf jeden für die BM gilt. Für die Heisenbergsche Matritzenvorstellung wäre anzumerken, dass mir keine ausgearbeitete QM-Theorie auf ihrer Basis bekannt ist. Gleiches gilt für die Feynmann-Diagramme...noch sind unsere Modelle ja auch noch vorläufige Modelle; die QM ist ja wohl noch nicht als abgeschlossenes Kapitel der Physik zu betrachten. Darauf hat Philipp Wehrli schon wunderschön geantwortet. Meine Frage an dich wäre dann, was gib es denn "da draußen"?

z.B. Atome mit diskreten Energieniveaus.

Feynmans Pfadintegrale sind übrigens nicht mit Feynmandiagrammen zu verwechseln: das Pfadintegral ist eine alternative und ausgereifte Beschreibung der Quantentheorie, die auch in Standardvorlesungen ageboten wird. Sie wird heutzutage bei Ansätzen zur Entwicklung einer Theorie der Quantengravitation der herkömmlichen Quantisierung i.d.R. vorgezogen. Feynmandiagramme dagegen sind "bloß" ein "Tool" der "herkömmlichen" relativistischen Quantenfeldtheorien (relativistische Wellengleichungen, "2. Quantisierung", etc.).

Ich denke, ähnliches kann man zu Heisenbergs Matrizenmechanik sagen. Ich hatte diesen Ansatz bei der quantenmchanischen Lösung des harmonischen Oszillators als Alternative zur Lösung der Schrödingergleichung kennengelernt. Fundamentaler Ausgangspunkt sind die Vertauschungsrelationen konjugierter Observabler; eine Möglichkeit, diese zu implementieren ist es, du wählst für die Observablen Differentialoperatoren, die auf Funktionen wirken ("Schrödinger"). Eine Alternative ist, du stellst die Observablen dar durch Matrizen, die auf Vektoren wirken ("Heisenberg").

Letztlich ist der Weg über die Lösung von Differentialgleichungen aber (von der klassischen Mechanik her) einfach der vertrautere und erschien mir persönlich auch systematischer. Die Matrizenmechanik ist vielleicht aus diesem Grund eher von physik-historischem Interesse.
Gruß,
Hawkwind

Hawkwind
19.04.12, 09:28
Hallo zusammen,

der "Kollaps" ist selbstverständlich ein im Rahmen der QM erklärbarer Prozess.

Das ist er selbstverständlich nicht; er ist pure Interpretation.

Hawkwind
19.04.12, 09:44
Natürlich ist die 'Welle' nicht in allen Beschreibungen gleich gut sichtbar. Aber in allen brauchbaren Theorien gibt es Elemente, die zur Wellenfunktion äquivalent sind. Wie sonst willst du auf die Interferenzmuster kommen?
Ich sage ja nicht, die Mathematik sei die Realität. Ich sage, sie sei isomorph zur Realität, wenigstens in guter Näherung. Wir können nichts daraus weglassen, ohne es durch etwas Äquivalentes zu ersetzen. Sonst stimmen die Rechnungen nicht mehr.


Bis hierhin 100% d'accor.


Die Natur kann auch nichts davon weglassen, sonst funktioniert die Welt nicht mehr.

Man sollte immer Natur und Modell auseinanderhalten.

RoKo
20.04.12, 16:47
Hallo Hawkwind,
hat etwas länger gedauert, deine Links zu studieren. Ein bischen Gedächnisauffrischung tut immer gut.
In Heisenbergs Matrizenmechanik, z.B.
http://en.wikipedia.org/wiki/Matrix_mechanics
kommst du ohne explizite Wellenfunktion und Schrödingergleichung aus. Schau dir an, wie du den harmonischen Oszillator ohne explizite Berechnung der Wfkt. mittels Schrödingergleichung rein algebraisch berechnen kannst. Heisenbergs Matrizenmechanik entstand ja noch vor Schrödingers Formulierung, die dann Heisenbergs Matrizenmechanik in der Beliebtheit schnell überholt hatte. Die Energie-Eigenzustände werden in diesem Formalismus als Zustandsvektoren und weniger als Funktionen gesehen.
Wobei man natürlich zunächst feststellen muss, dass sich diese Matritzen auf stehende Wellen beziehen - also auf die atomgebundenen Elektronen. Beim freien Quantenobjekt kommt auch Heisenberg nicht ohne Wellenfunktion(en) aus. Bei ihm sind sie allerdings bereits mit den Operatoren behaftet - ihr gemeinsamer Kern ist dann mit der Schrödinger-Lösung identisch. Der Nachteil der Heisenberg-darstellung ist eben, dass im Grunde zwei Modelle für das Gleiche benutzt werden müssen. Immer ein Zeichen, dass da noch etwas nicht ganz stimmig ist.)
Meines Wissens benötigst du auch in Feynmans Pfadintegral-Formulierung
http://www.quantumfieldtheory.info/Path_Integrals_in_Quantum_Theories.htm der Quantenmechanik keine Wellenfunktion. Dort werden unmittelbar Übergangswahrscheinlichkeiten bzw. Übergangsamplituden für Prozesse berechnet.Feynmann beschäftigt sich von vornherein nur mit freien Quantenobjekten. Der Vorteil seiner Darstellung ist, dass sie sich im Ortsraum abspielt. Ferner wird bei ihm erst so richtig deutlich, dass ein Quantenobjekt steht eine Veranstaltung des ganzen Universums ist. - Bei ihm kommt die Wellenfunktion im übrigen als Superposition aller möglichen Dirac-Impulse oder Gauskurven heraus.
Man kann in beiden Fällen jedoch Äquivalenz zur Schrödingerschen Formulierung nachweisen. Die Wellenfunktion "existiert" also in einer bestimmten Formulierung der Quantenmechanik. Allen Darstellungen ist gemeinsam, dass sie von Quantenobjekten handeln, die sich stets in einem Zustand befinden, der sich bis zur nächsten lokalen Wechselwirkung entwickelt und sich dann grundlegend verändert ("Kollaps" bzw. Zustandsreduktion) - und all das ist eben real existent. Und allen Darstellungen ist gemeinsam, dass sich der Zustand real entwickelt und nicht unerklärt zwischen zwei Messungungen springt. Einsteins Bett ist auch dann in seinem Zimmer, wenn er selbst nicht da ist. Darum gehts - aber nicht nur. Mindestens eines sollte klar sein: Es gibt in der Realität weder punktförmiges noch instantane Sprünge.

Hawkwind
21.04.12, 07:26
Wobei man natürlich zunächst feststellen muss, dass sich diese Matritzen auf stehende Wellen beziehen - also auf die atomgebundenen Elektronen.


Nein, so speziell ist Heisenbergs Matrizenmechanik nicht.


Feynmann beschäftigt sich von vornherein nur mit freien Quantenobjekten.


Das stimmt auch nicht. Vielleicht verwechselst du das wieder mit Feynmandiagrammen; dort werden die wechselwirkenden Teilchen vor und nach der WW als (annähernd) frei betrachtet.

Zum Messproblem in Feynmans Pfadintegral findet man anscheinend verhältnismäßig wenig in der Literatur. In folgendem Artikel gibt es eine Diskussion des quantenmechanischen Messproblems und mögliche Zusammenhänge mit der Gravitation.

QUANTUM MEASUREMENT PROBLEM AND THE POSSIBLE ROLE OF THE GRAVITATIONAL FIELD (http://arxiv.org/pdf/gr--qc/9808033.pdf)

Der Autor vermutet, dass es in der Pfadintegralbeschreibung der QM überhaupt kein Messproblem gibt:


In the Feynman path integral formalism of quantum theory, the measurement problem does not seem to occur, at least not explicitly. Recently, Kaiser and Stodolsky( 21 ) have claimed that the measurement problem does not arise in the Feynman path integral approach. ...


Ich fürchte, dass niemand von uns - selbst du nicht :) - in der Lage ist, diese Thematik hier halbwegs seriös zu diskutieren.

Gruß,
Hawkwind

RoKo
21.04.12, 10:29
Hallo Hawkwind,

danke für den Hinweis. Das kommt an einem regnerischen Wochenende genau richtig. Sicherlich hast du recht:
Ich fürchte, dass niemand von uns - selbst du nicht :) - in der Lage ist, diese Thematik hier halbwegs seriös zu diskutieren.

mir reicht für's erste die Feststellung:

o.a.A: "Several other physicists have also argued that the phenomenon of state vector reduction is an objective, real process, and not just a change in the state of knowledge of the observer"
, dass ich mit meiner realistischen Sichtweise nicht allein bin.

Philipp Wehrli
21.04.12, 15:41
Man sollte immer Natur und Modell auseinanderhalten.

Das tue ich doch auch. Aber wenn kein Modell möglich ist, das ohne ein bestimmtes Element auskommt, dann kommt auch die Natur nicht ohne ein Äquivalent zu diesem Element aus. Wenn gewisse Information nötig ist, um ein Resultat auszurechnen, dann braucht auch die Natur diese Information. Mathematik ist nicht Realität. Aber die Realität kann auch nicht gegen die Mathematik verstossen.

RoKo
21.04.12, 19:02
Hallo Philipp,

Das tue ich doch auch. Aber wenn kein Modell möglich ist, das ohne ein bestimmtes Element auskommt, dann kommt auch die Natur nicht ohne ein Äquivalent zu diesem Element aus. Wenn gewisse Information nötig ist, um ein Resultat auszurechnen, dann braucht auch die Natur diese Information. Mathematik ist nicht Realität. Aber die Realität kann auch nicht gegen die Mathematik verstossen.Sehr unglücklich formuliert; besser wäre:
Wenn unser Modell stimmt, dann verhält es sich genau so wie die Natur. Wenn unser Modell ohne ein bestimmtes Element nicht auskommt, ohne sein Verhalten zu ändern, dann würde auch die Natur ohne das Äquivalent zu diesem Element sich anders verhalten, als wir es beobachten.

--

Im Falle der QM (also unserem Modell) beschreibt die Wellenfunktion die zeitliche Veränderung des energetischen Zustandes eines Quantenobjektes. Die realistische Behauptung ist nun, dass sich auch der Teilbereich der Natur, den wir als Quantenobjekt ansehen, äquivalent verhält. Dem wird entgegengehalten, daß das nicht behauptet werden kann, weil der Zustand wegen seiner Komplexwertigkeit (in unserem Modell) keine Messgröße ist (und wir bei einem nicht atomgebundenen auch tatsächlich über kein Messverfahren verfügen). Bei diesem Argument wird jedoch übersehen, dass wir im Falle eines atomgebundenen Elektrons über die Emissionsspektren sehr wohl etwas über den enegetischen Zustand in Erfahrung bringen können. Wir können deshalb beruhigt davon ausgehen, dass die Wellenfunktion ein äquivalentes, tatsächliches Naturverhalten beschreibt.

Sofern Heisenberg-Matritzen und Pfadintegrale tatsächlich zur Wellefunktion äquivalent sind beschreiben sie das gleiche reale Naturverhalten.

pflanzenfreak
22.04.12, 00:52
"...Messung" ist also keinesfalls an menschliche Beobachter gebunden. Darüber hinaus ist, anders als in allen anderen Bereichen, eine quantenmechanische Messung keine Feststellung von "an sich" vorhandenen Eigenschaften, sondern es ist ihre Herstellung.....

Ein Teilchen nimmt einen bestiimten Zustand ein, wen es eine Information dazu zwingt ( Kolaps der Wellenfunktion).

Eine Messung ist eine Feststellung der registrierbaren Umstande oder auch Informationen (Messkurve).

Woher weiß das erstere vom zweiten?????

vg:confused:

RoKo
22.04.12, 02:43
Hallo planzenfreak,
Ein Teilchen nimmt einen bestiimten Zustand ein, wen es eine Information dazu zwingt ( Kolaps der Wellenfunktion).
Ein Teilbereich der Natur nimmt einen bestimmten Zustand ein, wenn es die eigenen und die äusseren Umstände (wie z.B. Wechselwirkungen) dazu veranlasst.
Eine Messung ist eine Feststellung der registrierbaren Umstande oder auch Informationen (Messkurve).
..durch informationsverarbeitende Wesen. Dies ist zugleich stets eine Wechselwirkung mit der Natur durch den Messapparat, der im Bereich der Quanten nicht mehr vernachlässigt werden kann.
Woher weiß das erstere vom zweiten?????Teilbereiche der Natur, die keine informationsverarbeitenden Wesen sind, wissen nichts.vg:confused:Das glaub' ich dir gern.

amc
22.04.12, 09:25
Moin RoKo,

Teilbereiche der Natur, die keine informationsverarbeitenden Wesen sind, wissen nichts.

wie definierst du "informationsverarbeitende Wesen"?

Grüße, AMC

Bauhof
22.04.12, 09:56
Hallo Philipp, Sehr unglücklich formuliert; besser wäre: Wenn unser Modell stimmt, dann verhält es sich genau so wie die Natur.
Hallo RoKo,

ebenso unglücklich formuliert.
Wir können nie wissen, ob unser Modell stimmt, daher können wir auch nie wissen ob es sich so verhält wie die Natur sich verhält. Die wahre 'Natur' der Natur werden wir niemals erfassen können, mit keinem Modell. Falls es so etwas überhaupt gibt wie die wahre 'Natur' der Natur.

M.f.G. Eugen Bauhof

RoKo
22.04.12, 09:59
Hallo amc,

wie definierst du "informationsverarbeitende Wesen"?
Einen guten Überblick findest du hier (http://www.inf.uni-konstanz.de/Veranstaltungen/ring/VortragHuebner.PDF).

RoKo
22.04.12, 10:46
Hallo Bauhof,
Wir können nie wissen, ob unser Modell stimmt, daher können wir auch nie wissen ob es sich so verhält wie die Natur sich verhält.
Wir müssen es auch nicht 100% wissen. Die Komplexität der Natur begrenzt ohnehin die Vorhersagekraft unserer Modelle. Andererseits kann das Wissen, was uns als Basis zur Produktion massenhaft funktionierender Technik dient, so ganz falsch nicht sein. Da ich weiss, wie Computer und Handys funktioneren, weiss ich ich auch, dass die aus der Elektrodynamik und der Quantnmechanik abgeleiteten Modelle von Teilbereichen der Natur funktionieren. Die wahre 'Natur' der Natur werden wir niemals erfassen können, mit keinem Modell. Falls es so etwas überhaupt gibt wie die wahre 'Natur' der Natur. Was soll die wahre 'Natur' der Natur sein? Von soetwas würde ich nicht sprechen. Es geht darum, dass Verhalten von Natur vorausschauend zu erkennen.von Niels Bohr:
Es gibt keine Quantenwelt. Es gibt nur eine abstrakte quantenphysikalische Beschreibung. Wenn es keine Quantenwelt gäbe, wäre so ziemlich Alles ein Wunder, einschliesslich der Existenz abstrakter quantenphysikalischer Beschreibungen.

Falls Niels Bohr tatsächlich so gedacht haben sollte, so hat er jedenfalls in diesem Punkt nicht nachgedacht.

Marco Polo
22.04.12, 12:17
Wenn es keine Quantenwelt gäbe, wäre so ziemlich Alles ein Wunder, einschliesslich der Existenz abstrakter quantenphysikalischer Beschreibungen.

Hallo Roko,

dein Hang zum "Realismus" ist zwar nachvollziehbar. Weiterbringen wird er dich aber nicht.

Natürlich kann man z.B. beim Doppelspaltversuch das Interferenzmuster mit Wellengleichungen beschreiben. Es bleibt aber eine rein mathematische Beschreibung.

Nimm von mir aus das Elektron. Ist es ein Teilchen oder eine Welle oder keins von Beiden? Die Tatsache, dass es sich abhängig vom Experiment mal so oder so verhält verrät, das es eigentlich keins von Beiden ist.

Die extrem gut funktionierende mathematische Beschreibung dieses Sachverhaltes bringt uns aber bezüglich der Frage nach der "wahren Natur" von Elektronen nicht weiter.

Deswegen ja auch die korrekte Bemerkung von Bohr, dass es keine Quantenwelt gibt. Zumindest nicht so, wie der "Realist" sich das gerne vorstellt.

Weiteres Beispiel: Die Polarisation verschränkter Photonen.

Diese ist völlig unbestimmt, solange man sie nicht misst. Dabei spielt es auch keine Rolle, ob man das ganze lokal oder nichtlokal betrachtet.

Die realistische Sicht auf die Quantenwelt, die du favorisierst, versagt hier leider völlig.

Gruss, Marco Polo

Bauhof
22.04.12, 14:14
Niels Bohr: Es gibt keine Quantenwelt. Es gibt nur eine
abstrakte quantenphysikalische Beschreibung.
Falls Niels Bohr tatsächlich so gedacht haben sollte, so hat er jedenfalls in diesem Punkt nicht nachgedacht.
Hallo RoKo,

das ist m.E. die kürzestmögliche Darstellung der Kopenhagener Deutung: Es gibt keine Quantenwelt, es gibt nur eine abstrakte quantenphysikalische Beschreibung - hinter den Erscheinungen gibt es keine zusätzliche verborgene Realität.

Es steht dir natürlich frei, der Kopenhagener Deutung nicht zu folgen. Aber es ist eine ziemliche Anmaßung, Niels Bohr zu attestieren, dass er nicht genügend nachgedacht habe. Niels Bohr hat im Urteil seiner Zeitgenossen sehr tief nachgedacht. Jedenfalls mehr als die heutigen Amateur-Philosophen, solche Leute wie du und ich.

M.f.G. Eugen Bauhof

EMI
22.04.12, 15:20
Sind die Interpretationen nicht letztlich ein Versuch, die vertrauten Paradigmen der klassischen Physik auch der Quantentheorie "aufzudrängen"?Nö Hawkwind,

da drängt niemand was auf.

Zu versuchen etwas klassisch zu erklären ist die einzige Möglichkeit es letztendlich zu verstehen.

Die oft formulierte Aussage, dass die klassische Mechanik (KM) "Grenzfall der QM" sei, meint nichts anderes als das die KM in der QM modellierbar ist.
Nach dem Korrespondenzprinzip ist es auch umgekehrt möglich das Teile der Aussagen der QM in der KM modellierbar sind.
Durch diese Modellierung werden die Aussagen aus der QM in der Sprache der KM interpretierbar.
Damit bekommt die QM überhaupt erst eine physikalische Bedeutung. Etwas verstanden haben heisst: das Gemeinte im Rahmen der KM darstellen zu können.
Die Aussagen der KM sind tatsächlich die synthetischen Urteile a priori.

Die KM ist die Grundlage aller theoretischen Physik.

Die QM ist eine viel umfangreichere Theorie als die KM, ihre Inhalte sind naturgemäß nicht alle vollständig klassisch modellierbar.
Es gibt also keine vollständigen mechanischen Modelle für die Quantenprozesse der QM.
Diese Nichtmodellierbarkeit schließt aber ein, dass es in der QM sinnvolle Aussagen gibt, denen keine Bedeutung in der KM zukommt.

Quantenbegriffe bedeuten nur etwas, wenn sich deren Begriffsumfang einem Begriffs der KM zuordnen lässt.
Durch den größeren Inhaltsreichtum der QM gibt es eine Menge von Aussagen und Begriffen in der QM, die mit keinem aus der KM korrespondieren.
Sie besitzen keine Interpretation in der KM. Das ist das größte Problem im Verständnis der Quantenphysik.


Die Physik muss nach EINSTEIN genau die wirkliche Welt abbilden und darf darüber hinaus nichts über Dinge aussagen, die es in der Realität nicht gibt.
Die QM enthält aber nach EINSTEIN auch solche unphysikalische Aussagen über irreale Prozesse und gestattet mathematisch neben den physikalischen auch eine Vielzahl von physikalisch nicht verständlichen Strukturen.

EINSTEIN zumindest gelangte zum Ergebnis, dass es noch umfassendere und strengere Naturgesetze geben muss als die QM.

Die QM und die ART bestimmen zwar die Bewegungsgesetze der Teilchen im Mikro- und Makrokosmus und die Ausbreitung der Felder in Raum und Zeit, sind aber nicht in der Lage, vorauszusagen, was für Teilchen und Felder überhaupt existieren können. Sie sind nicht in der Lage, zu erklären, warum Atome so klein und das Universum so groß ist.
EINSTEIN hatte immer wieder auf dieses Problem hingewiesen, obwohl dieses nunmehr durch die gewaltigen Erfolge und ungeahnten Anwendungsmöglichkeiten der derzeitigen Physik in den Hintergrund trat und dabei wohl in Vergessenheit geraten ist.

Die zukünftige Physik muss eine umfassende Antwort auf EINSTEIN's Problemstellung geben.

EINSTEIN bemerkte bereits 1909, dass das el.Elementarquantum e in der MAXWELLschen Elektrodynamik ein Fremdling ist und PAULI merkte 1949 an, das e auch in der QM ein Fremdling geblieben ist und hob hervor, das diese Tatsache für EINSTEIN einer der stärksten Gründe war, an der Entgültigkeit der Schritte zu zweifeln, die zur QM geführt hatten.

Schon EINSTEIN und HEISENBERG wiesen darauf hin, dass zwischen den großen Theorien (QM, ART) in deren letzter Konsequenz Widersprüche auftreten.
Die damit verbundenen Singularitäten lassen sich zwar mit Renormierungstechniken "umschiffen", sind aber logisch innerhalb der QM und der ART nicht auflösbar.
Es sind Zusatzhyphotesen.


Die Annahme, es gäbe verschiedene physikalische Weltbilder, das mechanische, das elektromagnetische, das quantenmechanische, die man sozusagen zur Auswahl hätte ist falsch und widerspricht der Einheit der Physik.
Die KM beschäftigt sich mit Raum und Zeit und der Bewegung von Substanzen in Raum und Zeit, etwas anderes kann die Elektrodynamik und die QM auch nicht tun.
So sah es einst ein MAX PLANCK und EMI sogar heute noch.

Gruß EMI

RoKo
22.04.12, 15:28
Hallo Marco Polo,


dein Hang zum "Realismus" ist zwar nachvollziehbar. Weiterbringen wird er dich aber nicht.Ich weiss nicht, was du dir so unter Realismus vorstellst. Ich gehe jedenfalls davon aus, dass eine objektive Realität gibt, die ausserhalb und unabhängig von mir und meinem Denken existiert, weil jede andere Annahme in letzter Konsequenz zu Widersprüchen führt. Mehr Philosophie setze ich nicht voraus. Alles andere überlasse ich der empirischen Wissenschaft.Natürlich kann man z.B. beim Doppelspaltversuch das Interferenzmuster mit Wellengleichungen beschreiben. Es bleibt aber eine rein mathematische Beschreibung.

Nimm von mir aus das Elektron. Ist es ein Teilchen oder eine Welle oder keins von Beiden? Die Tatsache, dass es sich abhängig vom Experiment mal so oder so verhält verrät, das es eigentlich keins von Beiden ist.Von nichts anderem gehe ich aus.Die extrem gut funktionierende mathematische Beschreibung dieses Sachverhaltes bringt uns aber bezüglich der Frage nach der "wahren Natur" von Elektronen nicht weiter.Ich habe ja auch nie von ein einer "wahren Natur" gesprochen.Deswegen ja auch die korrekte Bemerkung von Bohr, dass es keine Quantenwelt gibt. Zumindest nicht so, wie der "Realist" sich das gerne vorstellt.Also entweder - oder. Also entweder gibt es keine Quantenwelt oder es gibt eine Quantenwelt, sie verhält sich nicht so, wie der "Realist" (beachte die Anführungszeichen) sich das vorstellt.

Die Position, dass es keine Quantenwelt gibt, ist haltbar, weil sie in logischer Konsequenz jegliche Existenz verneinen muss. Also bleibt nur, ihre Existenz anzunehmen und ihr Verhalten zu klären. Nichts anderes tue ich. Weiteres Beispiel: Die Polarisation verschränkter Photonen.

Diese ist völlig unbestimmt, solange man sie nicht misst. Dabei spielt es auch keine Rolle, ob man das ganze lokal oder nichtlokal betrachtet.

Die realistische Sicht auf die Quantenwelt, die du favorisierst, versagt hier leider völlig.Lass dir doch einfach mal diesen sprachlichen Unsinn ("Diese ist völlig unbestimmt, solange man sie nicht misst") durch den Kopf gehen. Er widerspricht doch den Erkenntnissen aus der Optik. Licht ist normalerweise nicht polarisiert; es sei denn, man schickt es durch einen Polarisationsfilter. Allein dieser Sprachgebrauch vernebelt nur die Köpfe.

Über das, was passiert, kann man deutlich mehr aussagen.

Zwei verschränkte Photonen entstehen bei bestimmten Atomen, wenn der Übergang eines Elektrons auf ein niedrigeres Energieniveau über ein weiteres mögliches Energieniveau verläuft. Dadurch wird insgesamt eine EM-Welle abgestrahlt, die zwei räumlich getrennte Maxima hat und die geschickt in zwei Lichtleiter eingespeist werden. Durch den Entstehungsprozess bedingt sind die beiden Photonen miteinander verschränkt; d.h. sie befinden sich in einem gemeinsamen energetischen Zustand. Dieser gemeinsame Zustand ist die einzige unahängige Eigenschaft; von daher sind die beiden verschränkten Photonen im Grunde als ein Quantenobjekt zu betrachten.

Wenn nun, egal wo, auf diesen energetischen Zustand durch drittes eingewirkt wird, dann wird dadurch dieser Zustand dadurch verändert, dass zunächst ein neuer, gemeinsamer Zustand mit dem einwirkenden Dritten entsteht und gemäß der Dynamik der Einwirkung verändert wird. Wobei anschliessend zwei neue, jetzt unabhängige Zustände entstehen. Im Falle der Polarisation nun zwei gleich polarisierte Photonen.

Dieser Vorgang kollidiert natürlich mit einem Weltbild, dass auf unabhängigen Punktteilchen beruht. Das ist aber nicht mein Problem, sondern eher das generelle Problem einiger Gründerväter der QM.

Marco Polo
22.04.12, 15:42
Hallo EMI,

Es gibt also keine vollständigen mechanischen Modelle für die Quantenprozesse der QM.

Zu versuchen etwas klassisch zu erklären ist die einzige Möglichkeit es letztendlich zu verstehen.

der zum Scheitern verurteilte Versuch, Quantenprozesse klassisch zu erklären, führt in keinster Weise zum Verständnis dieser bei.

Wie denn auch, wenn es wie du oben schriebst keine vollständigen mechanischen Modelle für Quantenprozesse gibt.

Gruss, Marco Polo

Marco Polo
22.04.12, 15:55
Ich gehe jedenfalls davon aus, dass eine objektive Realität gibt, die ausserhalb und unabhängig von mir und meinem Denken existiert, weil jede andere Annahme in letzter Konsequenz zu Widersprüchen führt.

Ist diese "objektive Realität" nicht genau die "wahre Natur", von der du laut deiner Aussage nie gesprochen hast?

Ich habe ja auch nie von ein einer "wahren Natur" gesprochen.

Oder anders formuliert: Worin siehst du den Unterschied zwischen "objektiver Realität" und "wahrer Natur"?

Offtopic: Was machst du eigentlich in Hong Kong? Hockst du da die ganze Zeit im Internet-Cafe? :D

Grüsse, Marco Polo

RoKo
22.04.12, 16:38
Hallo Marco Polo,

Ist diese "objektive Realität" nicht genau die "wahre Natur", von der du laut deiner Aussage nie gesprochen hast?

Oder anders formuliert: Worin siehst du den Unterschied zwischen "objektiver Realität" und "wahrer Natur"?
"Objektive Realität" hat nur die Eigenschaft, ausserhalb und unabhängig von von mir und meinem Denken zu existieren.

Um begrifflich vergleichbar zu sein:

"Natur" hat aus philosophischer Sicht zunächst nur die Eigenschaft, ausserhalb und unabhängig von mir und meinem Denken zu existieren; alles weitere haben die Naturwissenschaften zu klären.

"Wahre Natur" vermengt nun den Begriff "Natur", deren Existenz anzunehmen ist, mit dem eher erkenntnistheoretischen Begriff "Wahrheit" und erweckt damit die falsche Vorstellung, dass wir wissen können, wie die Natur existiert. Zugänglich sind uns jedoch nur Ereignisse / Veränderungen, aus denen wir Rückschlüsse ziehen können. Überprüfbar ist daher für uns nur, wie sich die Natur verhält.

Ein abendfüllendes Thema..


Offtopic: Was machst du eigentlich in Hong Kong? Hockst du da die ganze Zeit im Internet-Cafe? :D
Offtopic Antwort. Ich arbeite und wohne derzeit hier; seit einem Jahr und für mindestens noch ein Jahr. Ich nehme hier die Automatisierungstechnik eines neuen Cargo-Terminals in Betrieb.

Da ich ohne Anhang hier bin, habe ich abends oft Zeit.

Marco Polo
22.04.12, 16:59
Ein abendfüllendes Thema..

Du sagst es. :)

Ich arbeite und wohne derzeit hier; seit einem Jahr und für mindestens noch ein Jahr. Ich nehme hier die Automatisierungstechnik eines neuen Cargo-Terminals in Betrieb.Das hört sich für mich nach ner Menge Arbeit an.

Da ich ohne Anhang hier bin, habe ich abends oft Zeit.Und die darfst du selbstverständlich nach deinem Gutdünken gestalten.

Ich wünsche dir eine gute Zeit in Hong Kong. Hab mir übrigens gerade noch ein paar selbstaufgenommene Videos vom Quantengrillen bei Jogi reingepfifffen. Du warst auch dabei. :)

Viele Grüsse, Marco Polo

Bauhof
22.04.12, 17:20
Quantenbegriffe bedeuten nur etwas, wenn sich deren Begriffsumfang einem Begriffs der KM zuordnen lässt.
Hallo EMI,

hier z.B. kann ich dir nicht zustimmen.
Es gibt mehrere Quantenbegriffe, die sich nicht dem Begriffsumfang der Klassischen Mechanik zuordnen lassen. Ein Beispiel ist die quantenmechnische Verschränkung (http://www.quantenwelt.de/quantenmechanik/vielteilchen/verschraenkung.html). Diese entzieht sich völlig dem klassischen Verständnis.

M.f.G. Eugen Bauhof

JoAx
23.04.12, 09:39
Hi Eugen!


Es gibt mehrere Quantenbegriffe, die sich nicht dem Begriffsumfang der Klassischen Mechanik zuordnen lassen.


Was müsste man jetzt tun, wenn man wissenschaftlich bleiben will?


Gruß, Johann

Hawkwind
23.04.12, 12:39
Hi EMI,


EINSTEIN bemerkte bereits 1909, dass das el.Elementarquantum e in der MAXWELLschen Elektrodynamik ein Fremdling ist


Ist sie das denn?
Davon ist mir nichts bekannt.



und PAULI merkte 1949 an, das e auch in der QM ein Fremdling geblieben ist


... und davon auch nicht.


Es gibt doch einen unmittelbaren Zusammenhang zwischen einer erhaltenen Ladung (z.B. e) und der Symmetrie unter globalen Eichtransformationen in den Theorien. Was ist denn so problematisch an der Elementarladung?

Bauhof
23.04.12, 12:58
Hi Eugen! Was müsste man jetzt tun, wenn man wissenschaftlich bleiben will? Gruß, Johann
Hallo Johann,

einem Paradigmenwechsel zustimmen und nicht versuchen, die klassische Mechanik in die Quantenmechanik hinüberzuretten.

M.f.G. Eugen Bauhof

JoAx
23.04.12, 13:21
einem Paradigmenwechsel zustimmen


Worin genau besteht der (dieser) Paradigmenwechsel?


Gruß, Johann

Bauhof
23.04.12, 14:16
Worin genau besteht der (dieser) Paradigmenwechsel?

Hallo Johann,

von einem Paradigmenwechsel spricht man dann, wenn sich bei wissenschaftlichen Revolutionen, wie z.B. bei der Quantentheorie die Wahrnehmung der Wissenschaftler radikal ändert.

Eine radikale Änderung in der Wahrnehmung der Wissenschaftler hinsichtlich der Quantentheorie sehe ich z.B. im Wechsel von der deterministischen Auffassung der klassischen Mechanik zur probalistischen Auffassung der Quantenmechanik durch Max Born.

M.f.G. Eugen Bauhof

JoAx
23.04.12, 14:25
Eine radikale Änderung in der Wahrnehmung der Wissenschaftler hinsichtlich der Quantentheorie sehe ich z.B. im Wechsel von der deterministischen Auffassung der klassischen Mechanik zur probalistischen Auffassung der Quantenmechanik durch Max Born.


Und da wiederhole ich meine Frage an dich, die ich etwas früher schon stellte:

Was war in der Arbeit von Max Born, in der er die Wellenfunktion als Wahrscheinlichkeit, dass das Elektron in eine bestimmte Richtung gestreut wird, interpretierte, "nicht-klassisch"?


Gruß, Johann

RoKo
23.04.12, 14:47
Hallo zusammen,

Eine radikale Änderung in der Wahrnehmung der Wissenschaftler hinsichtlich der Quantentheorie sehe ich z.B. im Wechsel von der deterministischen Auffassung der klassischen Mechanik zur probalistischen Auffassung der Quantenmechanik durch Max Born.


Und da wiederhole ich meine Frage an dich, die ich etwas früher schon stellte:

Was war in der Arbeit von Max Born, in der er die Wellenfunktion als Wahrscheinlichkeit, dass das Elektron in eine bestimmte Richtung gestreut wird, interpretierte, "nicht-klassisch"?


Eine wirklich radikale Änderung in der Wahrnehmung der Wissenschaftler wäre es gewesen, sich von unpassenden klassischen Begrifflichkeiten wie Teilchen oder Wellen zu verabschieden. Die QM handelt von den energetischen Zuständen von Quantenobjekten bzw. Systemen und ihren kleinsten Wirkungen auf andere Objekte. Dies drückt eine ihrer fundamentalen Beziehungen so aus:

2pi*v(p*q - q*p) = h*v

Bauhof
23.04.12, 15:21
Was war in der Arbeit von Max Born, in der er die Wellenfunktion als Wahrscheinlichkeit, dass das Elektron in eine bestimmte Richtung gestreut wird, interpretierte, "nicht-klassisch"?
Hallo Johann,

weil man mit Hilfe der Bornschen Wahrscheinlichkeitswelle nicht voraussagen kann, dass das Elektron in diese bestimmte Richtung mit der Wahrscheinlichkeit W=1 gestreut wird, sondern nur mit einer gewissen Wahrscheinlichkeit W<1.

In der klassischen Auffassung wäre W=1 prinzipiell möglich, wenn man alle Startbedingungen exakt kennen würde. In der probalistischen Auffassung ist das prinzipiell unmöglich.

M.f.G. Eugen Bauhof

JoAx
23.04.12, 15:37
Eine wirklich radikale Änderung in der Wahrnehmung der Wissenschaftler wäre es gewesen, sich von unpassenden klassischen Begrifflichkeiten wie Teilchen oder Wellen zu verabschieden.


Das alles klingt wunderbar. Es ist allerdings die Frage zu klären, ob das überhaupt sein muss. Nach der von Timm (und dann Uli) erwähnten Ensemble interpretation (http://en.wikipedia.org/wiki/Ensemble_interpretation) ist das schlicht nicht der Fall, da die "Wellenfunktion" danach nicht ein einzelnes Quantenobjekt oder ein einzelnes Quanten-System beschreibt.

Das tun aber alle anderen Interpretationen. Womit wir wieder (auch) bei der Frage wären, ob man einen Satz wie - "die Wellenfunktion beschreibt nur die Ergebnisse mehrerer Tests" und Zeilinger's interpretatorische Aussagen nebeneinander setzen kann. :)


Gruß, Johann

JoAx
23.04.12, 15:48
weil man mit Hilfe der Bornschen Wahrscheinlichkeitswelle nicht voraussagen kann, dass das Elektron in diese bestimmte Richtung mit der Wahrscheinlichkeit W=1 gestreut wird, sondern nur mit einer gewissen Wahrscheinlichkeit W<1.

In der klassischen Auffassung wäre W=1 prinzipiell möglich, wenn man alle Startbedingungen exakt kennen würde. In der probalistischen Auffassung ist das prinzipiell unmöglich.


Wenn man die Anfangsbedingungen prinzipiell nicht exakt kennen kann, was nach der Unschärferelation definitiv der Fall ist, dann kann man auch mit "klassischem Verständniss" der Wellenfunktion nie W=1 bekommen. Auch wenn es in der Tat eine solche bestimmte Richtung gäbe. Das kann aber nicht der Paradigmenwechsel sein. Oder?


Gruß

RoKo
23.04.12, 16:53
Hallo JoAx,

Das alles klingt wunderbar. Es ist allerdings die Frage zu klären, ob das überhaupt sein muss. Nach der von Timm (und dann Uli) erwähnten Ensemble interpretation (http://en.wikipedia.org/wiki/Ensemble_interpretation) ist das schlicht nicht der Fall, da die "Wellenfunktion" danach nicht ein Quantenobjekt oder ein Quanten-System beschreibt.Da wir ja mit einzelnen Quantenobjekten experimentieren können, benötigen wir eine Theorie für den Einzelfall. Die Ensemble Interpretation kann höchsten Streit vermeiden:) , aber nicht befriedigen.Das tun aber alle anderen Interpretationen. Womit wir wieder (auch) bei der Frage wären, ob man einen Satz wie - "die Wellenfunktion beschreibt nur die Ergebnisse mehrerer Tests" und Zeilinger's interpretatorische Aussagen nebeneinander setzen kann. :)Da man außer in der Mathematik und in der Logik nichts streng beweisen kann, könnte man es nebeneinander stehen lassen, wären "schwammige" Aussagen nicht der Türöffner für höchst unwissenschaftliche Spekulationen.

Ich bestehe daher mit einer gewissen Penetranz darauf, dass QM nicht nur eine mathetische Spielerei zur Vorhersage von Messergebnissen ist, sondern uns etwas über das reale Verhalten einer objektiv von unseren Erkenntnissen und Rechenkünsten unabhängig existierenden Natur zu sagen hat.

Quantenobjekt / system = menschliche Bezeichnung für einen Teilbereich der Natur, der bereits vor der Entstehung der Menschheit existiert hat und auch nach ihrem Untergang weiterexistieren wird.

Energetischer Zustand = menschliche Bezeichnung für in o.a. Teilbereich tatsächlich vorhandene Verhältnisse.

Wellenfunktion oder Pfadintegrale oder Matrizen usw. = von Menschen ausgedachte mathematische Tools zur Berechnung der Veränderung von o.a. Zuständen, die in o.a. Teilbereichen tatsächlich stattfinden.

Mehr "Realismus" setze ich nicht voraus - aber diese Voraussetzung muß sein, will man nicht alles vernebeln.

Hawkwind
23.04.12, 16:57
Wenn man die Anfangsbedingungen prinzipiell nicht exakt kennen kann, was nach der Unschärferelation definitiv der Fall ist, dann kann man auch mit "klassischem Verständniss" der Wellenfunktion nie W=1 bekommen. Auch wenn es in der Tat eine solche bestimmte Richtung gäbe. Das kann aber nicht der Paradigmenwechsel sein. Oder?


Gruß

Auch W=1 ist übrigens möglich. Das ist immer dann der Fall, wenn sich das Quantensystem bereits in einem sog. Eigenzustand zur gemessenen Observablen befindet ... z.B. ein präparierter Zustand ist. In so einem Fall aber könnte man andererseits für eine Messung der entsprechenden konjugierten Observablen überhaupt keine Vorhersage mehr machen (maximale Unschärfe, alles gleich (un)wahrscheinlich).

JoAx
23.04.12, 17:08
Da wir ja mit einzelnen Quantenobjekten experimentieren können, benötigen wir eine Theorie für den Einzelfall.

Exakt! Liefert die QM das denn auch? Das ist die Frage, die sich bsw. Einstein stellte, und wohl mit "Nein" beantwortete.


Die Ensemble Interpretation kann höchsten Streit vermeiden


Nein. Sie bezieht eine klare Stellung: Die QM ist keine Theorie über einzelne Quanten-Objekte/-Systeme. In dem Sinne ist sie (imho) in der Opposition zu allen anderen - KD, VWI, BM. Wie kann das dem Vermeiden des "Streites" dienen? :)


Ich bestehe daher mit einer gewissen Penetranz darauf, dass QM nicht nur eine mathetische Spielerei zur Vorhersage von Messergebnissen ist, sondern uns etwas über das reale Verhalten einer objektiv von unseren Erkenntnissen und Rechenkünsten unabhängig existierenden Natur zu sagen hat.
...


Das geht aber an der Fragestellung vorbei. Natürlich würde die QM imho auch nach der EI (ist klar was ich meine?) etwas über die "objektive Realität" aussagen, nur nicht im Sinne, dass die Wellenfunktion eine Beschreibung eines einzelnen Quantums liefert.


Gruß, Johann

RoKo
23.04.12, 17:34
Hallo JoAx,

.. Natürlich würde die QM imho auch nach der EI (ist klar was ich meine?) etwas über die "objektive Realität" aussagen, nur nicht im Sinne, dass die Wellenfunktion eine Beschreibung eines einzelnen Quantums liefert.

.. was könnte eine solche Interpretation an Erkenntnissen über Atomaufbau, chemische Affinitäten, Leitfähigkeit, Kristallgitterstrukturen usw. usw. beitragen und wie konsistent wäre das?

JoAx
23.04.12, 17:38
.. was könnte eine solche Interpretation an Erkenntnissen über Atomaufbau, chemische Affinitäten, Leitfähigkeit, Kristallgitterstrukturen usw. usw. beitragen und wie konsistent wäre das?

Warum fragst du so? :)

Bauhof
23.04.12, 18:11
Wenn man die Anfangsbedingungen prinzipiell nicht exakt kennen kann, was nach der Unschärferelation definitiv der Fall ist, dann kann man auch mit "klassischem Verständniss" der Wellenfunktion nie W=1 bekommen. Auch wenn es in der Tat eine solche bestimmte Richtung gäbe. Das kann aber nicht der Paradigmenwechsel sein. Oder?
Hallo Johann,

nach klassischem Verständnis kann man nichts anderes bekommen als W=1. Warum? Weil man nach klassischem Verständnis die Heisenbergsche Unbestimmtheitsrelation gar nicht kannte und daher dachte, dass immer jedes Ereignis mit W=1 vorauszusagen wäre.

Der Unterschied ist der, dass man nach klassischem Verständnis zwar auch nicht alle Anfangsbedingungen kennen konnte, aber man postulierte beim klassischen Verständnis, dass das prinzipiell möglich sei, aber nur an den unzureichenden Messmethoden scheitern würde.

M.f.G Eugen Bauhof

RoKo
23.04.12, 18:34
Hallo Bauhof,

.. Weil man nach klassischem Verständnis die Heisenbergsche Unbestimmtheitsrelation gar nicht kannte und daher dachte, dass immer jedes Ereignis mit W=1 vorauszusagen wäre.....und warum gab es lange vor der sogenannten Heisenbergschen Unbestimmtheitsrelation statistische Mechanik? Was besagt das "Gesetz der großen Zahl"? Und wo ist da der Paradigmenwechsel, wo doch nur die Variablen der klassischen Mechanik unbestimmt sind?

Fragen über Fragen!

JoAx
23.04.12, 19:18
Der Unterschied ist der, dass man nach klassischem Verständnis zwar auch nicht alle Anfangsbedingungen kennen konnte, aber man postulierte beim klassischen Verständnis, dass das prinzipiell möglich sei, aber nur an den unzureichenden Messmethoden scheitern würde.


In Ordnung, Eugen!

Ich darf ein Gedankenspiel formulieren, um zu testen, ob ich dich auch richtig verstanden habe:

Wir nehmen ein ganz normales, klassisches Billard. Große, exakt zu vermessende Kugeln, exakt definierte Orte und Impulse. Deine Sicht der QM kann man dann so beschreiben, dass trotz der exakten Anfangs- und Nebenbedingungen, die Bewegungsrichtungen zweier Billardbälle nach der Kollision zufällig sind. (Bei Erhaltung des Gesamtimpulses.)

Ist das korrekt?


Gruß, Johann

Hawkwind
23.04.12, 20:44
Hallo Bauhof,

..und warum gab es lange vor der sogenannten Heisenbergschen Unbestimmtheitsrelation statistische Mechanik? Was besagt das "Gesetz der großen Zahl"? Und wo ist da der Paradigmenwechsel, wo doch nur die Variablen der klassischen Mechanik unbestimmt sind?

Fragen über Fragen!

Das eine hat mit dem anderen nichts zu tun.

Die statistische Mechanik beschäftigt sich mit der Physik von Systemen, die aus immens vielen Komponenten besteht - so viele, dass man kein Interesse daran hat, die Bewegung jeder einzelnen Komponente zu diskutieren. Man interessiert sich eher für Mittelwerte oder andere Größen wie Temperatur, Druck etc. die das Gesamtsystem besser charakterisieren. Dabei kommen Methoden aus der Statistik zum Einsatz. Statistische Mechanik kann auf klassischer aber auch auf Quantenmechanik basieren. Im letzteren Fall spricht man dann manchmal von Quantenstatistik.

Die statistischen Eigenschaften der Quantenphysik dagegen sind ganz anders; sie sind prinzipieller Natur. Es ist hier nicht so, dass wir kein Interesse daran haben, etwas über die Physik des einzelnen Systems zu sagen. Wir können es einfach nicht mit Bestimmtheit.

richy
24.04.12, 00:30
hi
Ich freue mich, dass Bauhof Klartext bezueglich der Kopenhagener Interpretation schreibt. So haette das immer sein sollen. Ob man diesen Hypothesen dann zustimmt ist eine ganz andere Sache. Die Annahmen muessen zunaechst formuliert werden wie es Bauhof gerade zeigt.
@Bauhof Mich stoert lediglich, dass du teilweise den Eindruck vermittelst es gaebe keine realistische Deutungsmoeglichkeit. Die gibt es natuerlich. Lediglich der Einsteinsche Realismus, Bertelmanns Socken sind mit Bell widerlegt.

Zu Heisenberg. Da sollte man sich zunaechst mit der Unschaerfe der Nachrichtentechnik nochmals beschaeftigen. Zum Beispiel Musik. Um zu sehen dass das Prinzip rein mathematischer Natur ist und zum Beispiel bezueglich der direkten Variablenpaare Frequenz, Periodendauer nichts besonderes darstellt. Das besondere liegt in der Qm in der Tatsache, dass Auftrittswahrscheinlichkeiten solcher Paare ueber die FT verknuepft sind. Es sind somit zwei paar Stiefel. Und eine nochmalige Besonderheit nimmt die Kopenhagener Deutung an. Dass die Auswahl, der Zufall objektiv sei. Das ist so unvorstellbar, dass manche dazu keinerlei Zugang haben und es als nichts besonderes betrachten. Ein Phaenomen dass man oefters bei Vertretern der KD antrifft. Ebenso wie das geraede kontraere Phanomen dazu :
Denn Nichts besonderes bietet tatsaechlich die Ensembleinterpretation. Diese spricht einfach die Egodenhypothese an. Das erklaert nichts neues und der Minimalismus der Ensembleinterpretation kennt ausser diesem Nichterklaeren nichts das von Interesse waere. Wann sollte ich mich wundern ? Wann wird Triviales als besondere Erkenntnisse betrachtet/verkauft ? Warum stellen sich solche Fragen bei realistischen Interpretationen nicht ? Gruesse

EMI
24.04.12, 01:02
Doch, es gibt einen Wurm. Sogar mehrere. Wart ab.Ich warte noch immer auf die "Würmer" fossi.;)

Gruß EMI

Bauhof
24.04.12, 09:34
In Ordnung, Eugen! Ich darf ein Gedankenspiel formulieren, um zu testen, ob ich dich auch richtig verstanden habe: Wir nehmen ein ganz normales, klassisches Billard. Große, exakt zu vermessende Kugeln, exakt definierte Orte und Impulse. Deine Sicht der QM kann man dann so beschreiben, dass trotz der exakten Anfangs- und Nebenbedingungen, die Bewegungsrichtungen zweier Billardbälle nach der Kollision zufällig sind. (Bei Erhaltung des Gesamtimpulses.)
Ist das korrekt?
Hallo Johann,

das ist nicht korrekt.
Dein Gedankenspiel hat Hawkwind bereits beantwortet:
Die statistischen Eigenschaften der Quantenphysik dagegen sind ganz anders; sie sind prinzipieller Natur. Es ist hier nicht so, dass wir kein Interesse daran haben, etwas über die Physik des einzelnen Systems zu sagen. Wir können es einfach nicht mit Bestimmtheit. (http://www.quanten.de/forum/showpost.php5?p=67957&postcount=141)

M.f.G Eugen Bauhof

Bauhof
24.04.12, 13:48
Wir nehmen ein ganz normales, klassisches Billard. Große, exakt zu vermessende Kugeln, exakt definierte Orte und Impulse. Deine Sicht der QM kann man dann so beschreiben, dass trotz der exakten Anfangs- und Nebenbedingungen, die Bewegungsrichtungen zweier Billardbälle nach der Kollision zufällig sind.
Hallo Johann,

zur Verdeutlichung noch eine Ergänzung:
Beim klassischen Billard kann man den Kugeln exakt definierte Orte und Impulse theoretisch zuschreiben, auch wenn man diese niemals exakt messen könnte. Beim 'Quantenbilliard' darf man den "Kugeln" (den Quantenobjekten) exakt definierte Orte und Impulse nicht mal theoretisch vor der Messung zuschreiben.

M.f.G Eugen Bauhof

RoKo
24.04.12, 14:08
Hallo Hawkind,

Das eine hat mit dem anderen nichts zu tun.
Die statistische Mechanik beschäftigt sich mit der Physik von Systemen, die aus immens vielen Komponenten besteht - so viele, dass man kein Interesse daran hat, die Bewegung jeder einzelnen Komponente zu diskutieren. ..weil man es nicht kann. Es geht prinzipiell nicht. Das war immer schon eine Illusion von Physikern, die ihre Abstraktionen für Realität hielten.
Mit der Quantenphysik ist das nur deutlich geworden.

Hawkwind
24.04.12, 14:11
Die Physik des Billard ist übrigens hier recht detailliert beschrieben:

Amateur Physics for the Amateur Pool Player (http://www.sfbilliards.com/shepard_apapp.pdf)

basierend auf der guten alten klassischen Mechanik, aber schwierig genug: "Amateur Physics" aus dem Titel scheint mir schon ein wenig untertrieben.

Obwohl alles eigentlich streng deterministisch abläuft (wenn man denn wirklich alles genau genug kennen würde: Geschwindigkeiten, Spinbewegungen, Reibung, ...), spielen in der 2. Hälfte des Papiers dann statistische Analysen eine bedeutende Rolle. Wenn ich nach erstem Überfliegen recht verstehe, geht es um Gewinnwahrscheinlichkeiten für Spielstrategien und so Zeugs. Das hat wiederum nichts mit der statistischen Natur der Quantentheorie zu tun, denn diese ist hier natürlich außen vor: Billardbälle sind keine Quanten. :)

Hawkwind
24.04.12, 14:14
Hallo Hawkind,

..weil man es nicht kann.


Nein, es interessiert auch einfach nicht. Wozu sollte man die Bahn oder Zustand von Molekül H2O Nummer 21 345 678 234 beim Zustandsübergang einer Menge Wasser in die Dampfphase kennen wollen?

richy
24.04.12, 15:29
Ich meine Joax Aussage kann man als makroskopisches Analogon zur Verdeutlichung der Akausalitaet schon verwenden.
Wir nehmen ein ganz normales, klassisches Billard. Große, exakt zu vermessende Kugeln, exakt definierte Orte und Impulse. Deine Sicht der QM kann man dann so beschreiben, dass trotz der exakten Anfangs- und Nebenbedingungen, die Bewegungsrichtungen zweier Billardbälle nach der Kollision zufällig sind. (Bei Erhaltung des Gesamtimpulses.)
@Bauhof
Hier die Heisenbergsche Unschaerferelation ins Spiel zu bringen traegt gerade nicht dazu bei den objektiven Zufall zu charakterisieren. Die BM nimmt einen solchen nicht an, aber dennoch bleibt Heisenberg davon unberuehrt, alleine schon weil dies ein rein mathematischer Zusammenhang ist. Der von dir in "aktuelle Meldungen" vorgsestellte Versuch unterstreicht dies.Frueher wurde wie von Prof. Zeh mehrfach bemerkt die Unbestimmtheitsrelation unsachgemaess als Begruendung fuer die Irrationalitaet der KD oder gar Nichtlokalitaet verwendet. Diese Argumentationsweise wird heute wohl dank der Realistischen Aufklaerung kaum mehr ziehen. Mir scheint dass jedoch oefters noch der Trugschluss existiert, dass die Bornsche Hypothese lediglich die KI stuetzt. Die Interpretationen untescheiden sich lediglich darin wie die Auswahl zustande kommt. Und das ist primaer eine philosophische Frage. Objektiver=akausaler zufall oder determinierter Zufall=Chaos. Ich meine sogar, dass man hier ausnahmsweise persoenliche Meinungen einfuegen kann. Also z.B. eine BM mit ueberlagertem objektivem Zufall. 5n der Plauderecke gibt es ein schoenes Paper zum Quantenbilliard, dass zeigt, dass selbst ein Elektron am Rande unserer Galaxie einen Billiardstoss beeinflusst. Oder etwas praktischer die Stoesse z.b. der Molekuele der Luft. Damit zeigt sich, dass es voellig ausgeschlossen ist ein Inertialsystem ohne Chaos praktisch aufzubauen. Man muss in der Regel stets von einer Ueberlagerung aus objektivem und determiniertem Zufall ausgehen.'#Gruesse

RoKo
24.04.12, 16:41
Nein, es interessiert auch einfach nicht. Wozu sollte man die Bahn oder Zustand von Molekül H2O Nummer 21 345 678 234 beim Zustandsübergang einer Menge Wasser in die Dampfphase kennen wollen?
a) und wen interessiert, ob ein Elekton nun mit dem Winkel a oder ß am Atom gestreut wird?
b) wie willst du das Molekül H2O Nummer 21 345 678 234 kennenlernen?

Die klassische Mechanik basiert auf Abstraktionen - Punkten und unendlichiche Genauigkeit. Beides ist menschliche Zutat zur Naturerkenntnis. Jeder, der 5 Minuten Zeit zum Nachdenken hatte, der konnte das wissen. Empirische Relevanz hatte es ohnehin nie.

In der Quantenmechanik hat man nun, wenig überraschend, festgestellt, dass Punktteilchen eine Illussion sind. Jedes Quantenobjekt nimmt einen gewissen Raum ein und hat ein gewisses Impulsspektrum. Anstatt das zur Kenntnis zu nehmen, hält man an der Illussion fest und bemüht einen ideologischen Apparat, um das zu rechtfertigen. ("Es gibt keine Quantenwelt" oder auch "man darf dem Quantenobjekt vor der Messung keinen Wert [Ort, Impuls oder was auch immer] zuweisen"). Dabei ist jedem klar, dass eine "Messung" an einem Quantenobjekt keine Messung ist.

Hawkwind
24.04.12, 17:07
a) und wen interessiert, ob ein Elekton nun mit dem Winkel a oder ß am Atom gestreut wird?
b) wie willst du das Molekül H2O Nummer 21 345 678 234 kennenlernen?

Die klassische Mechanik basiert auf Abstraktionen - Punkten und unendlichiche Genauigkeit. Beides ist menschliche Zutat zur Naturerkenntnis. Jeder, der 5 Minuten Zeit zum Nachdenken hatte, der konnte das wissen. Empirische Relevanz hatte es ohnehin nie.

In der Quantenmechanik hat man nun, wenig überraschend, festgestellt, dass Punktteilchen eine Illussion sind. Jedes Quantenobjekt nimmt einen gewissen Raum ein und hat ein gewisses Impulsspektrum. Anstatt das zur Kenntnis zu nehmen, hält man an der Illussion fest und bemüht einen ideologischen Apparat, um das zu rechtfertigen. ("Es gibt keine Quantenwelt" oder auch "man darf dem Quantenobjekt vor der Messung keinen Wert [Ort, Impuls oder was auch immer] zuweisen"). Dabei ist jedem klar, dass eine "Messung" an einem Quantenobjekt keine Messung ist.

Schon klar, Physik basierte schon immer auf Näherungen und Idealisierungen. Genau berechnen lässt sich schlicht gar nichts; das Ziel ist es immer, den Fehler akzeptabel klein zu halten, wenn man irgendeine quantitative Vorhersage machen will.

In der Quantentheorie kommt da nun noch eine neue "Qualität" hinzu: selbst, wenn man von diesen Abstraktionen oder Näherungen absieht, die nötig sind um rechentechnisch etwas lösen zu können, gibt es dort immer noch prinzipielle Unschärfen. Selbst ein (abstraktes) wechselwirungsloses 1-Teilchen-Quantensystem lässt keine simultane Orts- und Impulsmessung (beliebiger Genauigkeit) zu.

amc
24.04.12, 20:58
Hi RoKo,

In der Quantenmechanik hat man nun, wenig überraschend, festgestellt, dass Punktteilchen eine Illussion sind. Jedes Quantenobjekt nimmt einen gewissen Raum ein und hat ein gewisses Impulsspektrum.

wie groß ist dieser Raum? Fliegt ein Quant in alle Richtungen gleichzeitig? Wie muss/kann man sich das vorstellen?

Ich bin absolut bei dir, das Punktteilchen eine idealisierte Illusion sind. Aber ich denke, dass man einem Quant eben grundsätzlich z.B. keinen Raum zusprechen kann (Wenn dann vielleicht nur so: Hier ist es ein Bisschen mehr, und dort ist es ein Bisschen weniger). Wenn die Quantenwelt real ist, dann schätze ich, doch wahrscheinlich auf eine ganz andere Art, als wir es gewohnt sind.

Grüße, AMC

richy
24.04.12, 22:19
Wenn die Quantenwelt real ist, dann schätze ich, doch wahrscheinlich auf eine ganz andere Art, als wir es gewohnt sind.
Eine zusaetzliche zeitartige Dimension genuegt um nichtphysikalisches zu vermeiden. So ungewohnt ist das nicht.Und der Ausdruck real irreall ist bischen irrefuehrend.Die ueberlagerten Zustaende wuerde man auch in einem Realismus nicht unserer Realitaet zuordnen und daher nicht als real bezeichnen. Realistische Interpretationen sind jedoch rein physikalisch. Die KI ist das nicht. Das ist der Unterschied.

Nochmal kurz zu Heisenberg. Fuer zwei konjungierte Groessen ergibt sich anhand der Fouriertransformation eine Abhaengigkeit der Bandbreiten der Funktionen. Sie beschreiben ja das selbe und sind daher voneinander abhaengig.Schmale Urfunktion => Breitbandige Bildfunktion und umgekehrt. Jeder Nachrichtentechniker kennt das Phaenomen und auch entsprechende Abschaetzungsregeln. Das ist noch einfache Mathematik.

Phaenomen 1 :
Die Funktionen stellen Wahrscheinlichkeiten dar. Das fuehrt zur Unbestimmtheit und zwar bei allen Interpretationen.
Phaenomen 2 :
Eine Unbestimmtheit erfordert eine Form des Zufalls. Und hinsichtlich dieses mehr philosophischen Aspekts unterscheiden sich nun die Interpretationen. Wie kommt die Auswahl zustande ? Objektiv zufaellig oder chaotisch z.B. ? Wobei man meint besonders die BM hiemit in Verlegenheit bringen zu koennen. Bisher gelang dies aber nicht wirrklich.
Gruesse

fossilium
24.04.12, 23:06
Hi Emi,

leider komme ich erst jetzt dazu, zu der Sache mit dem „Wurm“ etwas zu sagen.

Zunächst als Erstes: ich habe keinerlei Wurm gefunden.

Ich habe mal wieder im übermässigen Gedankenschwall, und weil ich Deinen Text nicht genau gelesen habe, den Mund zu voll genommen. Das ist eine üble und ärgerliche Angewohnheit von mir, die manchmal durchbricht. Es tut mir leid. Das ist hier doch nicht ein Wurmaufinde-Wettbewerb.

Ich bin momentan total im Stess bin (habe seit drei Monaten eine 80 Stundenwoche) - was keine Entschuldigung sein soll. Es fällt mir auch von daher schwer, auf Deine Argumente einzugehen. Ich habe mich aber trotzdem zu später Stunde an eine Analyse gemacht, die ich hier poste. Vielleicht gibt es dazu ein paar kritische Kommentare.

Betrachten wir mal ein Elektron, dass in einer Apparatur, z.B. einer Bildröhre, aus einer Quelle emittiert wird und mit der Geschwindigkeit von 1000 Meter pro Sekunde (1000 V Beschleunigungsspannung) durch die evakuierte Apparatur fliegt (Ultrahochvakuum).
In einem Kubikmilimeter Raumvolumen im Inneren dieser Apparatur befinden sich nur einige 100 Photonen der Wärmestrahlung (300 Kelvin), und vielleicht noch 1000 Gasmoleküle, also das Elektron hat gute Chancen durchzukommen, ohne mit Photonen oder Molekülen zusammenzustossen oder durch Streuung von seiner Richtung abgelenkt zu werden. Das alles entspricht ja auch den experimentellen Ergebnissen, z.B. in einer Fernsehröhre kann man sich das Elektron als Teilchen vorstellen, das den Raum zwischen Kathode und Schirm auf einer geradlinigen Bahn ungestört durchquert.

Beim Aufprall auf den Schirm wird es dann von der Innenbeschichtung absorbiert. Dabei entsteht darin eine winzige kleine Leuchtemission, der uns signalisiert: genau hier war das Elektron zum Zeitpunkt des Aufpralls. Nun gibt es aber - anders als beim Beispiel der Bahn eines Planeten – keine Theorie, die uns einen Rückschluss auf die Bahn des Elektrons vor dem Aufprall erlaubt. Die Theorie, die die Bewegung des Elektrons durch die Apparatur beschreibt, ist die Quantentheorie. Diese Theorie sagt über die Bahn des Elektrons vor dem Aufprall leider nichts aus (die KD z.B. verzichtet auf eine Aussage) ! Die Theorie liefert bei Kenntnis der Anfangsbedingungen bei der Emission als Vorhersage für alle Flugzeiten nur den wahrscheinlichen Ort des Aufpralls (die Wahrscheinlichkeiten dafür aber mit höchster Genauigkeit). Nach jedem Aufprall kann man zwar eine Bahn mit Anfangs- (Emissionsort) und Endpunkt (Ort des Aufpralls) gedanklich konstruieren. Das bringt uns aber nichts. Wiederholt man das Experiment mit einem zweiten Elektron unter exakt gleichen Bedingungen, dann kann man den Aufprallort nicht genau vorhersagen, und damit auch nicht die Bahn, die das Elektron nehmen wird. Man kann sich bei jedem Experiment daher nur eine wahrscheinliche Bahn vorstellen – aber was das pysikalisch bringen soll, ist mir schleierhaft. Es gibt nun in der Physik derzeit keine Erklärung dafür, dass das Elektron beim Flug durch die Röhre mal diesen, mal jenen Weg nimmt. Man geht daher vielfach davon aus, das die Bahnen rein zufällig verteilt sind, unterstellt also, dass der Liebe Gott hier das des Ursache-Wirkung-Prinzip ausser Kraft gesetzt hat.
Wenn Du also sagst,
Deshalb wird in der Gesamtheit der Systeme, die der Messung nur zum Teil unterworfen wurden, durch die Messung gerade der Zustand bestimmt, der vor ihr bestand.
dann ist zu fragen, welchen Zustand, der vorher bestand, Du meinst, bzw, was diese Aussage für das Beispiel der Bahn des Elektrons zu bedeuten hat.

Sorry noch mal für meine späte Reaktion.
Bitte vergess die Sache mit dem Wurm.

Grüsse Fossilium

RoKo
24.04.12, 23:29
Hallo richy,

.. Mir scheint dass jedoch oefters noch der Trugschluss existiert, dass die Bornsche Hypothese lediglich die KI stuetzt. Nicht dur dir geht das so. Und die Born'schen Originaltexte geben das m.E. auch nicht her.

Die Interpretationen unterscheiden sich lediglich darin wie die Auswahl zustande kommt. Und das ist primaer eine philosophische Frage. Objektiver=akausaler zufall oder determinierter Zufall=Chaos. Ich meine sogar, dass man hier ausnahmsweise persoenliche Meinungen einfuegen kann. Also z.B. eine BM mit ueberlagertem objektivem Zufall. Durch die Annahme von Anfangsbedingungen, und mehr fügt die BM zur Standard-QM nicht hinzu, wird der Zufall nur vom Ende an den Anfang gelegt.
5n der Plauderecke gibt es ein schoenes Paper zum Quantenbilliard, dass zeigt, dass selbst ein Elektron am Rande unserer Galaxie einen Billiardstoss beeinflusst. Oder etwas praktischer die Stoesse z.b. der Molekuele der Luft. Damit zeigt sich, dass es voellig ausgeschlossen ist ein Inertialsystem ohne Chaos praktisch aufzubauen. Man muss in der Regel stets von einer Ueberlagerung aus objektivem und determiniertem Zufall ausgehen.Detlef Dürr zeigt in seinem Buch "BM als Grundlage der QM", wie man das Thema mit statistischen Methoden behandeln kann. Die Art des Zufalls spielt dabei keine Rolle; die Methode ist nicht abhängig von der Annahme von Anfagsbedingungen. Ob der Zufall nun objektiv ist oder nicht oder eine beliebige Mischung, ist also erstens egal und zweitens nicht prüfbar.

RoKo
25.04.12, 00:37
Hallo amc,

wie groß ist dieser Raum? Fliegt ein Quant in alle Richtungen gleichzeitig? Wie muss/kann man sich das vorstellen?
Theoretisch ist der Raum steht das ganze Universum. Näheres sagt uns jedoch die Dichte des Objektes. Es gibt zwar keinen scharfen Rand; i.d.R. ist das Raumgebiet näherungsweise begrenzt.Fliegt ein Quant in alle Richtungen gleichzeitig? Ein Quantenobjekt breitet sich in alle möglichen Richtungen aus. Wie muss/kann man sich das vorstellen?Die Dichte im Raum kann man sich als Wolke vorstellen, wenn man dabei im Kopf behält, dass es sich um ein Objekt handelt, dass erstens kein Individuum ist und zweitens seine Dichte im Raum vom energetischen Zustand abhängt, der nur als Ganzes durch eine lokale Wechselwirkung verändert werden kann. Deren Wahrscheinlichkeit ergibt sich widerum aus der Dichte.

RoKo
25.04.12, 06:00
Hallo Hawkwind,

In der Quantentheorie kommt da nun noch eine neue "Qualität" hinzu: selbst, wenn man von diesen Abstraktionen oder Näherungen absieht, die nötig sind um rechentechnisch etwas lösen zu können, gibt es dort immer noch prinzipielle Unschärfen. Selbst ein (abstraktes) wechselwirungsloses 1-Teilchen-Quantensystem lässt keine simultane Orts- und Impulsmessung (beliebiger Genauigkeit) zu.

Ja - eben weil es keine punktförmigen "Teilchen" gibt.

Nochmals:
Ein Quantensystem hat nur einen energetischen Zustand. Das ist die einzige variable Eigenschaft. Eigenschaften wie Ort oder Impuls hat ein solches System schlicht nicht. Deshalb kann man sie auch nicht messen. An einen Quantensystem kann man schlicht garnichts messen ausser den Systemkonstanten Masse, Ladung und Spin, weil man den energetischen Zustand nicht messen kann. Will man etwas über den energetischen Zustand eines Quantensystems in Erfahrung bringen, dann muss man ihn wirken lassen oder auf ihn einwirken, also ein Wirkungsquantum abziehen oder hinzufügen. Damit verändert man selbstverständlich diesen Zustand.

Die Observablen eines Quantensystems sind keine intrinsischen Eigenschaften des Systems, wie das Kochem-Specker-Theorem ausdrücklich zeigt. Das, was da Messgeräte anzeigen, sind Resultate der Messwechselwirkung. Letztere ergibt sich aus den Operatoren, die auf die Wellenfunktion angewendet werden. Wie der Prozess der Auswahl des angezeigten Wertes vor sich geht, ist in der Standard-QM ein offenes Problem. Auf jeden Fall wird aber das Quantensystem in den energetischen Zustand gebracht, der dem angezeigten Wert entspricht; das ist die allseits bekannte Zustandsreduktion.

Bei einer sogenannten Orts-"Messung" bringt man das Quantensystem in einen energetischen Zustand, dem ein kleines Raumvolumen und ein breites Impulsspektrum entspricht; bei einer sogenannten Impuls-"Messung" ist es genau umgekehrt. Würde man nun Ort und Impuls gleichzeitig "messen", dann käme nichts vorhersagbares dabei raus, weil es keinen energetischen Zustand gibt, dem ein kleines Raumvolumen und ein schmales Impulsspektrum entspricht.

Bauhof
25.04.12, 09:48
Eine Unbestimmtheit erfordert eine Form des Zufalls. Und hinsichtlich dieses mehr philosophischen Aspekts unterscheiden sich nun die Interpretationen. Wie kommt die Auswahl zustande ? Objektiv zufaellig oder chaotisch z.B. ? Wobei man meint besonders die BM hiemit in Verlegenheit bringen zu koennen. Bisher gelang dies aber nicht wirrklich.
Hallo Richy,

die Form des quantenmechanischen Zufalls ist objektiv zufällig. Dies ist seit rund 100 Jahren weltweit stille Übereinkunft der allermeisten Physiker. An dieser Tatsache ändert auch das lautstarke Getöse einer Amateur-Philosophen-Minderheit nichts.

M.f.G. Eugen Bauhof

JoAx
25.04.12, 12:25
Hallo zusammen!

@EMI:
Das mit der el. Ladung, dass sie ein Fremdling ist, würde mich auch näher interessieren. Kannst du dazu mehr sagen, vlt. eine (beide) Quelle(n) angeben?

@Rolf:
Die klassische statistische Mechanik interessiert sich imho nicht für Abstraktionen, die du genannt hast. Man könnte sich da auch ruhig "feste, ausgedehnte Kugelchen" an der Stelle von Molekülen denken. Da bin ich ganz bei Uli.

@Uli:
Auch W=1 ist übrigens möglich. Das ist immer dann der Fall, wenn sich das Quantensystem bereits in einem sog. Eigenzustand zur gemessenen Observablen befindet ... z.B. ein präparierter Zustand ist. In so einem Fall aber könnte man andererseits für eine Messung der entsprechenden konjugierten Observablen überhaupt keine Vorhersage mehr machen (maximale Unschärfe, alles gleich (un)wahrscheinlich).

Wenn die Polarisationsrichtung eines Photons im Eigenzustand ist, was wäre dann die konjugierte Observable dazu?

@Eugen:

Dein Gedankenspiel hat Hawkwind bereits beantwortet:


Das beantwortet meine Frage nicht. Oder wäre die Unbestimmtheit der einzelnen Impulse nach der Kollision in meinem Beispiel nicht "prinzipieller" Natur? Die Aussage - prinzipiell anderer Natur - ist eine "relativ-qualitative" Aussage. Sie sagt nichts über die Natur selbst aus. Nur, dass es halt anders ist.


Beim 'Quantenbilliard' darf man den "Kugeln" (den Quantenobjekten) exakt definierte Orte und Impulse nicht mal theoretisch vor der Messung zuschreiben.


Ist das eine Aussage der Theorie, oder "nur" einer Interpretation (Beschreibung) der selben?
====================

Ich musste da wieder an EMI's Aussage denken, dass der Mensch nur "klassisch" versteht/denkt. Schon vor längerer Zeit habe ich ja damit dagegen argumentiert, dass die Natur sich ja nicht daran halten muss, dem Menschen verständlich zu sein. Man könnte das aber auch anders betrachten. Völlig unabhängig davon, ob wir die Natur je "wahrhaft" werden "verstehen" können, müssen wir über diese sprechen, um schlicht wieter zu kommen. Und sprechen können wir natürlich nur darüber, was wir verstehen. Und nur so, wie wir verstehen (können). Ergo - "klassisch".

Das öffnet natürlich die Möglichkeit, dass es in den auf klassischen Begrifflichkeiten basierten Beschreibungen - Widersprüchliches ergibt. Wie etwa - Verletzung des speziellen Relativitätsprinzips, oder - ungeheuer bis unabzählbar viele "klassische" Welten, oder - "das ist das Problem der Atome, und nicht der Quanten, dass nicht alle auf diesen reagieren". Dazu muss man dann aber stehen und nicht verneinen.

Etwas drastisch und sicher auch überspitzt formuliert - ich kann mit Unbestimmtheit in der Theorie leben, aber "argumentative Willkür" in der Beschreibung ist eine "Unschärfe" vom ganz anderen Kaliber. Da fällt mir nur was von Einstein, aus Einstein's reply to criticisms (http://www.marxists.org/reference/subject/philosophy/works/ge/einstein.htm), ein:
Such a type of derivation (in the sense of logical deducibility) is nowhere to be had, not even in the domain of pre-scientific thinking.
Eine solche Art der Ableitung (im Sinne der logischen Ableitbarkeit) ist nirgends zu haben, nicht einmal im Bereich des pre-wissenschaftlichen Denkens.In diesem Sinne ...

Gruß, Johann

Bauhof
25.04.12, 13:49
Zitat von Bauhof:
Beim 'Quantenbilliard' darf man den "Kugeln" (den Quantenobjekten) exakt definierte Orte und Impulse nicht mal theoretisch vor der Messung zuschreiben.
Ist das eine Aussage der Theorie, oder "nur" einer Interpretation (Beschreibung) der selben?
Hallo Johann,

das ist nicht nur eine Aussage, sondern ein Grundkonsens der Theorie. So ist es überall in der Litreratur nachzulesen. Ich weiß wirklich nicht, was man manche Leute so lesen, um das in Zweifel ziehen zu können.

M.f.G. Eugen Bauhof

Hawkwind
25.04.12, 14:57
Wenn die Polarisationsrichtung eines Photons im Eigenzustand ist, was wäre dann die konjugierte Observable dazu?



Die Situation bei drehimpulsartigen Observablen (Bahndrehimpuls, Spin, Isospin, ...) ist in der Quantentheorie nicht einfach durch ein Paar zueinander konjugierter Observabler beschreibbar.
Man kann maximal immer die Drehimpulskomponente in einer ausgewählten Raumrichtung ("traditionell" die z-Achse) und den Betrag des Drehimpulses zugleich beliebig genau messen. Dreimpulskomponenten in 2 unterschiedliche Raumrichtungen kann man nicht gleichzeitig scharf messen.

Auf den ersten beiden Seiten von
http://users.physik.fu-berlin.de/~russ/qm_folien4b.pdf

werden die Kommutatoren der Drehimpulsoperatoren untereinander diskutiert, z.B.

[Lx,Ly] = ihLz

bedeutet Drehimpuls in x und y-Richtung sind nicht zugleich scharf messbar.

[L^^2,Lz] = 0

bedeutet Quadrat (bzw. Betrag) des Drehimpulses und Drehimpulskomponente in z-Richtung sind zugleich scharf messbar (weil rechte Seite = 0 ist - der Kommutator "verschwindet").

Dort findest du auch, welche Koordinaten und Komponenten des linearen Impulses gleichzeitig mit den Drehimpulskomponenten messbar sind (immer die Paare von Observablen, deren Kommutatoren = 0 sind, sind zugleich messbar).

Gruss,
Hawkwind

richy
25.04.12, 16:22
Hi Bauhof
Obwohl ich einen objektiven Zufall vor allem aus ethischen Gruenden (mit dem erforderlichen Respekt) annehme, kann ich deine Aussagen nicht akzeptieren.
die Form des quantenmechanischen Zufalls ist objektiv zufällig.
Das ist eine willkuerlich dazu Meta physikalisdche Annahme. Diese ist natuerlich von Kopenhagen dadurch begruendet, dass man dort realistische Einfluesse, Chaos durch Komplexitaet, besser vermeidet. Vor dem Wellenkollaps existiert in der Tat keine Quantenwelt in physikalischer Form und daher ist ein objektive Zufall sicherlich leichter handelbar als Chaos.
Experimentell laesst sich bisher kaum etwas in dieser Richtung beweisen. Auch mit Kochem Specker nicht, wobei ich sogar so weit gehe anzunehmen, dass diese Fragestellung nach der Art des Zufalls prinzipiell nicht entscheidbar ist.
Dies ist seit rund 100 Jahren weltweit stille Übereinkunft der allermeisten Physiker.
So kann man das nicht formulieren. Zunaechst ist der objektive nichtkausale Zufall im Gegensatz zu Chaos metaphysikalisch und daher ein Thema der Philosophen, die die Interpretation der KD uebernommen hatten.Es ergeben sich mit dem objektiven Zufall ethisch und relogioes recht gut vertretbare Konsequnezen, so dass diese Ansichten auch ein Physiker teilen kann. Dennoch bleibt der objektive Zufall ein Horrogebilde, an das man sich jedoch gewoehnt hat. Mit dem man sich arrangiert hat. Ebenso gab es lange dazu keine Alternative. Weil die KD eine Monopolstellung inne hatte und die Chaostheorie nicht genuegend bekannt war.(denke mal bis Mitte 80 er)
An dieser Tatsache ändert auch das lautstarke Getöse einer Amateur-Philosophen-Minderheit nichts.
Wen meinst du damit ? Doch nicht womoeglich die nichtlinearen Systemdynamiker ? Der nichtobjektive Zufall ist beschreibbar und damit kein metaphysikalisches oder metamathematisches Gebilde. Die Chaostheorie existierte schon zu Bohr/Einsteins Zeiten, aber erst mit dem Digitalrechner erlebte sie in den 80 ern eine erste allgemeine Verbreitung.Ob Bohr und Einstein deren Konsequenzen bereits genuegend bewusst waren weiss ich nicht. Aber auf jeden Fall ist das keine Privatphilosophie sondern ganz normale Mathematik. Dazu hochaktuell in allen Bereichen der Quantenmechanik
Gruesse

Hawkwind
25.04.12, 17:09
Hi Bauhof
Obwohl ich einen objektiven Zufall vor allem aus ethischen Gruenden (mit dem erforderlichen Respekt) annehme, kann ich deine Aussagen nicht akzeptieren.
...


Ich denke, mit dieser Ansicht stehst du so ziemlich allein auf weiter Flur.
Was bedeutet "objektiver Zufall"? Für mich ist das ein Zufall, der nicht auf unvollständige Kenntnis seitens des Beobachers zurückzuführen ist (wie etwa beim "Laplaceschen Dämon" in der klassischen Mechanik, der im Prinzip alles vorhersagen könnte oder wie in der statistischen Mechanik) sondern prinzipieller Natur ist.

Als Schlusswort ein Zitat von John von Neumann:

Gewiss hat es immer wieder Versuche gegeben, auch den quantenphysikalischen Zufall als einen nur scheinbaren zu erweisen. Am bekanntesten dürfte die Theorie der "verborgenen Parameter" von David Bohm (1952) sein, die inzwischen jedoch mehrfach experimentell widerlegt ist, so dass heute kaum ein Physiker mehr an der Existenz des realen und objektiven Zufalls zweifelt. ...
Die moderne Wissenschaft geht vor allem seit der Existenz der Quantenphysik davon aus, dass es sowohl Zufallsereignisse gibt ... . Damit ist Zufall objektiv und eine ontologische Kategorie.

amc
25.04.12, 17:30
das ist nicht nur eine Aussage, sondern ein Grundkonsens der Theorie. So ist es überall in der Litreratur nachzulesen. Ich weiß wirklich nicht, was man manche Leute so lesen, um das in Zweifel ziehen zu können.

Dazu sicherlich auch immer interessant:
http://de.wikipedia.org/wiki/GHZ-Zustand

Grüße, AMC

fossilium
25.04.12, 20:58
Hi Hawkwind,
Ich denke, mit dieser Ansicht stehst du so ziemlich allein auf weiter Flur.

Das glaube ich nicht, das Richy damit allein auf weiter Flur steht.

Wenn es einen Zufall prinzipeller Natur gäbe, dann wäre das ja einer, bei dem die Ursache-Wirkung Beziehung ausser Kraft gesetzt wäre. Die Frage ist dann: wo kommt die Wirkung her ? Aus dem Nichts ? Und es stellt sich die Frage, was ist die Ursache des Zufalls ? Auch ein Zufall ?
Und warum ist das Nicht-Zufällige nicht auch zufällig ?

Also Du landest in logischen Zirkelschlüssen, im Paradoxien. Ich glaube nicht, dass die meisten Physiker sich an diesem Standort wiederfinden wollen.

Die Frage, ob es einen objektiven Zufall gibt, ist einfach ungeklärt und am meistens wird sie ignoriert - wen jukt´s, manche hoffen insgeheim auf noch auf verborgene Variablen. Das ist jedenfalls eine taffere Haltung als den ganzen paradoxen Kram der Quantenphysik als natürlich zu akzeptieren.

Grüsse Fossilium

richy
25.04.12, 21:11
Ich denke, mit dieser Ansicht stehst du so ziemlich allein auf weiter Flur.
Mich stoert dass Bauhof den objektiven Zufall mit der naturwissenschaftlichen Vorgehensweise verknuepft :
Dies ist seit rund 100 Jahren weltweit stille Übereinkunft der allermeisten Physiker. Die Physiker akzeptieren die Hypothese aus von mir bereits genannten Gruenden. Ohne die Existenz der KD haette jedoch kein Physiker dieser Welt so eine akausale Annahme formuliert. Die Interpretation vor dem Wellenkollaps ist bei Kopenhagen nunmal auch ein von Philosophen gehandeltes Thema. Genau das versucht Bauhof umzudrehen (Zu Determinismus,Physikalitaet schreibt er):
An dieser Tatsache ändert auch das lautstarke Getöse einer Amateur-Philosophen-Minderheit nichts
In den realistischen Interpretationen gibt es weitaus weniger Platz fuer Philosophen. Nichtlineare Systemdynamik betrifft Mathematik und Physik. Ein objektiver Zufall ist dagegen ein philosophisches Konstrukt, das sich nichteinmal mathematisch beschreiben laesst.Deine Beschreibung ist zutreffend, aber auf die Akausalitaet gehtst du nicht ein. Ob der objektive Zufall dennoch existiert moechte ich gar nicht zur Diskussion stellen. Da ich einen freien Wille bevorzuge nehme ich einen solchen Zufall an, aber vorstellen kann ich ihn mir nicht wirklich.
Am bekanntesten dürfte die Theorie der "verborgenen Parameter" von David Bohm (1952) sein, die inzwischen jedoch mehrfach experimentell widerlegt ist, so dass heute kaum ein Physiker mehr an der Existenz des realen und objektiven Zufalls zweifelt. ...
Obwohl ich kein Fan der BM bin : Das ist doch glatt gelogen.
Dreimpulskomponenten in 2 unterschiedliche Raumrichtungen kann man nicht gleichzeitig scharf messen.
Das ist durchaus interessant und hier sollte ich mich mal weiter informieren. Fuer das Wesen des Zufalls ist Heisenberg jedoch unerheblich. Statistiken existieren auch fuer determinierte Groessen. Vor Jahren hatte ich im Forum uebrigends mal die kniffelige Frage gestellt : Kann man einen rein determinierten Zufall praktisch realisieren. Also ohne Ueberlagerung von QM Effekten wie thermisches Rauschen. Ja das geht wenn der Prozesse diskret ist. Und ein Beispiel dafuer waere die Rabbit (goldene) Sequenz..
Gruesse

Hawkwind
26.04.12, 09:58
Fuer das Wesen des Zufalls ist Heisenberg jedoch unerheblich.


Sehe ich nicht so: in der Quantentheorie ist die Heisenbergsche Unschärfe die Begründung für die prinzipielle Natur des Zufalls dort.



Statistiken existieren auch fuer determinierte Groessen.


Das wäre dann halt kein objektiver Zufall (wenn ich "objektiven Zufall" recht verstehe).



Vor Jahren hatte ich im Forum uebrigends mal die kniffelige Frage gestellt : Kann man einen rein determinierten Zufall praktisch realisieren. Also ohne Ueberlagerung von QM Effekten wie thermisches Rauschen. Ja das geht wenn der Prozesse diskret ist. Und ein Beispiel dafuer waere die Rabbit (goldene) Sequenz..
Gruesse

Ist das nicht irgendein Algorithmus, um Zufallszahlen zu generieren?
Warum ist der genannte besser geeignet als andere?

RoKo
26.04.12, 13:51
Hallo Bauhof,



.. Beim 'Quantenbilliard' darf man den "Kugeln" (den Quantenobjekten) exakt definierte Orte und Impulse nicht mal theoretisch vor der Messung zuschreiben.
das ist nicht nur eine Aussage, sondern ein Grundkonsens der Theorie. So ist es überall in der Literatur nachzulesen. Ich weiß wirklich nicht, was man manche Leute so lesen, um das in Zweifel ziehen zu können.


Selbst dann, wenn die o.a. Aussage tatsächlich "überall in der Literatur" zu lesen wäre, wäre sie zu kritisieren:

1. Obige Aussage geht implizit davon aus, dass ein Quantenobjekt ein Punktteilchen ist; sonst wären Eigenschaften wie Ort und Impuls sinnlos. Diese implizite Voraussetzung ist aber durch die Standard-QM nicht gedeckt.

Ein Quantenobjekt/system ist ausschliesslich durch seinen Zustand (Vektor im Hilbertraum) definiert. (Das ist das 1. Postulat der Standard-QM) Der Zustand ist die einzige unabhängige Variable des Systems. Abhängig vom Zustand ergibt sich daraus in den allermeisten Fällen ein Orts -und Impulsspektrum.

"Type mismatch" kann man dazu nur sagen.

2. Mit Ausnahme eines kleinen Kreises von Quantenphysikern (und deren kritiklosen Jüngern) versteht man unter "Messung" das Feststellen an sich vorhandener Werte von an sich vorhandenen Eigenschaften. In diesem Sinne ist obige Aussage ein Widerspruch in sich. Vor allem aber vernebelt sie das eigentliche Geschehen.

Vor z.B. einer Ortsmessung, die man physikalisch als lokale Wechselwirkung zu betrachten hat, befindet sich das Quantensystem in der Regel nicht an einem Ort sondern ist zustandsbedingt räumlich verteilt. Erst mit dieser Wechselwirkung wird es (objektiv zufällig, jedoch abhängig von der[Wahrscheinlichkeits]dichte) lokalisiert. Bekanntermaßen nennt man das auch "Zustandsreduktion" oder Kollaps der Wellenfunktion.

3. Stellt sich mir die (rhetorische) Frage, warum du einerseits die Nichtexistenz der Quantenwelt wie ein Banner vor dich herträgst, andererseits aber von der Existenz von Quantenobjekten und deren Eigenschaften ausgehst?

--
Bezüglich meiner Literatur möchte ich dich auch nicht im unklaren lassen.
Ich beziehe mein Wissen hauptsächlich aus seriösen Quellen wie z.B. Vorlesungsskripten (http://www.phys.ethz.ch/~mrg/QM/QM.pdf) und vor allem aus englischsprachlichen Veröffentlichungen unter ArXiv.quant-ph; z.B. Do we really understand quantum mechanics?
Strange correlations, paradoxes and theorems. (http://arxiv.org/abs/quant-ph/0209123v2)

Hawkwind
26.04.12, 17:12
Ein Quantenobjekt/system ist ausschliesslich durch seinen Zustand (Vektor im Hilbertraum) definiert. (Das ist das 1. Postulat der Standard-QM)


Nicht "ausschließlich" sondern "vollständig" muss es heissen.
In der Praxis reicht z.B. i.a. auch Angabe des Satzes relevanter Quantenzahlen um einen Zustand zu charakterisieren, z.B. die Orbitale im H-Atom.

Philipp Wehrli
26.04.12, 23:17
Als Schlusswort ein Zitat von John von Neumann:
Gewiss hat es immer wieder Versuche gegeben, auch den quantenphysikalischen Zufall als einen nur scheinbaren zu erweisen. Am bekanntesten dürfte die Theorie der "verborgenen Parameter" von David Bohm (1952) sein, die inzwischen jedoch mehrfach experimentell widerlegt ist, so dass heute kaum ein Physiker mehr an der Existenz des realen und objektiven Zufalls zweifelt. ...
Die moderne Wissenschaft geht vor allem seit der Existenz der Quantenphysik davon aus, dass es sowohl Zufallsereignisse gibt ... . Damit ist Zufall objektiv und eine ontologische Kategorie.
Von Neumann ist in diesem Punkt ein denkbar schlechter Zeuge. Er behauptete nämlich, er habe mit seinem von Neumannschen Theorem bewiesen, dass Theorien mit verborgenen Parametern mathematisch gar nicht möglich seien. Dies wohlgemerkt zu einer Zeit, als Bohm längst eine solche aufgestellt hatte.
Übrigens ist auch die Viele-Welten Theorie eine lokal realistische Theorie mit verborgenen Parametern.

richy
27.04.12, 12:00
Sehe ich nicht so: in der Quantentheorie ist die Heisenbergsche Unschärfe die Begründung für die prinzipielle Natur des Zufalls dort.
Ist das Fenster (die Huellenkurve) einer Zeitfunktion kurz (Impuls,Knall) so ist deren Spektrum breitbandig. (Moped mit defektem C am Unterbrecherkontakt streute in alle Frequenzbaender des Analog TV). Diese Eigenschaft der Fouriertransformation gilt auch fuer Ortsfunktionen und Wellenzahlen und ebenso fuer Wahrscheinlichkeiten Auto und Kreuzkorrelierten.. Weisses Rauschen (Konstantes unendlich breites Spektrum) weist einen Diracpuls als AKF Autokorrelierte) auf. => Keinerlei Autokorrelation und dies waere tatsaechlich objektiver akausaler Zufall. Der aber aufgrunde der unendlichen Bandbreite physikalisch nicht existieren kann.Objektiven Zufall muss man sich auch daher als Idelaisierung vorstellen, die ein bandbegrenzendes Element durchlaeuft. Hmm und war das Element vor der Bandbegrenzung nicht ideal erhaelt man das selbe Ergebnis. Heisenberg trifft bereits aus den einfachen genannten Aspekten keine Aussage warum |Psi|^2 eine Wahrscheinlichkeit darstellt. Da waere dann Born. Und der trifft widerum keine Aussage welcher Art dieser Zufall ist. Und folgende Aussage war bereits eine Begrundung dafuer :
Statistiken existieren auch fuer determinierte Groessen. Weil deine Aussage eben zutreffend ist :
Das wäre dann halt kein objektiver Zufall (wenn ich "objektiven Zufall" recht verstehe).
Genau. Determinierter Zufall=Chaos.
Gruesse

Hawkwind
27.04.12, 13:39
Ist das Fenster (die Huellenkurve) einer Zeitfunktion kurz (Impuls,Knall) so ist deren Spektrum breitbandig. (Moped mit defektem C am Unterbrecherkontakt streute in alle Frequenzbaender des Analog TV). Diese Eigenschaft der Fouriertransformation gilt auch fuer Ortsfunktionen und Wellenzahlen und ebenso fuer Wahrscheinlichkeiten Auto und Kreuzkorrelierten.. Weisses Rauschen (Konstantes unendlich breites Spektrum) weist einen Diracpuls als AKF Autokorrelierte) auf. => Keinerlei Autokorrelation und dies waere tatsaechlich objektiver akausaler Zufall. Der aber aufgrunde der unendlichen Bandbreite physikalisch nicht existieren kann.Objektiven Zufall muss man sich auch daher als Idelaisierung vorstellen, die ein bandbegrenzendes Element durchlaeuft. Hmm und war das Element vor der Bandbegrenzung nicht ideal erhaelt man das selbe Ergebnis. Heisenberg trifft bereits aus den einfachen genannten Aspekten keine Aussage warum |Psi|^2 eine Wahrscheinlichkeit darstellt. Da waere dann Born. Und der trifft widerum keine Aussage welcher Art dieser Zufall ist. Und folgende Aussage war bereits eine Begrundung dafuer :
Weil deine Aussage eben zutreffend ist :

Genau. Determinierter Zufall=Chaos.
Gruesse


Ihr argumentiert hier also alle (zumindest Roko, Philip und du) allen Ernstes, dass die Quantenmechanik deterministisch sein soll???

Determinismus bedeutet, dass die Vorhersagen für Messwerte eindeutig folgen, wenn man alle Informationen - wie Anfangsbedingungen, Randbedingungen etc. - zum Problem hat. Das gilt doch nun einmal für die Quantenmechanik offensichtlich nicht; wir sind höchstens in der Lage, Erwartungswerte und Wahrscheinlichkeiten zu berechnen.

Es sei denn, ihr habt in euren Schubladen eure privaten Erweiterungen der Quantenmechanik, die ihren Zufall auf einen Algorithmus zurückführen. Mit Standard-Quantenmechanik hat so etwas sicher nichts zu tun.

RoKo
27.04.12, 14:50
Hallo Hawkwind,

Ihr argumentiert hier also alle (zumindest Roko, Philip und du) allen Ernstes, dass die Quantenmechanik deterministisch sein soll???Wo und wann habe ich jemals argumentiert, dass die Quantenmechanik deterministisch sei?

Z.B. bin ich mit dem letzten Satz des von dir ins Spiel gebrachten John von Neumann sehr einverstanden:
von John von Neumann: Damit ist Zufall objektiv und eine ontologische Kategorie.
Was er jedoch zur Bohm'schen Theorie sagt, ist objektiv falsch und längst widerlegt; ebenso wie alle vermeintlichen Widerlegungsversuche. Die Bohm'sche Mechanik ist gültige Quantenmechanik, die nur zusammen mit der Schrödinger-Gleichung falsifiziert werden kann und welche das paradoxe Messproblem nicht hat. Aber bei Physikern scheint es ja beliebter zu sein, mit Paradoxien zu leben.

Übrigens: Auch die Bohmsche Mechanik schafft den objektiven Zufall nicht aus der Welt; sie verschiebt ihn nur dahin, wo er hingehört - an den Anfang.

Hawkwind
27.04.12, 15:02
Hallo Hawkwind,

Wo und wann habe ich jemals argumentiert, dass die Quantenmechanik deterministisch sei?

Z.B. bin ich mit dem letzten Satz des von dir ins Spiel gebrachten John von Neumann sehr einverstanden:


Na gut - ich fühlte mich hier schon wie der letzte Mohikaner, der noch die banale Schulphysik in Form der Standard-Quantenmechanik vertritt. :)

RoKo
27.04.12, 15:17
Hallo Hawkwind,
.. banale Schulphysik ..Manchmal ist die Schulphysik nicht nur banal, sondern auch falsch. Ich hatte in meiner Schulzeit das große Glück, in Chemie wesentlich mit dem Buch von Linus Pauling "Die Natur der chemischen Bindung" arbeiten zu dürfen. Mein Physikbuch hingegen war grottenschlecht.

Mit solchen Wortungetümen wie "Aufenthaltswahrscheinlichkeit" ist man halt nicht in der Lage, Chemie zu erkären.

Bauhof
27.04.12, 15:40
Mein Physikbuch hingegen war grottenschlecht.
Hallo RoKo,

mein aufrichtiges Beileid.
Dadurch werden viele deiner Äußerungen, die nicht konform mit der Standardphysik sind, für mich nun nachvollziehbar. :rolleyes:

M.f.G. Eugen Bauhof

Bauhof
27.04.12, 15:46
Heisenberg trifft bereits aus den einfachen genannten Aspekten keine Aussage warum |Psi|^2 eine Wahrscheinlichkeit darstellt. Da waere dann Born. Und der trifft widerum keine Aussage welcher Art dieser Zufall ist.

Hallo Richy,

wenn ich meinen Büchern lese, dann erfahre ich im Tenor genau das Gegenteil dessen, was du und RoKo dauernd von sich geben. Über die Begriffe objektive Wahrscheinlichkeit und objektiver Zufall lese ich z.B. in meinen Büchern folgendes:

Abner Shimony schreibt Seite 71 in [1]:
Wenn der Quantenzustand eines Systems eine vollständige Beschreibung des Systems darstellt, dann ist eine Größe, welche in diesem Quantenzustand einen unbestimmten Wert hat, objektiv unbestimmt; ihr Wert ist dem Wissenschaftler, der das System zu beschreiben versucht, nicht nur einfach unbekannt.
Weil außerdem das Ergebnis einer Messung einer objektiv unbestimmten Größe durch den Quantenzustand nicht festgelegt wird, andererseits aber der Quantenzustand die vollständige Information über das System enthält, ist das Ergebnis einer Messung eine Sache des objektiven Zufalls, nicht einfach eine Sache des Zufalls in dem Sinne, dass der Wissenschaftler es nicht vorhersagen kann. Und schließlich ist die Wahrscheinlichkeit für jedes mögliche Ergebnis einer Messung eine objektive Wahrscheinlichkeit.

Ingeborg Strohmeyer schreibt auf Seite 173 in [2]:
Die objektive Unbestimmtheit gründet darin, dass physikalische Größen nach Heisenberg nur als Gegenstände möglicher Messung und Beobachtung definiert sind, d.h. dass ihnen an sich, unabhängig von ihrer Messbarkeit und Messung kein Sein zukommt.
Eine wichtige Folge der Unbestimmtheitsrelation und der dadurch ausgedrückten objektiven Unbestimmtheit ist der quantenmechanische Indeterminismus, der eine definitive Verletzung der Kausalität in der Natur bedeutet – derart, dass sich hinter der beobachteten statistischen Welt nicht noch eine 'wirkliche' Welt verbirgt, in der das Kausalgesetz gilt.

Mit freundlichen Grüßen
Eugen Bauhof

[1] Shimony, Abner
Die Realität der Quantenwelt.
Aufsatz im Sammelband: Quantenphilosophie. (http://www.amazon.de/Quantenphilosophie-Wolfgang-Neuser/dp/3860253662/ref=sr_1_3?s=books&ie=UTF8&qid=1335536886&sr=1-3)
Herausgegeben von Wolfgang Neuser und Katharina Neuser von Oettingen
Heidelberg 1996.

[2] Strohmeyer, Ingeborg
Quantentheorie und Transzendentalphilosophie. (http://www.amazon.de/Quantentheorie-Transzendentalphilosophie-Grundlagen-exakten-Naturwissenschaften/dp/3860257161/ref=sr_1_1?s=books&ie=UTF8&qid=1335537072&sr=1-1)
Heidelberg 1995. ISBN=3-86025-716-1

RoKo
27.04.12, 15:51
Hallo Bauhof,

..
Dadurch werden viele deiner Äußerungen, die nicht konform mit der Standardphysik sind, für mich nun nachvollziehbar.

Den Nachweis müsstest du erst einmal erbringen. Weisst du überhaupt, was in der Physik Standard ist? Deine von mir kritisierte Aussage gehört mit Sicherheit nicht dazu. Du bist doch nicht einmal in der Lage, sie zu verteidigen.

amc
27.04.12, 16:23
Übrigens: Auch die Bohmsche Mechanik schafft den objektiven Zufall nicht aus der Welt; sie verschiebt ihn nur dahin, wo er hingehört - an den Anfang.

Als wirklich objektiv kann man den Zufall nach meinem Verständnis bei DeBroglie-Bohm definitiv nicht bezeichnen.

Zwar ist es wohl so, dass unsere Unkenntnis über die Anfangsbedingungen prinzipieller Natur ist und dass die Standard-Interpretation und DeBroglie-Bohm im praktischen Ergebnis für uns keinen Unterschied bedeuten, aber es ist doch ein ganz wesentlicher Unterschied, ob wir die Ursachen nur prinzipiell nicht in Erfahrung bringen können, sie aber vorhanden sind, oder ob Ursachen prinzipiell nicht existieren.

http://de.wikipedia.org/wiki/Bohmsche_Mechanik#Determinismus
Determinismus

Die De-Broglie-Bohm-Theorie beschreibt die Quantenphänomene deterministisch, d. h., alle Zustandsänderungen sind durch die Anfangsbedingungen (Wellenfunktion und Konfiguration) vollkommen festgelegt. Alle Wahrscheinlichkeitsaussagen sind lediglich der Unkenntnis der speziellen Anfangsorte geschuldet.

Im Gegensatz dazu wird in der üblichen Auffassung (http://de.wikipedia.org/wiki/Kopenhagener_Deutung) die prinzipielle Zufälligkeit von Quantenphänomenen behauptet, zum Beispiel beim Akt der Messung.

Es muss jedoch betont werden, dass aufgrund der Quantengleichgewichtshypothese die Unkenntnis über die Anfangsbedingungen in der De-Broglie-Bohm-Theorie prinzipiell ist und somit der deskriptive Gehalt beider Theorien identisch ist. Vornehm ausgedrückt wird aus der ontologischen Unbestimmtheit der Quantenphysik eine „epistemische“ Unbestimmtheit in der De-Broglie-Bohm-Theorie.


Grüße, AMC

RoKo
27.04.12, 16:56
Hallo zusammen,

Standard-QM ist das, was weltweit im Schnitt an den Universitäten gelehrt wird.
Die Standard-QM basiert i.d.R. auf 5 Postulaten, die je nach Vorlieben des Professors bzw. der Professorin leicht abgewandelt formuliert sind. Nach Abzug der Mathematik ergibt sich ungefähr folgendes Bild:

1. Ein QM-System wird vollständig durch seinen Zustand beschrieben.
2. Der Zustand entwickelt sich gemäß einer Wellenfunktion, die Lösung der Schrödinger-Gleichung ist.
3. Messbaren Größen (Observablen) entsprechen Operatoren, die auf die Wellenfunktion angewandt werden.
4. Mögliche Messwerte sind stets Eigenwerte, deren Wahrscheinlichkeit sich aus der Wahrscheinlichkeitsdichte in Verbindung mit dem Operator berechnen lässt.
5. Nach der Messung befindet sich das System in der zum Messwert gehörenden Eigenfunktion.

Ferner gehören zur Standard-QM einige Theoreme wie die von Heisenberg, Bell, Kochem-Specker und GHZ.

Da Physik eine theoriegeleitete Erfahrungswissenschaft ist, gehören selbstverständlich auch die wesentlichen empirischen Erkenntnisse dazu. Dies sind im wesentlichen
- die Doppelspaltversuche mit Laserstrahlen, Elektronenstrahlen und einzelnen Photonen, Elektronen, Neutronen sowie Fulleren;
- die davon abgeleiteten Versuche mit „verzögerter Wahl“
- die Versuche mit Stern-Gerlach-Magneten, insbesondere auch deren Parallel- und Serienschaltung;
- der sogenannte „Bombentestversuch“;
- die diversen Versuch mit verschränkten Photonen;
- der Nachweis der Verschränkung mit Ionen und ähnlichem;
- usw. usw.

Ferner muß man selbstverständlich in Betracht ziehen, dass die Standard-QM wesentlich zur Entwicklung der Chemie und zur Erklärung der chemischen Bindung beigetragen hat. Ohne den Begriff der Elektronegativität kommt deshalb nicht aus.

Des weiteren ist die Standard-QM Basis der Materialwissenschaften. Neben der Erklärung der verschiedenen Bindungskräfte innerhalb der unterschiedlichsten Festkörper ist sie extrem notwendiger Bestandteil zur Erklärung elektrischer Leitfähigkeit und aller Halbleiter-phänomene. Auf letzterem beruht mittlerweile derart viel in Massenproduktion hergestellter und funktionierender Technik, dass ich die diesbezüglichen QM-Erkenntnisse als gesichert betrachte.

Nichtsdestotrotz gibt es einen Widerspruch zwischen Postulat 2 und Postulat 5, der seiner Auflösung harrt.

richy
27.04.12, 17:00
Ihr argumentiert hier also alle (zumindest Roko, Philip und du) allen Ernstes, dass die Quantenmechanik deterministisch sein soll???Ich habe schon mehrmals erwaehnt, dass ich den objektiven Zufall aus privaten religioes ethischen Gruenden (freier Wille) bevorzuge, aber natuerlich kann ich deshalb die Annahmen von Bohm oder Everett nicht einfach ausklammern.Und wenn man mal ganz ehrlich ist, sind das im Grunde die weitaus vernuenftigeren Annahmen.
Übrigens: Auch die Bohmsche Mechanik schafft den objektiven Zufall nicht aus der Welt; sie verschiebt ihn nur dahin, wo er hingehört - an den Anfang.So sehe ich das auch. Dann hat man die Hauptproblematiken kompakt beisammen, aber es muss nicht so sein. Und mir faellt dann immer der Ausdruck "Createur" ein.
wenn ich meinen Büchern lese, dann erfahre ich im Tenor genau das Gegenteil dessen, was du und RoKo dauernd von sich geben
Ganz neu sind deine Buecher nicht. Und sicherlich wird darin erwahent werden, dass die Zitate nur persoenliche Meinungen darstellen ohne Anspruch auf allgemeine Gueltiggkeit. Ansonsten waeren es falsche Aussagen, die bis Mitte der 90 er aufgrund der KD Monopolstellung durchaus ueblich waren. Als spezielle Aussagen der KD geht das ansonsten in Ordnung. Wobei das Dekohaerenzprogramm wohl letztendlich auf den nichtlinearen Mehrkoerpereigenschaften der Umgebung basiert. Das ist nunmal vom Realismus, Determinismus ausgeliehen.
aber es ist doch ein ganz wesentlicher Unterschied, ob wir die Ursachen nur prinzipiell nicht in Erfahrung bringen können, sie aber vorhanden sind, oder ob Ursachen prinzipiell nicht existieren.
Alleine die Erkenntnis, dass der Unterschied vom physikalischen Aspekt aus gar nicht erfassbar ist, ist fuer mich schon happig. Manche sehen das alles deshalb widerum weitaus lockerer. Zufall ist wie Null und Unendlichkeit ein Thema, dass uns tief in unserer Vorsellungswelt und Selbstverstaendnis trifft. den einen ebenfalls mehr und den anderer weniger.
#Sodele. Ich wollte diesbezueglich ja noch die Verhulst Gleichung weiter entzaubern :-)
Gruesse

RoKo
27.04.12, 17:27
Hallo amc,

bei wikipedia kann jeder halbgares Wissen verbreiten. Nachdenken ist da meisst erfolgreicher.

..Als wirklich objektiv kann man den Zufall nach meinem Verständnis bei DeBroglie-Bohm definitiv nicht bezeichnen..

Die Betrachtungen der BM, die über die Standard-QM hinausgehen, beziehen sich wesentlich auf nicht atomgebundene Quantensysteme. Das, was in der Standard-QM der Fluß der Wahrscheinlichkeit ist, sind in der BM die möglichen Flüsse der Energie, bzw. der "Bahnen" des "Teilchens", von denen eben eine durch die angenommenen Anfangsbedingungen ausgezeichnet ist. Wie diese Anfangsbedingungen zustande kommen, kann auch die BM nicht klären.

Man bedenke z.B. ein Elektron, dass aus einem heißen Wolframdraht austritt. Vor dem Austritt war das Elektron ununterscheidbarer Teil des Leitungsbandes dieses Materials - es war nicht existent; es ist schlicht nur ein Quantum davon. Vor diesem Ereignis lässt sich also garnicht über Anfangsbedingungen reden; erst danach. Ich gehe jedenfalls, bis zur Widerlegung, davon aus, dass auch die BM den objektiven Zufall nicht beseitigt.

amc
27.04.12, 18:06
Hi Roko,

Ich gehe jedenfalls, bis zur Widerlegung, davon aus, dass auch die BM den objektiven Zufall nicht beseitigt.

sei dir selbst überlassen. Dann must du Bohm aber in letzter Konsequenz auch zu den akausalen / indeterministischen Interpretationen zählen. Damit stehst du nicht nur im Widerspruch zu Wiki.

Übrigends kann man es auch als durchaus leichter beurteilen, einen Aufsatz in einem Fachjournal oder Buch zu veröffentlichen, als bei Wiki einen Beitrag zu verfassen, der auch längerfristig erhalten bleibt. Diesen prüfen nämlich nicht nur eine Hand voll Leute, sondern tausende ...

Grüße, AMC

RoKo
27.04.12, 18:22
Hallo Bauhof,

.. wenn ich meinen Büchern lese, dann erfahre ich im Tenor genau das Gegenteil dessen, was du und RoKo dauernd von sich geben..


Der wesentliche Unterschied zwischen mir und dir scheint mir darin zu liegen. dass ich in meinen Büchern nicht nur lese, sondern das hier Gelesene mit dort Gelesenem vergleiche, es wiederum in Beziehung bringe mit anderem Gelesenem; also über das Gelesene nachdenke.

Wenn z.B. Abner Shimony schreibt
Wenn der Quantenzustand eines Systems eine vollständige Beschreibung des Systems darstellt, dann ist eine Größe, welche in diesem Quantenzustand einen unbestimmten Wert hat, objektiv unbestimmt dann hat er zwar einerseits Recht, andererseits macht er den gleichen Denkfehler wie Du. "Type mismatch". Raum bzw. Räume lassen sich nicht mit drei Realvariablen (namens Ort.x, Ort.y, Ort.z) darstellen.

Und wenn Ingebirg Strohmeyer schreibt:
Die objektive Unbestimmtheit gründet darin, dass physikalische Größen nach Heisenberg nur als Gegenstände möglicher Messung und Beobachtung definiert sind, d.h. dass ihnen an sich, unabhängig von ihrer Messbarkeit und Messung kein Sein zukommt., dann disqualifiziert sie sich in meinen Augen schon dadurch, dass sie "physikalische Größen" als "Gegenstände" betrachtet. Auch scheint sie es mit der Unterscheidung zwischen einer "physikalische Größe" und ihrem Wert nicht sehr genau zu nehmen. Es ist bei obigem Satz völlig unklar, ob nun der Größe oder ihrem Wert kein Sein zukommt.

Sorry, aber was haben diese Zitate mit Standard-QM zu tun?

amc
27.04.12, 18:26
Übrigens: Auch die Bohmsche Mechanik schafft den objektiven Zufall nicht aus der Welt; sie verschiebt ihn nur dahin, wo er hingehört - an den Anfang.So sehe ich das auch.

Und einem anderen hier hast du wegen genau dieser Sache fehlende Kenntnis seiner vermeintlich favorisierten Interpretation vorgeworfen? Schon witzig ;)

Man kann ja so ein Modell entwerfen, ein solches annehmen, aber Standard DeBroglie-Bohm ist das dann, nach allem was ich meine zu wisssen, ganz bestimmt nicht, worauf du selbst oft vehement hingewiesen hast.

Grüße, AMC

P.S. Hoffe dir geht es zumindest in kleinen Schritten besser. Nicht das Lächeln verlieren, das hilft bei allem :)

RoKo
27.04.12, 18:33
.. akausalen / indeterministischen ..Indeterministisch ist nicht akausal. Das Kausalititätsprinzip wird auch durch den objektiven Zufall nicht ausser Kraft gesetzt. Der Determinismus hingegen schon. Letzterer setzt voraus, dass die Natur sich unendlich genau verhält.

amc
27.04.12, 18:35
Indeterministisch ist nicht akausal. Das Kausalititätsprinzip wird auch durch den objektiven Zufall nicht ausser Kraft gesetzt. Der Determinismus hingegen schon. Letzterer setzt voraus, dass die Natur sich unendlich genau verhält.

Ok. Dann bleiben wir eben bei indeterministisch. Ändert ja nix :)

Grüße, AMC

richy
27.04.12, 19:58
Und einem anderen hier hast du wegen genau dieser Sache fehlende Kenntnis seiner vermeintlich favorisierten Interpretation vorgeworfen? Schon witzig Kannst du den Fall mal konkret verlinken ? In der BM bin ich nicht so sicher wie Roko. Ich meine aber dass ich nie etwas anderes behauptet haette als dass diese einen Determinismus annimmt. Und Emi hat das gegenteil behauptet. Ebenso schon mehrfach erwaehnt : Dieser Determinismus koennte durchaus ausnahmsweise nicht zwingend sein. Aber der objektive Zufall ist nun mal ein so philosophisch abgefahrenes Konzept, das die BM einen solchen sicherlich allgemein nicht annimmt. Dafuer sogar einige Unannehmlichkeiten in Kauf nimmt. Diskretisiert kann man es sich vielleicht am besten vorstellen. Auch wenn in jedem Schritt alle Groessen determiniert waeren, kein Gott wuerfelt stellt sich die Frage wie dann die ersten Anfangswerte zustande kamen.
Was war vor dem Urknall ? Sicherlich nichts physikalisches Das ist nunmal auch immer nahe zu Gottesfragen. Und die Unsymetrie der Anfangswerte sieht man ueberall. Zum Beispiel an der Verteilung der Sterne.
Man kann ja so ein Modell entwerfen, ein solches annehmen, aber Standard DeBroglie-Bohm ist das dann, nach allem was ich meine zu wisssen, ganz bestimmt nicht, worauf du selbst oft wehement hingewiesen hast.
Jetzt blicke ich es nimmer. Standard BM ist sicherlich determiniert bis auf den Urknallzustand. den klammert aber die BM sicherlich aus. Den daraus folgenden Createur. Und ROKO weicht scheinbar etwas davon ab. Das sehe ich wie du, wobei er aber nur den Anfangszustand mit betrachtet. Also keine wirkliche Abweichung. Das ist wie bei der VWI. Ist diese nun determiniert ? Ja aber nur hochdimensional. Und EMI traut sich natuerlich nicht Bauhofs Lehrmeinung grundlegend zu widersprechen.
P.S. Hoffe dir geht es zumindest in kleinen Schritten besser.
Danke. Leider nicht wirklich. Im Moment geht es aber :-)
Gruesse

EMI
27.04.12, 20:22
Und EMI traut sich natuerlich nicht Bauhofs Lehrmeinung grundlegend zu widersprechen.Unsinn! EMI traut sich nicht, einfach UNSINN!!

richy
27.04.12, 20:33
Hi Roko
Indeterministisch ist nicht akausal.
aber es bestehen Abhaengigkeiten.
Das Kausalititätsprinzip wird auch durch den objektiven Zufall nicht ausser Kraft gesetzt.
Na betrachte vereinfacht doch einfach die AKF von weissem Rauschen. Fuer jedes noch so kleines tau existiert keine Wirkung auf das Ereignis bei tau=0. Es gibt einfach keinerlei Grund fuer dessen auftreten. Das betrachte ich als akausal. Wie siehst du die Bandbegrenzung aus nachrichtentechnischer Sicht ? Und geht man noch tiefer. Aufgrund des Zeitpfeils der vor der Dekohaerenz nicht existiert kann es ebenfalls keine Kausalitaet geben, denn diese erfordert solch einen Zeitpfeil.
Der Determinismus hingegen schon. Letzterer setzt voraus, dass die Natur sich unendlich genau verhält.Der objektive Zufall ist wohl sogar so definiert. Als nicht beschreibbar.
Gruesse

amc
27.04.12, 21:06
In der BM bin ich nicht so sicher wie Roko. Ich meine aber dass ich nie etwas anderes behauptet haette als dass diese einen Determinismus annimmt.

Das ist richtig. Deshalb kann sie ja auch, nach meinem Verständnis, keinen objektiven Zufall enthalten.

Jetzt blicke ich es nimmer. Standard BM ist sicherlich determiniert bis auf den Urknallzustand. den klammert aber die BM sicherlich aus.

Falls RoKo mit dem "Anfang", wo der Zufall hingeschoben sein soll, den Urknall meinte, dann habe ich ihn missverstanden. Nach folgenden Aussagen glaube ich aber, du hast ihn missverstanden:

Man bedenke z.B. ein Elektron, dass aus einem heißen Wolframdraht austritt. Vor dem Austritt war das Elektron ununterscheidbarer Teil des Leitungsbandes dieses Materials - es war nicht existent; es ist schlicht nur ein Quantum davon. Vor diesem Ereignis lässt sich also garnicht über Anfangsbedingungen reden; erst danach.

Detlef Dürr versucht, alles auf statistische Quantenmechanik zuückzuführen. Das entspräche dem determinierten Zufall. M.E. gibt es aber an der Quelle objektiven Zufall. Wenn z.B. die Wellenfunktion eine Kugelwelle beschreibt, dann sind alle Möglichkeiten gleichberechtigt. Da nur eine realisiert werden kann, ist das die Grenze jeglicher Determiniertheit.

Das zweite Zitat ist eineinhalb Jahre alt und stammt aus diesem Beitrag (http://www.quanten.de/forum/showpost.php5?p=53852&postcount=15), in dem er von dir gestellte Fragen beantwortet. Da wird es eigentlich alles recht deutlich. Also nach RoKo's Einschätzung und Überlegungen existiert nicht bloß Zufall beim Urknall, sondern jener ist objektiv und wählt die Anfasngsbedingungen und damit die Trajektorie / die Bahn des Teilchens. Das will ich nicht kritisieren, nur kann man dann IMHO eben nicht mehr von einem echten Determinismus sprechen, welcher DeBroglie-Bohm aber allseits zugesprochen wird. Wobei wir aber jedoch aus prinzipiellen Gründen die Anfangsbedingungen nicht in Erfahrung bringen können, was vielleicht auch EMI damit meinte, als er sagte, De Broglie sei nicht deterministisch.

Mal eine eher rhetorische Frage dazu: Wie soll denn bei DeBroglie-Bohm das stets punktförmige Teilchen an den "Anfangsort" gekommen sein, wenn dieser durch den echten Zuall bestimmt wird? Durch Geisterhand?

Das Kausalititätsprinzip wird auch durch den objektiven Zufall nicht ausser Kraft gesetzt. Der Determinismus hingegen schon. Letzterer setzt voraus, dass die Natur sich unendlich genau verhält.

Also stimmt RoKo zu, wenn DeBroglie-Bohm, dann favorisiert er eine indeterministische Variante. Erscheint mir erst einmal suspekt, aber er hat ja seine Gründe.

Grüße, AMC

P.S. Oft vehement vorgeworfen hast du es nicht. In dem Jahr, seit dem ich hier bin, kann ich mich nur an einen Fall erinnern. :) Um das Thema nicht noch weiter unnötig aufzukochen, verlinke ich jetzt nicht darauf. Vielleicht sollten alle versuchen, vehemente Vorwürwe ein Bisschen weniger vehement zu äußern, ich denke wirklich, dass dies keiner verdient hat.

richy
28.04.12, 03:12
Hi amc
Bei der BM ertappe ich mich oefters dabei, das ich dieses bild als (falsche) Orientierung verwende.
http://upload.wikimedia.org/wikipedia/commons/thumb/0/02/Doppelspalt.svg/249px-Doppelspalt.svg.png
Da existiert der eigenschaftslose Prototyp eines Teilchenns und im Falle des DS wird eine der obigen Trajektorien ausgewaehlt und durchlaufen ? Und auch in anderen Faellen. Und das fuehrt zu neuen Anfangsbedingungen und einer neuen Trajektorienauswahl ? Noe, denn das entspraeche einer Quantelung der Zeit, zeitliche Iterationsschritte. Davon ist in der BM aber keine Rede. Das muss noch abgefahrener ablaufen. Seit dem Urknall liegt eben das Set von Trajektorien schon fest. Die leeren die man einzeichnet, koennte man sich sparen. Oder wie siehst du dies ? Was rechtfertigt eine zeitlich schrittweise Betrachtung von Trajektorienset zu Trakektorienset?
Und ein Verlegenheisboehmianer wird noch einen typischen Fehler begehen. Den Konfoigurationsraum muss man natuerlich verheimlichen. Denn oh Schande dieser zeigt, dass auch die BM eine Variante von vielen Welten ist. Laesst man diesen jedoch links liegen kommt man noch mehr in Erklaerungsnot mit dem Determinismus. Ich wuesste im Moment auch nicht ob in der BM ein hochdimensionaler Determinismus gemeint ist wie in der VWI. Dem muesste im Grunde so sein. Man darf den Aspekt einfach nicht uebergehen. Selbst in der KI, der abstrakten VWI nicht.
Mit dem Wiki Artikel komme ich auch nicht so ganz klar, denn dieser suggeriert fuer mich ein springen von Auswahl zu Auswahl. Wie die Auswahl zustande kommt, klaro das steht in den Sternen. Im wahrsten Sinne des Wortes. Aber wie oft stellt sich diese Auswahlfrage ueberhaupt ?
http://de.wikipedia.org/wiki/De-Broglie-Bohm-Theorie#Quantengleichgewichtshypothese
Einmal beim Urknall. Und dann ?

Gruesse

Bauhof
28.04.12, 09:27
Zitat von richy:
Und EMI traut sich natuerlich nicht Bauhofs Lehrmeinung grundlegend zu widersprechen.
Unsinn! EMI traut sich nicht, einfach UNSINN!!
Hallo EMI,

das ist nicht nur Unsinn von Richy, sondern UNSINN² !

M.f.G. Eugen Bauhof

RoKo
28.04.12, 11:13
Hallo amc, hallo richy,

die BM (1) kann die Vorhersagen der Standard-QM nur reproduzieren, wenn sie von einer |psi|²- Verteilung der möglichen Anfangswerte annimmt. Weil die Wahrscheinlichkeit eine Erhaltungsgröße ist, ist Standard-QM nichts anderes als die Abbildung einer |psi|²- Verteilung zum Zeitpunkt t=Anfang auf eine andere |psi|²- Verteilung zum Zeitpunkt t=Ende. Damit wird m.E. der objektive Zufall nur verschoben.

Nehmen wir als Beispiel eine Elektronenkanone, einen Doppelspalt und einen Schirm zur späteren Lokalisierung. Ich gehe davon aus, dass der genaue Austritt der Elektronen aus der Kanone ein objektiv zufälliger Prozess ist. Unmittelbar nach dem Austritt hat man dann eine typische Gaussche Normalverteilung der möglichen Anfangskonfigurationen. Von da ab ist es dann ein völlig deterministischer Prozess. Das Bild der Bohmschen Bahnen ist übrigens genau so berechnet worden. Aus Sicht der Standard-QM sind es keine Bahnen, sondern die Iso-Linien der Wahscheinlichkeit bzw. eine Darstellung des Wahrscheinlichkeitsflusses.

Mir reicht diese minimale Ergänzung der Standard-QM aus, um aus ihr eine sinnvolle Theorie zu machen.

Selbstverständlich steht es frei, auch solche Prozesse wie den Austritt von Elektronen aus einem Festkörper als durch Anfangsbedingungen determiniert zu betrachten. Eine analytische Gleichung hierzu habe ich aber noch nicht gesehen. Ich seh deshalb keinen Grund, meine Position aufzugeben.

RoKo
28.04.12, 11:17
.. Aber wie oft stellt sich diese Auswahlfrage ueberhaupt ?
.. Einmal beim Urknall. Und dann ?
Ich vermute, immer dann, wenn aus einer Kugelform heraus die Entscheidung für eine Raumrichtung fallen muss.

kingcrimson04
28.04.12, 12:03
Hallo RoKo,

erspare uns bitte in Zukunft deine Polemik.


So einfach ist das leider nicht. Ich weiß nicht, was du mit 'Zustand' genau meinst, ich unterstelle mal, dass du dir vorstellst, dass man dem Elektron in jedem Augenblick eine wohldefinierte Position und eine wohldefinierten Impuls zusprechen kann. Das ist nicht der Fall. Anton Zeilinger formuliert das in seinem Buch [1] auf Seite 84 wie folgt;



Zeilinger schreibt weiter auf Seite 92:



Das heißt, wir dürfen dem Elektron vor der Messung nicht irgendeinen Zustand zusprechen.

Mit freundlichen Grüßen
Eugen Bauhof

[1] Zeilinger, Anton
Einsteins Spuk.
Teleportation und anderer Mysterien der Quantenphysik (http://www.amazon.de/dp/3570006913)
München 2005. ISBN=978-3-570-00691-7

Da muss ich Herrn Bauhof recht geben. Denn wenn RoKo sagt ein Elektron befindet sich immer in einem Zustand. Dann muss er sich fragen lassen wer diesen Zustand nun festlegt

mfg
aus derzeit Old Germany
kc

RoKo
28.04.12, 13:25
Hallo kingcrimson04,

Denn wenn RoKo sagt, ein Elektron befindet sich immer in einem Zustand. Dann muss er sich fragen lassen wer diesen Zustand nun festlegt. Niemand ausser der Natur selbst.

RoKo
28.04.12, 14:49
Hallo zusammen,

nur mal so zur Information am Rande:

Vermutlich hat Niels Bohr niemals den hier oft vorgetragenen Satz "Es gibt keine Quantenwelt" gesagt.

Siehe hier:
http://www.todayinsci.com/B/Bohr_Niels/BohrNiels-Quotations.htm
When asked ... [about] an underlying quantum world, Bohr would answer, 'There is no quantum world. There is only an abstract quantum physical description. It is wrong to think that the task of physics is to find out how nature is. Physics concerns what we can say about Nature.'

As quoted in Aage Petersen, 'The Philosophy of Niels Bohr', Bulletin of the Atomic Scientists, 1963, 19, 12.
Note: Bohr's remark, although in quotation marks, should not be regarded as a direct quote in these exact words. It is a generalised statement in the article author's words to represent Bohr's viewpoint. This is explained in a footnote in Michael Frayn, The Human Touch (2007), 431 based on an article by N. David Mermin in Physics Today (Feb 2004).

und hier:
F. Laloe
In this context, it is understandable why he once even stated that
“there is no quantum concept” [32]!

[32] C. Chevalley, “Niels Bohr’s words and the Atlantis of Kantianismin”,
pages 33-57 in “Niels Bohr and contemporary philosophy”, J. Faye
and H. Folse eds., Dordrecht Kluewer (1994).

o.a.A, 13 (http://arxiv.org/abs/quant-ph/0209123v2)

amc
28.04.12, 15:06
Moin RoKo,

Vermutlich hat Niels Bohr niemals den hier oft vorgetragenen Satz "Es gibt keine Quantenwelt" gesagt.

Ich würde es begrüßen, wenn er ihn nicht gesagt haben sollte. Obwohl ich es mir bei Bohr gut vorstellen kann. (Ich rechne es Bohr extrem hoch an, dass er unser Weltbild so entscheidend verändert und IMHO vorangebracht hat. Das war vielleicht nur möglich, wenn er dabei möglicherweise auch mal zu extrem in seinen Ansichten war.)

Solch ein absolutistischer Satz entspricht jedenfalls nicht meinem Verständnis von Kopenhagen. Ich würde es eher so sagen: "Wir gehen von keiner Quantenwelt aus, aber im Grunde wissen wir es nicht." Fragen, die offen sind, die müsen eben auch offen bleiben.

nur mal so zur Information am Rande

http://de.wikipedia.org/wiki/De-Broglie-Bohm-Theorie#Geschichte
Bohm bezeichnete die Theorie später als ontologische bzw. kausale Interpretation der Quantenmechanik.

;)

Grüße, AMC

Bauhof
28.04.12, 16:46
Ich würde es begrüßen, wenn er ihn nicht gesagt haben sollte. Obwohl ich es mir bei Bohr gut vorstellen kann. (Ich rechne es Bohr extrem hoch an, dass er unser Weltbild so entscheidend verändert und IMHO vorangebracht hat. Das war vielleicht nur möglich, wenn er dabei möglicherweise auch mal zu extrem in seinen Ansichten war.)

Hallo AMC,

doch, Niels Bohr hat es gesagt. Ich habe das jedenfalls in zwei verschiedenen Büchern gefunden und mir nicht aus den Fingern gesogen.

In [1] auf Seite 86:

Bohr erkennt an, dass der Mensch ein Bedürfnis nach einer Beschreibung hat, ein Bedürfnis danach, »die richtigen Worte« zu finden. Indem er darauf hinweist, dass die Beschreibung eines isolierten Systems »abstrakt« ist, distanziert er sich jedoch von der ontologischen Vorstellung, nach der eine solche Beschreibung ein konkretes Abbild der wirklichen Natur darstellt.

»Es gibt keine Quantenwelt«, so behauptete Bohr. »Es gibt nur eine abstrakte Quantenbeschreibung.«

In [2] auf Seite 82:

Die Kopenhagener Deutung beantwortet die metaphysische Frage über die Natur der Welt mit einem klaren Nein: Die objektive Realität ist eine Illusion, die wir zu unserem eigenen Wohl erschaffen. Bohr kam zu der Überzeugung, dass wir uns bestenfalls ein geschlossenes Modell der Welt schaffen können, das ihre gemessenen Eigenschaften reproduziert, ohne dass wir behaupten können zu beschreiben, was wirklich ist. In einem Brief an einen Kollegen erläutert er seine Sichtweise von der Aufgabe der Wissenschaft: «Unsere Aufgabe besteht nicht darin, in das Wesen der Dinge einzudringen, dessen Bedeutung wir ohnehin nicht kennen, sondern Konzepte zu entwickeln, mit deren Hilfe wir in fruchtbarer Weise über die Phänomene in der Natur sprechen können.»

An anderer Stelle brachte er denselben Gedanken etwas deutlicher zum Ausdruck: «Es gibt keine Quantenwelt. Es gibt nur eine abstrakte quantenphysikalische Beschreibung. Es ist falsch zu denken, die Aufgabe des Physikers sei zu ergründen, wie die Natur wirklich ist. Die Physik bezieht sich auf das, was wir über die Natur sagen können.»

Am Beispiel des Wasserstoffatoms können wir Bohrs Einstellung verdeutlichen. Die Frage lautet nicht mehr: «Wie ist das Atom wirklich aufgebaut?», sondern: «Mit welcher Wahrscheinlichkeit finden wir das Elektron hier oder dort?» Die höchste Form der Realität ist nach Bohr nicht mehr das Ding an sich, sondern die Menge all unserer Informationen über das Ding, quantifiziert in den Begriffen der Wahrscheinlichkeit. Die Wellenfunktion, in der diese Information kodiert ist, besitzt daher mehr Realität als die Vorstellung von einem Elektron, das wie ein Planet den Atomkern umkreist.

Mit freundlichen Grüßen
Eugen Bauhof

[1] Malin, Shimon
Dr. Bertlmanns Socken.
Wie die Quantenphysik unser Weltbild verändert. (http://www.amazon.de/Bertlmanns-Socken-Quantenphysik-Weltbild-ver%C3%A4ndert/dp/3379008095/ref=sr_1_1?s=books&ie=UTF8&qid=1335627324&sr=1-1)
Leipzig 2003. ISBN=3-379-00809-5

[2] Baeyer, Hans Christian von
Das informative Universum.
Das neue Weltbild der Physik.
(http://www.amazon.de/Das-informative-Universum-Weltbild-Physik/dp/3406527078/ref=sr_1_1?s=books&ie=UTF8&qid=1335627380&sr=1-1)München 2005. ISBN=3-406-52707-8

RoKo
28.04.12, 17:02
Hallo Bauhof,

.. doch, Niels Bohr hat es gesagt. Du gibst zwei indirekte Quellen an. Welche Quelle geben den diese Quellen an? Vor allem, in welcher Sprache hat er es gesagt?

mermanview
28.04.12, 20:54
---

Nach der Kopenhagener Deutung wird spästestens im Bereich der Planck-Skala das "Ursache-Wirkungsprinzip" "verneint", man wähnt überlagerte Zustände, gleichzeitige Möglichkeiten, ... wie auch immer man es nennen mag.

Meiner Meinung nach muss man Einstein u.co. zugestehen, dass sich zumindest die tatsächliche Lösung im Verborgenen befindet, unabhängig davon, ob überlagert oder chaotisch determiniert.

Die Befürworter verborgener Variabeln haben zudem die Komplexheit des Ganzen auf ihrer Seite:

Mögen sich quantenmechanische Prozesse im Kleinen abspielen, dennoch sind sie Teil des Universums, des Mikro- des Midi- des Meso und des Makrokosmosses, eingebettet in dessen kausale Komplexität.

Kausale Anstösse müssen nicht unbedingt nur die Richtung von Klein nach Groß haben. Schon garnicht wenn man die Gravitation in die Quantenwelt mit einbezieht.

Ich bin kein Experte, aber soweit ich weiß, haben Bohr und vor allem auch Heisenberg stets daruf hingewiesen, dass ihre Theorie lediglich mathematische Modelle zur Beschreibung des Beobachteten sind.

Mathematische Modelle bergen immer die Gefahr etwas außer Acht zu lassen.

Entschuldigt, ... als Garnicht-Trinker stehe ich heute unter dem Einfluss von 0,375l Bier, ich kann nicht mehr beurteilen was ich schreibe, Hicks.

Gruß Merman

richy
28.04.12, 21:36
Die Aussagen die Bauhof formuliert spiegeln meiner Meinung sehr gut die Ansichten der Kopenhagener Deutung wieder. Und es ist ungemein wichtig diese explizit auszudruecken.

»Es gibt keine Quantenwelt«, so behauptete Bohr. »Es gibt nur eine abstrakte Quantenbeschreibung.«Genau so verstehe ich die KD,KI. Und zwar wortwoertlich. Die abstrakte Beschreibung ist die nichtreale nichtphysikalische Entitaet. Es existiert nichts beschriebenes.Keine physikalische Quantenwelt. Nur so gelingt eine nichtreealistische Interpretation. Yepp unvorstellbar. Ich dachte als Schueler/Student jahrelang ich muss das falsch verstehen.

RoKo
29.04.12, 01:44
Hallo zusammen

Die Aussagen die Bauhof formuliert spiegeln meiner Meinung sehr gut die Ansichten der Kopenhagener Deutung wieder. bzw. das, was andere sich darunter vorstellen.Und es ist ungemein wichtig diese explizit auszudruecken... um es zu kritisieren. Die Ablehnung des Realismus und seine populärwissenschaftliche Darstellung führt bei Menschen, die es nicht gelernt haben, mit den "mathematischen Tools" umzugehen, zu Unverständnis, Missverständnis oder Esotherik.

Bezüglich des angeblichen Bohr-Zitates "Es gibt keine Quantenwelt" ist aber zumindest klar, dass er es nie wörtlich gesagt hat. (o.a.A) Ich

amc
29.04.12, 14:39
Moin RoKo, Richy,

Bezüglich des angeblichen Bohr-Zitates "Es gibt keine Quantenwelt" ist aber zumindest klar, dass er es nie wörtlich gesagt hat.

Wie soll sowas denn klar sein können? Wir werden das doch wohl nie wirklich klären können. Vielleicht hat ja einer von uns irgendwann das Glück, und kann ihn dazu im Jenseits befragen.

Nach meiner (bescheidenen) Einschätzung jedenfalls würden z.B. Weizsäcker oder Zeilinger so eine Aussage im Regelfall nur mit dem Nachstz: "Nach allem was wir wissen" treffen/getroffen haben. Ich kann mir gut vorstellen, dass Bohr diesen Nachsatz zwar auch immer im Kopf hatte, er ihn aber durchaus desöfteren weg lies.

Wir können nur von dem ausgehen, von dem wir wissen, dass es existiert. Wenn wir es nicht wissen, dann können wir höchstens spekulative Annahmen machen. Das ist mein Kopenhagen-Verständnis. Es gibt sicher andere, so wie sich Ansichten unter Viele-Weltlern z.B. ja auch unterscheiden können.

Mal nebenbei, um nochmal darauf rumzureiten, so ganz waschechte Realisten seit ihr nun ja nicht mit eurem objektiven Zufall, oder? Wenn man überspitzen will, dann könnte man euch IMHO auch ein verkapptes abgewandeltes Kopenhagen vorwerfen. Ihr schiebt die vermeintlichen Irrealitäten bzw. vermeintlichen Problematiken doch nur an eine andere Stelle, oder nicht? :) Ich hab ja nichts dagegen ich meine nur ...

Ich würde übrigends nicht soweit gehen und sagen, dass ich an einen reinen Zufall glaube, dass ich von diesem ausgehe. Momentan sieht alles danach aus, mehr lässt sich wohl nicht sagen. Ich glaube auch eher daran, das dies letztendlich für immer ein nicht lösbares Geheimnis bleiben wird. Richy, wenn ich dich richtig erstehe, siehst du das genauso?

Grüße, AMC

RoKo
29.04.12, 16:30
Hallo amc,

.. [bezüglich des Bohr-Zitates]
Wie soll sowas denn klar sein können? Wenn die o.a. Link und den Inhalt aufmerksam gelesen hättest, dann wüsstes du es. dort steht:
von http://www.todayinsci.com/B/Bohr_Niels/BohrNiels-Quotations.htm: (http://www.todayinsci.com/B/Bohr_Niels/BohrNiels-Quotations.htm)Note: Bohr's remark, although in quotation marks, should not be regarded as a direct quote in these exact words. It is a generalised statement in the article author's words to represent Bohr's viewpoint. This is explained in a footnote in Michael Frayn, The Human Touch (2007), 431 based on an article by N. David Mermin in Physics Today (Feb 2004). Es passt auch nicht ins Bild. Niels Bohr wahr ein philosophisch gebildeter Mensch - wie alle damaligen Physiker. Er hätte m.E. eine solche absolute ontologische Aussage nicht gemacht, weil Widerspruch steht zu Aussagen, die er tatsächlich gemacht hat.

So, wie (o.a.A) Laloe es auslegt: "There is no quantum concept" erscheint es mir schon deshalb sinniger, weil "There is no quantum world" die logische Konsequenz "There is no world" zur Folge hat. Die objektive Existenz einer Welt, wie sie die klassische Physik beschreibt, hat Bohr nie geleugnet. Er hat vielmehr (ebenso Heisenberg) darauf bestanden, unsere Erkenntnisse über Quanten in den Begriffen der klassischen Physik auszudrücken, damit wir überhaupt wissen, wovon wir reden.

Wir können nur von dem ausgehen, von dem wir wissen, dass es existiert. Wenn wir es nicht wissen, dann können wir höchstens spekulative Annahmen machen.
Das ist richtig. Nur ist unser Wissen gegenüber 1927 erheblich gewachsen. Wir können deshalb nicht geistig in diesem Jahr hängen bleiben. (Und so kommt mir einiges doch vor.) Das ist ein Aspekt.

Hinzu kommt: Ob etwas existiert, wissen wir prinzipell nur über Veränderungen von Existentem. Alles andere sind Schlußfolgerungen. Weil wir "sehen" können [beachte den physikalischen und biologischen Prozess des Sehens] sind wir uns dessen nicht unmittelbar bewusst. Dazu bedarf es intellektueller Anstrengung.

Bezogen auf den Diskussionsgegenstand Quantenphysik bedeutet das, dass wir in der Tat prinzipiell nicht wissen können, wie die Natur auf dieser Skala "ist". Wir können stets nur wissen, wie sich die Natur verhält, wenn wir gezielt auf sie einwirken. Es ist m.E. extrem wichtig, diesen Unterschied zu sehen.

Nach allem, was wir derzeit wissen, müssen wir davon ausgehen, dass die Wellenfunktion reales Naturverhalten beschreibt und nicht nur ein mathematisches Tool zur Berechnung von Wahrscheinlichkeiten ist.

Mal nebenbei, um nochmal darauf rumzureiten, so ganz waschechte Realisten seit ihr nun ja nicht mit eurem objektiven Zufall, oder? Der objektive Zufall ist ja nun keine Willkür. Wir wissen stehts genau, was möglich und wie wahrscheinlich es ist.

amc
29.04.12, 19:46
Wenn die o.a. Link und den Inhalt aufmerksam gelesen hättest, dann wüsstes du es. dort steht:

Das habe ich gewiss gemacht. Genauso wie ich mir nicht sicher sein kann, ob jemand etwas gesagt hat, wenn irgendwo steht, dass es so sei, kann ich mir nicht sicher sein, dass es nicht so ist, wenn irgendwo steht, dass es nicht so sei. Da gehört schon mehr dazu. Und selbst wenn mehr dazu kommt, bewegt man sich dann doch oft nur auf einer Ebene von als eher wahrscheinlich oder eher unwahrscheinlich anzunehmenden Gegebenheiten auszugehen.

Nach allem, was wir derzeit wissen, müssen wir davon ausgehen, dass die Wellenfunktion reales Naturverhalten beschreibt und nicht nur ein mathematisches Tool zur Berechnung von Wahrscheinlichkeiten ist.

Ich kann mir das ja sogar sehr gut vorstellen. Aber dennoch scheint es mir eindeutig momentan nicht so zu sein, als dass man unbedingt zwingend davon ausgehen müsste. Das ist eben wohl nur deine Ansicht. Und wenn man tatsächlich davon ausgehen muss, dann wird sich das auch durchsetzen - du kannst dann doch also ganz beruhigt sein, dich zurücklehnen, und es freudig abwarten. Kein Grund zur Aufregung :)

Viele Grüße nach HongKong
AMC

Hawkwind
29.04.12, 21:53
Nach allem, was wir derzeit wissen, müssen wir davon ausgehen, dass die Wellenfunktion reales Naturverhalten beschreibt und nicht nur ein mathematisches Tool zur Berechnung von Wahrscheinlichkeiten ist.


Im Kontext der Bohmschen oder Everettschen Deutung hast du ja vermutlich sogar recht damit - für minmale statistische oder Kopenhagener Deutungen wohl sicher nicht.

Es ist halt ein Punkt, der debattiert wird.

RoKo
30.04.12, 00:48
Im Kontext der Bohmschen oder Everettschen Deutung hast du ja vermutlich sogar recht damit - für minmale statistische oder Kopenhagener Deutungen wohl sicher nicht.

Es ist halt ein Punkt, der debattiert wird.In der Standard-QM ist die Wellenfunktion kein mathematisches Tool.

Hawkwind
30.04.12, 20:01
In der Standard-QM ist die Wellenfunktion kein mathematisches Tool.

Diese Diskussion wird so langsam langweilig. :(
Ich könnte dich jetzt mit entsprechenden Zitaten zumüllen; eines soll reichen:


The subjective view, that the wave function is merely a mathematical tool for calculating the probabilities in a specific experiment, is a similar approach to the Ensemble interpretation.


aus
http://en.wikipedia.org/wiki/Copenhagen_interpretation

Diese Frage kann nur im Kontext der Deutungen der QM diskutiert werden, und dort dann recht kontrovers. Da die Kopenhagener Deutung immer noch die weitestverbreitete Interpretation ist, kann man sie als "Standard" ansehen. In der Standard-QM ist die Wellenfunktion somit ein Tool, das Berechnungen von Vorhersagen ermöglicht.

Ich verstehe nicht, warum du die diversen Sichten bzw. Deutungen unbedingt verdrängen möchtest.

RoKo
01.05.12, 03:36
Hallo Hawkwind,

kurz zur Klarstellung:
.. Ich verstehe nicht, warum du die diversen Sichten bzw. Deutungen unbedingt verdrängen möchtest.

Mir geht es nicht um die Verdrängung von Sichten und Deutungen, sondern um Klarstellung.

Zu dieser Klarstellung gehört, dass die Standard-QM sich im Laufe der Zeit gewandelt hat. Diesen Wandel kann man nicht an der Begrifflichkeit "Kopenhagener Deutung" ausmachen, sondern an den Postulaten.

Die Standard-QM beruht heute in der Regel auf den Postulaten, die ich oben bereits angeführt hatte. Die Kopenhagener Deutung (KD1.0) beruht hingegen auf den reinen Messpostulaten (Postulate 2,3,4 der Standard-QM) mit der zusätzlichen Vorschrift, Messgeräte müssen klassisch beschrieben werden. (Ein gute Quelle ist hier: Pietschmann, "Quantenmechanik verstehen")

Der wesentliche Unterschied besteht darin, daß in der KD1.0 in der Tat keinerlei Bezug auf eine zu Grunde gelegten Quantenwelt genommen wird. In der Standard-QM gibt es ein minimalistisches Konzept: Ein Quantensystem befindet sich in einem Zustand, der durch eine Wellenfunktion beschrieben wird.

Zu dieser Klarstellung gehört vor allem, daß nach rund 100 Jahren Quantenmechanik zumindest der Fachwelt klar sein sollte:

Observable sind keine Eigenschaften des zu untersuchenden Quantensystems.

Als mit dem Fach vertrauter Mensch darf man deshalb nicht schreiben:
..
einem Elektron kann man vor der Messung überhaupt keinen Zustand zusprechen.
Erst durch die Konfiguration der Messanordnung wird festgelegt, welcher Aspekt des Elektrons nach der Messung in Erscheinung tritt.


Richtig ist:
Einer Observablen kann man vor der Messung überhaupt keinen Wert zusprechen.
Erst durch die Konfiguration der Messanordnung wird festgelegt, welche Observable gemessen wird.

Für Laien sollte man dann fairerweise hinzufügen, dass Observable keine Eigenschaften des Elektrons sind sondern Resultat der Messwechselwirkung.

RoKo
01.05.12, 07:00
Fortsetzung.

Zur Klarstellung gehört aber auch:

Aus physikalischer Sicht sind derzeit die mindestens die KD1.0, die Standard-QM (KD2.0) und die BM konsistent und gleichwertig. Die Wahl steht dem Physiker also frei. Diese physikalischen Standpunkte sind aber zugleich mit erkenntnistheoretischen und philosophischen Standpunkten verbunden, die man mit der freien Wahl übernimmt und die man dann auch mit allen Konsequenzen zu vertreten hat.

Bauhof
01.05.12, 18:46
Solch ein absolutistischer Satz entspricht jedenfalls nicht meinem Verständnis von Kopenhagen. Ich würde es eher so sagen: "Wir gehen von keiner Quantenwelt aus, aber im Grunde wissen wir es nicht." Fragen, die offen sind, die müsen eben auch offen bleiben.

Hallo AMC,

nicht nur Niels Bohr, sondern auch Wolfgang Pauli und Max Born haben ähnliche Aussagen gemacht, die man absolutistisch nennen könnte. Besonders Wolfgang Pauli hat sich hier hervor getan. Er kritisiert dabei die rückschrittlichen Bemühungen von Schrödinger, Bohm, Einstein und anderen. Max Born schreibt dazu auf Seite 143 seines Buches [1] folgendes:

Trotzdem glaube ich nicht, dass Schrödingers Artikel einen positiven Beitrag zur Lösung der philosophischen Schwierigkeiten bedeutet. Es ist nicht leicht für mich, die philosophischen Ansichten eines Freundes zu kritisieren, den ich als großen Gelehrten und tiefen Denker aufrichtig bewundere. Ich will daher zur Verteidigung meiner eigenen Auffassung eine Methode wählen, die zu benutzen auch Schrödinger selber nicht zu stolz ist, nämlich die Berufung auf anerkannte Autoritäten, die meine Ansicht teilen. Zu meinen Zeugen wähle ich Wolfgang Pauli, der allgemein als der kritischste und logisch sowie mathematisch anspruchsvollste unter den Fachgenossen angesehen wird, die zur Entwicklung der Quantenmechanik beigetragen haben. Der folgende Abschnitt stammt aus einem Brief, den ich kürzlich von Pauli erhalten habe:

"Entgegen allen rückschrittlichen Bemühungen (Schrödinger, Bohm usw. und in gewissem Sinne auch Einstein) bin ich gewiss, dass der statistische Charakter der
Ψ-Funktion und damit der Naturgesetze — auf dem Sie von Anfang an gegen Schrödingers Widerstand bestanden haben — den Stil der Gesetze wenigstens für einige Jahrhunderte bestimmen wird. Es mag sein, dass man später, z.B. im Zusammenhang mit den Lebensvorgängen, etwas ganz Neues finden wird, aber von einem Weg zurück zu träumen, zurück zum klassischen Stil von Newton-Maxwell (und es sind nur Träume, denen sich diese Herren hingeben) scheint mir hoffnungslos, abwegig, schlechter Geschmack. Und, könnten wir hinzufügen, es ist nicht einmal ein schöner Traum."

Das lässt keine Fragen offen. Wolfgang Pauli war der schärfste Kritiker seiner Kollegen. Die Pauli-Kritiken haben wesentlich dazu beigetragen, dass bereits vor rund 100 Jahren die großen Fortschritte in der Quantenmechanik gemacht wurden, weil schnell die Spreu vom Weizen getrennt wurde. Den 'Weizen' lieferten Bohr, Born, Pauli, Heisenberg u.a.

Mit freundlichen Grüßen
Eugen Bauhof

[1] Born, Max
Physik im Wandel meiner Zeit. (http://www.amazon.de/Physik-im-Wandel-meiner-Zeit/dp/3528085398/ref=sr_1_1?s=books&ie=UTF8&qid=1335893754&sr=1-1)
Braunschweig 1983. ISBN=3-528-08539-8

RoKo
02.05.12, 00:54
"Kenntnisse, die keiner kennt, sind keine."Erwin Schrödinger, 1935

RoKo
02.05.12, 05:21
Hallo Baufhof, hallo amc

Zitat von Wolfgang Pauli: Entgegen allen rückschrittlichen Bemühungen (Schrödinger, Bohm usw. und in gewissem Sinne auch Einstein) bin ich gewiss, dass der statistische Charakter der
Ψ-Funktion und damit der Naturgesetze — auf dem Sie von Anfang an gegen Schrödingers Widerstand bestanden haben — den Stil der Gesetze wenigstens für einige Jahrhunderte bestimmen wird. (o.a.A)
Naturgesetze beschreiben Änderungszusammenhänge in der Natur - unabhängig davon, ob sie statistischen oder deterministischen Charakter haben.

.. Das lässt keine Fragen offen. Genau. Wolfgang Pauli lässt sich mit diesem Zitat nicht als Zeuge für die Ansichten
a) Es gibt keine Quantenwelt
b) Ψ-Funktion = mathematisches Tool
c) Ψ = Wissen
anführen.

JoAx
02.05.12, 09:53
Hi!

Sorry, dass ich momentan nicht zeitnah antworten kann.


Ich weiß wirklich nicht, was man manche Leute so lesen, um das in Zweifel ziehen zu können.


Das

Beim 'Quantenbilliard' darf man den "Kugeln" (den Quantenobjekten) exakt definierte Orte und Impulse nicht mal theoretisch vor der Messung zuschreiben.ist halt keine ausschöpfende Antwort, Eugen.
Da will sich manch einer bsw. ein Elektron über ein makroskopisches Gebiet verschmiert sehen. Nicht nur bildhaft gesprochen. Wäre das bsw. korrekt?


Gruß, Johann

amc
04.05.12, 22:36
Das lässt keine Fragen offen. Wolfgang Pauli war der schärfste Kritiker seiner Kollegen. Die Pauli-Kritiken haben wesentlich dazu beigetragen, dass bereits vor rund 100 Jahren die großen Fortschritte in der Quantenmechanik gemacht wurden

Hallo Eugen,

es ist auch meine Ueberzeugung, dass eine Rueckkehr zum "klassischen Stil" ein Irrweg, eine Traeumerei ist. Wenn man dieser Ueberzeugung ist, darf und muss man das natuerlich auch klar sagen. Allerdings ist es fahrlaessig, alle Tueren fuer einen, wie auch immer gearteten, zugrundeliegenden Realismus komplett zu verschliessen. Und diese "Hintertuer" hat Pauli in diesem Zitat ebenso zum Ausdruck gebracht:

Es mag sein, dass man später, z.B. im Zusammenhang mit den Lebensvorgängen, etwas ganz Neues finden wird

Klar ist, vom "klassischen Stil" distanziert sich Pauli ausdruecklich. Genaugenommen muss man natuerlich eine "Hintertuer" in alle Richtungen offen lassen. Aber darum geht es mir nicht. In irgendeiner Form muss man ja auch mal seine Ansichten deutlich formulieren. Es geht mir auch nicht so sehr darum, was irgendwer irgendwann gesagt hat, sollte es sicher uns allen nicht gehen, sondern um grundsaetzliche Einstellungen, die ein wissenschaftliches Vorgehen widerspiegeln. Und da sind wir uns bestimmt alle sicher, dass man da bei diesen herausragenden Persoenlichkeiten keinen Zweifel haben muss. Aber man darf eben nicht vergessen, diese "Hintertueren" auch immer mal wieder deutlich zu machen. Wie Bohr sich hier im Verhaeltnis zu anderen verhalten hat, kann ich nicht beurteilen.

Fuer mich persoenlich findet sich dieses wissenschaftliche Vorgehen in besonderer Weise in einer entsprechenden Auslegung der Kopenhagener Interpretation (KI) wieder. Man sollte eben nur soweit versuchen zu gehen, wie man auch schauen kann. Fragen die offen sind, muessen offen bleiben. Wir koennen nicht sagen ob es eine Quantenwelt gibt, aber wir haben bisher keine zwingende Notwendigkeit erkennen koennen, von dieser auszugehen. Bisher spricht alles eher gegen einen zugrundeliegenden, durch die Wellenfunktion beschriebenen real existierenden, physikalischen Vorgang. Nach meinem Bild koennte man auch von einer liberalen Interpretation sprechen - alles kann, aber nichts ist bewiesen. "Bruecken-Interpretation" gefaellt mir auch sehr gut ;) Man kann und muss in alle Richtungen denken, und diese Ueberlegungen muss man auch ernst nehmen, vielleicht liegen sie ja doch Richtig, aber fuer mich erscheint eine in dieser Form gemaessigte KI eine sehr sinnvolle Ansicht zu sein.

Ob wir die "Lebensvorgaenge", von denen Pauli sprach, wohl je verstehen werden? Wie leicht zu verstehen waere die welt doch ohne uns mittendrin. Aber sinnlos, irgendwie sehr sinnlos.

Schoenes Wochenende, AMC

amc
04.05.12, 22:45
Da will sich manch einer bsw. ein Elektron über ein makroskopisches Gebiet verschmiert sehen. Nicht nur bildhaft gesprochen. Wäre das bsw. korrekt?

Hallo Johann,

ueber solche Fragen muss man wohl einfach (noch) schweigen. Scheint mir ein wenig zu sein, als wuerde das Elektron sagen: Du sollst dir kein Bild von mir machen. ;)

Gruesse, AMC

RoKo
05.05.12, 03:27
Hallo amc,

..Bisher spricht alles eher gegen einen zugrundeliegenden, durch die Wellenfunktion beschriebenen real existierenden, physikalischen Vorgang. ..

Was spricht dagegen?

Ich weiss nur, was dafür spricht: Das Prinzip der Energieerhaltung.

In der Quantenmechanik, einer Theorie über die Bewegungsgesetze von Mikroobjekten (v<<c), ist es da nicht anders als in der klassischen Mechanik, einer Theorie über die Bewegung von Makroobjekten (v<<c).

Allgemein ausgedrückt: Damit sich ein Objekt aus dem Quellgebiet A in das Quellgebiet B bewegt, bedarf es zunächst einer potentiellen Energie in A gegenüber B. Diese potentielle Energie wird durch die Bewegung von A nach B übertragen und dort letztlich in andere Energieformen umgesetzt. In dieser Allgemeinheit gibt es keinen Unterschied zwischen klassischer und Quantenmechanik.

Die Schrödinger-Gleichung ist nicht nur zur Information von Physikern eine Energiegleichung. Um aus ihr für einen konkreten Fall die Wellenfunktion zu berechnen, macht man ja nichts anderes als zunächst eine Energiebetrachtung für eine analoge klassische Situation anzustellen und diese mittels Quantisierungsvorschriften in die Quantenmechanik zu übersetzen. Danach hat man dann den energetischen Zustand des Quantenobjektes. Dessen weitere Entwicklung ist ein vollkommen deterministischer Prozess, der leicht an Hand der Wellenfunktion berechnet werden kann. Da gibt es nichts geheimnisvolles.

Das Geheinmissvolle ergibt sich nur, wenn man klassische Vorurteile hat und sich Quantenobjekte als punktförmige "Teilchen" imaginiert und ihnen Eigenschaften andichtet, die sie nach der Theorie garnicht haben können.

RoKo
05.05.12, 05:32
Nachtrag.

Die wesentlichen Grundlagen für die mathematisch strenge Formulierung der Quantenmechanik wurden im Jahr 1932 durch John von Neumann formuliert. Demnach lässt sich ein physikalisches System allgemein durch drei wesentliche Bestandteile beschreiben: Seine Zustände, seine Observablen und seine Dynamik (das heißt durch seine zeitliche Entwicklung).

In der Standard-QM wird eindeutig mit dem ersten Postulat festgestellt:
Ein quantenmechanisches System wird vollständig durch seinen Zustand beschrieben. Observable hat demzufolge das zu betrachtende System nicht. Alle Versuche, den quantenmechanischen Zustand zu messen sind bisher auch gescheitert. Daraus den philosophischen Schluss zu ziehen, dieser Zustand habe keine Entsprechung in der Natur, ist nicht nur philosophisch zweifelhaft, sondern auch physikalisch, da mit dem Energieerhaltungsprinzip unvereinbar.

Sofern in der Standard-QM von Observablen die Rede ist, beziehen diese sich stets auf den Messprozess und sind daher die Observablen des gemeinsamen Zustandes von Quantenobjekt und Messgerät. Dies ist nach 1932 (s.o.) lang und breit erörtert worden. Die genaue Dynamik dieses Prozesses wird in der Standard-QM nicht betrachtet; sie wird per Postulat als unhinterfragbarer Fakt vorausgesetzt.

Im Rahmen der Standard-QM lässt sich jedoch eine ungefähre Vorstellung gewinnen. Sobald das Quantenobjekt mit dem Messgerät in Berührung kommt, verschränken sich die Zustände beider zu einem gemeinsamen Zustand. Wir haben es also mit einem neu entstandenen System zu tun. Dieses hat eine neue Dynamik, die wesentlich eine Kombination der Dynamik des ursprünglichen Quantenobjektes und der Messdynamik ist. Die Gleichung dafür lässt sich aufstellen. Diese zeigt uns zunächst eine Überlagerung aller möglichen Messwerte. Nun kommt der objektive Zufall ins Spiel und verwandelt eine der Möglichkeiten in einen eindeutigen Messwert. Die Gleichung des verschränkten Zustandes wandelt sich daher in eine logische Wenn-dann-Beziehung.

Zum weiteren Verständnis sollte man sich auch mit dem physikalischen Energiebegriff beschäftigen. Demzufolge ist Energie die Möglichkeit, irreversibel zu wirken. Ferner sollten einem die Energieflussbetrachtungen aus der Elektrodynamik (Stichwort Pointing-Vektor) geläufig sein. Betrachtet man die QM unter diesem Aspekt, dann beschreibt die Wellenfunktion die Ausbreitung von Möglichkeiten, irreversibel zu wirken; und ihr Absolutquadrat beschreibt die Wahrscheinlichkeit eines solchen Ereignisses.

Nun kann man geneigt sein, die Ausbreitung von Möglichkeiten als physikalisch nicht real anzusehen. Dagegen spricht jedoch, dass diese Möglichkeiten durch Einsatz unzweifelhaft real existierender makroskopischer Objekte beeinflusst werden kann. Sehr eindrucksvoll ist hier der sogennante Bombentestversuch.

Jan
28.10.12, 13:04
Hallo Mirko,

Ich bin ebenfalls Neuling in diesem Forum - mein erster Beitrag - und finde deine Fragen sehr interessant.

Einige Anmerkungen von meiner Seite:

Quantenmechanische Effekte spielen erst dann eine relevante Rolle, wenn z.B. einige wenige Atome, etc. betrachtet werden. Bei deinem Beispiel mit dem Schmecken von Süß, Sauer, usw ist das nicht der Fall.

Nach meiner Kenntnis ist die Deutung der Quantenmechanik immer noch heftig diskutiert. Was genau passiert bei einer Messung eines quantenmechanischen Systems? Durch eine Messung wird das System auf einen der möglichen Zustände festgelegt. Was passiert aber mit den Anteilen der Wellenfunktion, die anderen Zuständen entsprechen? Eine mögliche Deutung ist die Entstehung von Parallelwelten.

Lg
Jan